PDA

توجه ! این یک نسخه آرشیو شده میباشد و در این حالت شما عکسی را مشاهده نمیکنید برای مشاهده کامل متن و عکسها بر روی لینک مقابل کلیک کنید : رفع اشكال المپیاد نجوم



صفحه ها : 1 [2] 3

پیمان اکبرنیا
01-10-2013, 12:06 AM
خب اون فرض مساله نیست که سمت براش ثابته!اگر اون طوری بگیم خوب ساده میشه تقریبا!
حالا به جز اون مساله من یه سوال دیگه هم دارم.توی یکی از سوالات جهانی در مورد تابش بند گفته بود.میشه بگین تابش بند چیه و چه کار می کنه و کلا محاسباتش چه طوریه؟
ممنون!

تابش بند یک صفحه افقی است که در قسمت بالای پنجره به سمت بیرون تعبیه می کنند تا نور خورشید کمتر وارد اتاق بشه یا اصلا وارد نشه :)

یک شکل سر هم بندی شده توی پینت کشیدم که نمای یک اتاق را از بقل نشون میده و پنجره تابش بند هم داره :دی همان طور که می بینید تابش بند از ورود نور خورشید جلوگیری کرده :)

http://up.avastarco.com/images/ksln0kid3jir7o55h3g.jpg

المپیاد نجوم
01-14-2013, 07:58 PM
بابا دیگه امتحانای ترمم تموم شد بیاین یکم فعال باشیم.من میگم هر روز یه سوال بذاریم بعد مسابقه ای بیایم سوالا رو حل کنیم.هر کی جوابو در آورد یه توضیح کوچیکیم بده و بعد بریم سوال بعدی.
بنیانشم خودم می ذارم اولین سوالو می ذارم(قرار نیست مساله ها حتما سخت یا طولانی باشن فقط مساله باشن!)
سوال:ناظری در عرض جغرافیایی 38 درجه ی شمالی نوک قله ی کوهی را همیشه در کنار ستاره ی نسبتا روشنی می بیند.اگر ناظر دیگری در نوک کوه ناظر اول را در فاصله سمت الراسی 130 درجه مشاهده کند ارتفاع کوه را بر حسب کیلومتر محاسبه کنید.

المپیاد نجوم
01-15-2013, 08:40 PM
بابا چرا هیچ کس دست مارو نمی گیره.
یه سوال آسونتر میذارم(خودم همین الان حلش کزدم هیجان زده شدم گفتم این جا بذارمش)اول اینو حل کنیم بعد رو سوال قبلیه بحث کنیم!
سوال:ناظری در عرض جغرافیایی 21 درجه به هنگام طلوع خورشید سمت ستاره ای روی دایره البروج را برابر سمت خورشید می بیند.تاریخ این پدیده را محاسبه کنید.
راهنمایی نمی تونم کنم چون جواب یه خطه و راهنمایی یعنی جواب!
تا فردا شب همین موقع اگه کسی جواب نده خودم جوابشو می ذارم و اون یکیم اگه تا فردا کسی جواب نداد و خودمم حل نکردم دیگه جوابشو می پرسیم!

A.ALAVI
01-15-2013, 09:09 PM
بابا دیگه امتحانای ترمم تموم شد بیاین یکم فعال باشیم.من میگم هر روز یه سوال بذاریم بعد مسابقه ای بیایم سوالا رو حل کنیم.هر کی جوابو در آورد یه توضیح کوچیکیم بده و بعد بریم سوال بعدی.
بنیانشم خودم می ذارم اولین سوالو می ذارم(قرار نیست مساله ها حتما سخت یا طولانی باشن فقط مساله باشن!)
سوال:ناظری در عرض جغرافیایی 38 درجه ی شمالی نوک قله ی کوهی را همیشه در کنار ستاره ی نسبتا روشنی می بیند.اگر ناظر دیگری در نوک کوه ناظر اول را در فاصله سمت الراسی 130 درجه مشاهده کند ارتفاع کوه را بر حسب کیلومتر محاسبه کنید.
این طرح طرح خیلی خوبیه! اگه دنبال بشه البتّه!
اگه منبع سوالو بگین بهتر نیست؟! حالا عیبی داره که بگیم این سوال از مجموعه تمارین آقای چرتاب بوده؟!
خب چون گفتید اون یکی ساده تره من این یکی رو جواب میدم! :wink:
مگه آسمون نمی چرخه؟! مگه نباید جای هر ستاره به جز ستاره ای که تو یه مکان خاصّه عوض شه؟!
منظور این سوال اینه که اون ستاره،ستاره قطبیه که همیشه مکانش ثابته دیگه!
پس ناظر ارتفاع قلّه ی کوهو ارتفاع ستاره قطبی می بینه دیگه!
ناظر روی قلّه وقتی زاویه ی بین سمت الراس و ناظر را 130 می بینه میشه یه مثلّث تشکیل داد!
حالا فکر کردن خیلی ساده میشه!
http://up.avastarco.com/images/9eb9r389yylgn00yo4ge.png (http://up.avastarco.com/)
حالا با نوشتن را بطه ی سینوس مثلّث مسطّحه میشه ارتفاع کوهو بدست آورد که تقریباً 180 کیلومتر میشه!

A.ALAVI
01-15-2013, 09:31 PM
بابا چرا هیچ کس دست مارو نمی گیره.
یه سوال آسونتر میذارم(خودم همین الان حلش کزدم هیجان زده شدم گفتم این جا بذارمش)اول اینو حل کنیم بعد رو سوال قبلیه بحث کنیم!
سوال:ناظری در عرض جغرافیایی 21 درجه به هنگام طلوع خورشید سمت ستاره ای روی دایره البروج را برابر سمت خورشید می بیند.تاریخ این پدیده را محاسبه کنید.
راهنمایی نمی تونم کنم چون جواب یه خطه و راهنمایی یعنی جواب!
تا فردا شب همین موقع اگه کسی جواب نده خودم جوابشو می ذارم و اون یکیم اگه تا فردا کسی جواب نداد و خودمم حل نکردم دیگه جوابشو می پرسیم!
یه کمک که شاید شما رو گمراه کنه!:دی دایره ی عظیمه ای که از دو نقطه رد میشه یکتاست!
البتّه به شرط اونکه فاصله ی دو نقطه 180 نباشه!

celestial boy
01-16-2013, 04:34 PM
سلام.
آخییییییییییش بالاخره منم از امتحانات نجات پیدا کردم.با یه مسابقه حل مسئله موافقم.
خب با اجازه سوال دومتون که آسون تر بود.
شرط هم سمت شدن دو جسم اینه که خط واصلشون بر افق عمود باشه.خب خورشید و اون ستاره روی دایره البروجن.پس باید تاریخ عمود شدن دایره البروجو پیدا کرد.من یکم ناشیانه عمل کردم نفهمیدم چی شد.از نقطه اعتدال استفاده کردمو یه مثلث سمت الراس-اعتدال و برخورد استوا با نصف النهار (؟؟؟!!!!) درست کردم.بعد با سینوس ها یه کارایی کردم.در اومد 26 روز اینور و اونور اعتدال.درسته؟راه اصلیش چیه؟؟

المپیاد نجوم
01-16-2013, 05:00 PM
جوابتون درسته!راه حلتون مثل منه؟من آخه درست نفهمیدم راهتونو!
من از Zچرخون استفاده کردم و با توجه به عمود بودن دایره البروج و افق در اون لحظه و با توجه به این که قطباشونم عمودن مساله رو حل کردم.اگه تو راه حل من اشکالی می بینید تذکر بدید که بنویسم و جوابو بذارم.
حالا سوال بعد:ستاره ای در شهری در سمت الراس دیده میشه.همزمان دو ناظر a,bارتفاع این ستاره را 75 درجه اندازی می گیرند.ناظر aدر شرقی ترین نقطه و ناظر bدر غربی ترین نقطه نسبت به شهر قرار گرفته اند.فاصله ی بین این دو ناظر 2250 کیلومتره!میل ستاره چیه؟
آیا ستاره براشون غروب می کنه؟اگه می کنه کی؟(چه مدت بعد برای هر کدوم؟)

A.ALAVI
01-16-2013, 06:20 PM
حالا سوال بعد:ستاره ای در شهری در سمت الراس دیده میشه.همزمان دو ناظر a,bارتفاع این ستاره را 75 درجه اندازی می گیرند.ناظر aدر شرقی ترین نقطه و ناظر bدر غربی ترین نقطه نسبت به شهر قرار گرفته اند.فاصله ی بین این دو ناظر 2250 کیلومتره!میل ستاره چیه؟
آیا ستاره براشون غروب می کنه؟اگه می کنه کی؟(چه مدت بعد برای هر کدوم؟)
من میل ستاره رو تقریباً 70- بدست آوردم غلط نیست؟!
با این اوصاف ستاره برای اون دو تا دور قطبی میشه!
غلط نیست!؟ اشتیباه محاسباتی ندارم؟!

المپیاد نجوم
01-16-2013, 06:37 PM
فکر می کنم اشتباه محاسباتی دارید.من عددم با شما فرق داره!یه بار دیگه بنویسید.بقیه هم همین عددو در آوردن؟؟؟

Siavash9874
01-16-2013, 11:21 PM
حدودا ۶۵ درجه میل رو بدست آوردم ، که غروب هم می‌کنه .

راه حل : فاصله یکی‌ از ناظر‌ها تا شهر ۱۵ درجس ، فاصله ۲تا ناظر حدودا ۲۰ درجه . ناظر‌ها و شهر‌ها همه در یک عرضه جغرافیا ای‌ بودن ، یه ۲تا معادله بود که با ماشین حساب سلو کردم :دی

A.ALAVI
01-17-2013, 12:33 AM
حدودا ۶۵ درجه میل رو بدست آوردم ، که غروب هم می‌کنه .

راه حل : فاصله یکی‌ از ناظر‌ها تا شهر ۱۵ درجس ، فاصله ۲تا ناظر حدودا ۲۰ درجه . ناظر‌ها و شهر‌ها همه در یک عرضه جغرافیا ای‌ بودن ، یه ۲تا معادله بود که با ماشین حساب سلو کردم :دی
شما مطمئنّی که حتماً اون سه تا شهر تو یه عرض جغرافیایین؟! من میگم وقتی شرقی غربی ترین حالت اتّفاق میفته که زاویه ی بین شهر مرکزی- شهر a یا b - قطب شمال سماوی 90 بشه! (معادلشو بنویسیم و مشتق بگیریم می بینیم بیشترین تفاوت طول جقرافیایی وقتیه که اون زاویه 90 بشه!! :دی )
به نظر من سوال منظورشو از شرقی یا غربی ترین نقطه درست بیان نکرده!
من شکل زیر تو ذهنمه که تو اون a فاصله ی بین دو شهر و تقریباً 21 درجه هست!)
http://up.avastarco.com/images/obq7wqe962jgka248xrg.png (http://up.avastarco.com/)

المپیاد نجوم
01-17-2013, 10:47 PM
جواب آقای علوی درست بود اگر مشکلی باشه جوابشو می نویسم و میذارم اگر نه هم که هیچی.این سوالم که میذارم آخرین سوالیه که من میذارم بعدش شماها هم سوال بذارین منم حل کنم.
اگر در کهکشانی چگالی متناسب با عکس مجذور فاصله تغییر کنه مدت زمان فروریزش رو برای اون پیدا کنین.اگر توی گرفتن انتگرال به مشکل برخوردید بپرسید!

Siavash9874
01-18-2013, 10:37 AM
http://up.avastarco.com/images/s85skm0w3uzf56vwtoey.png
v همون سرعت هستش ، در آخر v رو dr/dt قرار میدیم و انتگرال میگیریم

erfan bayat
01-20-2013, 07:40 PM
بابت نبودم تو این مدت عذر میخوام مشکلاتی پیش اومده بود
خوب پیرو حرفه المپیاد نجوم منم یه سوال میگم:امکان اینکه تحلیل ابعادی تو مرحله اول بدن هست این سوال تحلیل ابعادی تا حدی:
http://up.avastarco.com/images/vp08pqg5j7xq1yz1yi0.jpg (http://up.avastarco.com/)

المپیاد نجوم
01-20-2013, 08:47 PM
این آپلود سنتر اعصاب منو خرد کرد!
جواب آ[ر سوال آقای بیاتو من 1.53ضربدر ده به توان منفی 36 در آوردم آگر اشتباهه بگید اصلاح کنم.
سوال کهکشان هم جواب آخرش میشه یک روی رادیکال 8.gkکه kهمون صورت رابطه ی یک بر آر دو در محاسبه ی چگالیه!
بازم مشکلی بود بهم بگید لطفا!
ممنون!

المپیاد نجوم
01-28-2013, 08:06 PM
حالا که دوباره می بینم اصلا جواب اون سوال آقای بیات این نیست.اگه یه سری فرضا کنیم میشه 3.5 تقریبا.منتها به نظرم میاد این سوال اصلا درست نباشه.آخه چه طوری یه طرفه تساوی فقط یه چیزی با بعد جرم هست و اون طرف طول و اینا هم داریم.حس می کنم یا رابطه ناقصه یا بعضی از او ثابتای بی بعد بی بعد نیستن یا من هنگ کردم!

المپیاد نجوم
02-16-2013, 05:34 PM
سلام یه سوال کروی دوره هست که اگه ممکنه یک نفر راه حل کامل تا جواب آخرشو برام بذاره.ممنون میشم.
فردی در عرض جغرافیایی 50 درجه ی شمالی در پای برجی ستاره ای با میل 30 درجه را در حال طلوع می بیند.او در جهتی به سمت جنوب غربی با سرعت 360 کیلومتر بر ساعت با ماشین خود به مدت 360 دقیقه در مسیری مستقیم می راند و در اثر این حرکت عرض جغرافیایی او نیم درجه عوض می شود.در این هنگام او همان ستاره را در نوک همان برج می بیند.ارتفاع برج را بیابید!
ممنون!

Siavash9874
02-16-2013, 11:16 PM
خوب فکر می‌کنم راه حالش اینه : مسافتی که ماشین رفته رو محاسبه می‌کنیم ، طول کمانش می‌شه حدودا ۱۹ درجه .

توی خود سوال گفت که نوک برج می‌‌بینه ، پس اگه سمت ستاره یا اون شهر اولی‌ رو پیدا کنیم سوال حال تقریبا .

سمتش حدودا ۷۶ درجه هستش .

حالا یدونه مثلث pzx رو حل می‌کنیم ، میل ستاره ، عرضه جغرافیأ شهر و سمت ستاره رو داریم . ارتفاع به دست میاد ۲۹.۲۷ درجه.

حالا یه مثلث مسطحه رو حل می‌کنیم ، یکی‌ از اضلاع شعاع زمین هستش ، یکی‌ شعاع+ارتفاع ، زاویه بین ۲تا شهر ، و ارتفاع به علاوه ۹۰

فقط یکم ارتفاع برج زیاد شد :-؟؟ ۲۰۴۶ km

پیمان اکبرنیا
02-16-2013, 11:55 PM
دوستان دقت کنید که برای حل این سوال باید در نظر داشته باشید که در زمانی که ماشین حرکت می کند، آسمان هم می چرخد و ستاره جا به جا می شود :) اثر اون را هم باید در نظر گرفت :)

Siavash9874
02-17-2013, 01:51 PM
من دقیقا نمی‌فهمم چطوری ستاره بالای برج هستش . بدون در نظر گرفتن قضیه برج کلا ، ستاره ارتفاع ۱۲ درجه خواهد داشت برای ناظر ( زاویه ساعتی‌ ستاره در این زمان ۹۰ درجه کرده ، تغییرات طول جغرافیأ رو هم داریم ، زاویه ساعتی‌ ناظر به دست میاد . ) ولی‌ اینکه میگه بالای برج دقیقا یعنی‌ چی‌ ؟

جواب سوال آقای بیات هم ۲.۹ می‌شه ، b هم ۱.۵ می‌شه

COLDFIRE
02-18-2013, 11:06 PM
منم مدتی درگیر این سوال بودم... ولی متاسفانه قادر به حلش نشدم... دوستان اگر کسی تونست حل کنه لطف کنه راه حل رو بگه...
ـــــــــــــــــــــــــ ـــــــــــــــــــــــــ ـــــــــــــــــــــــــ ـــــــــــــــــــــــــ
در ضمن اگه چرخش زمین رو در نظر نگیریم ارتفاع ستاره برای ناظر بعد از حرکت منفی میشه(که دلیلش هم واضحه!)، پس اصلا بدون درنظر گرفتن چرخش زمین امکان حل سوال وجود نداره

Laya
02-18-2013, 11:58 PM
به نظر من راه حل سوال بايد اين باشه، اولا بخاطر حركت ما سمت ستاره تغيير مي كنه چون سمت كاملا به عرض جغرافيايى مربوطه پس يك تغيير در ارتفاع ستاره به خاطر تغيير در سمته. و همچنين گردش زمين در ٦ ساعت هم باعث تغيير ارتفاع ستاره در عرض جغرافيايى ٤٩.٥ درجه مى شود. ارتفاع ستاره تقريباً ١٣ درجه بدست مي ايد. سپس براى بدست اوردن ارتفاع برج يك مثلث داريم به يك ضلع شعاع زمين يك ضلع ديگر به اندازه شعاع+ارتفاع و يك زاويه ٩٠+١٣ درجه (مقابل ضلعى كه مساوى شعاع +ارتفاع است )و يك زاويه ديگر به اندازه ى زاويه بين دو شهر كه تقريباً ١٩ درجه است ارتفاع هم تقريباً ٩٠٠ مي شود. زاويه مقابل ضلع به اندازه ى شعاع زمين مساوى است با(١٣+٩٠) -١٨٠

المپیاد نجوم
02-19-2013, 09:30 PM
ا خوب پس درست حل کردم.منم همینو در آوردم ولی این قدر بزرگ بود شک کردم!
یه سوال دیگه که عدد آخرو مشکل دارم اینه.
امسال 19 آذر ماه شاهد ماه گرفتگي بوديم كه از نوع كلي بود. طبق اعلام منجمين آغاز اين پديده در ساعت 16:15 به
وقت ايران شروع شد ولي به دليل طلوع نكردن ماه ناظران ديرتر شاهد اين پديده بودند. در اين صورت محاسبه كنيد براي
ناظري در تبريز( fi =38,l=46) چه مدت زماني(چند دقيقه) پس از آغاز ماه گرفتگي ماه به طور كامل طلوع كرده
است(قرص ماه كامل ديده مي شود)؟ ( مدار زمين را دايروي فرض كنيد . نصف النهار استاندارد ايران داراي طول جغرافيايي
52,5 درجه است)
من اومدم گفتم با توجه به اختلاف طول جغرافیایی تبریز و استاندارد در تبریز طلوع جرم سماوی 26 دقیقه زودتر اتفاق می افتد و از طرفی اندازه ی ظاهری ماه هم که 0.5 درجه است و بایک تناسب می توانیم ببینیم چند دقیقه طول می کشد تا کامل طلوع کند و عددم شد 33 ولی جواب 49 هست.
چیزی که بهش شک کردم اینه که اون رابطه فقط برای خورشید باشه و در اون صورت تاریخ و ... رو هم باید وارد کنیم و مساله یکم سخت تر میشه.درسته؟

Laya
02-20-2013, 10:08 AM
فكر مى كنم بايد راه حلش اين باشه، اولا اختلاف زمانى ماه گرفتگى بين ساعت رسمى و تبريز كه ٢٦ دقيقه مي شه و ثانياً بايد ميل ماه در ١٩ اذر رو بدست بياريم چون ماه گرفتگى داريم بنابراين ماه روى دايرة البروج قرار دارد و ميل ماه با ميل خورشيد برابر مي باشد ميل ماه مي شود ٢٣٠٢- . حالا بايد زمانى را بدست بياوريم كه ماه ٠،٥ درجه از افق بالاتر است جواب ٤h٣٨m مي شود بايد اين جواب را با ١٢ جمع كنيم كه بدست مى ايد ١٦h٣٨m اگر انرا از ١٦h١٥m كم كنيم داريم ٢٣ دقيقه و وقتى ٢٣ را با٢٦ جمع كنيم جواب مى شود ٤٩ دقيقه.

strn_au
04-08-2013, 10:26 PM
سوال: با فرض ماده غالب بودن جهان ، قرمز گرایی بیابید که در آن نسبت نوترون به پروتون 1 به 5 باشد!
کسی ایده ای داره؟

erfan bayat
04-08-2013, 10:56 PM
سوال: با فرض ماده غالب بودن جهان قرمز گرایی بیابید ک در آن نسبت نوترون ب پروتون 1 به 5 باشد!
کسی ایده ای داره؟
باید از تابع توزیع بولتزمن استفاده کنی! توی فصل اول فیلیپس هستش.از اینجا دمای متناظر با اون قرمز گرایی بدست میاد. از طرفی اثبات میشه که دما با معکوس ضریب مقیاس رابطه داره که با استفاده از اینکه دمای حال 2.725 کلوینه و ضریب مقیاس الان 1 هستش ضریب مقیاس اون موقع و درنتیجه قرمز گرایی بدست میاد.اگه متوجه نشدین میتونم روابطشم واستون بزارم

strn_au
04-09-2013, 07:38 AM
ممنون میشم اگه روابطشم بگید!

erfan bayat
04-09-2013, 09:27 PM
ممنون میشم اگه روابطشم بگید!
ابتدا با استفاده از رابطه ی بولتزمان(فصل 13 کتاب فیزیک ستاره ها فیلیپس) و با استفاده از اینکه نسبت تعداد نوترون به پروتون رو داریم دما بدست میاد(تو رابطه دلتاm اختلاف جرم پروتون نوترون و k ثابت بولتزمان)فک کنم دما نزدیکه9^10 کلوین در میاد.
http://up.avastarco.com/images/502cfmsypeuxodppe8.png (http://up.avastarco.com/)
بعد از اینکه دما به دست اومد با توجه به اینکه تو کیهان شناسی اثبات میشه دما با معکوس ضریب مقیاس را بطه داره که با استفاده از اینکه دمای حال 2.725 کلوینه و ضریب مقیاس الان 1 هستش ضریب مقیاس اون موقع به دست میاد و خوده ضریب مقیاس با قرمز گرایی رابطه داره که میشه از روی اون قرمز گراییو بدست آورد
http://up.avastarco.com/images/adx95110hf7nf2najuly.png (http://up.avastarco.com/)
http://up.avastarco.com/images/aseg5shkar4a350py79q.png (http://up.avastarco.com/)

saeed77
04-09-2013, 10:01 PM
1-نسبت روشنایی دو ستاره چه رابطه ای با قطر ظاهری آنها در آسمان دارد؟
2-نسبت اختلاف تعداد ستارگان مشاهده شده در آسمان چه رابطه ای با اختلاف قدر دارد؟(برای مثال به ازای افزایش قدر ستارگانی که در آسمان میتوان مشاهده کرد از 5 به 6 چند ستاره بیشتر دیده میشوند؟)

arashgmn
04-10-2013, 07:57 PM
1-نسبت روشنایی دو ستاره چه رابطه ای با قطر ظاهری آنها در آسمان دارد؟
2-نسبت اختلاف تعداد ستارگان مشاهده شده در آسمان چه رابطه ای با اختلاف قدر دارد؟(برای مثال به ازای افزایش قدر ستارگانی که در آسمان میتوان مشاهده کرد از 5 به 6 چند ستاره بیشتر دیده میشوند؟)
1 - اگر روشنایی مربوط به تابش بولومتری رو در نظر بگیریم ، روابط مثل زیر می شن :

http://up.avastarco.com/images/qm43pbbjjbp8tsyo2o2.png (http://up.avastarco.com/)

2- ستارگانی که ما از یه قدر خاص می بینیم به روشنایی ستاره بستگی داره. مثلا ممکنه ستاره ای پرنور و دور با ستاره ای کم سو و نزدیک هم قدر باشه. برای محاسبه ی دقیق تعداد ستاره ها در یک قدر ، باید ستاره هایی رو پیدا کنیم که نسبت درخشندگی به توان دو ی فاصله شون از ما ، یه مقدار ثابت خاص باشه. اما چون درهشندگی هیچ تابعیتی به فاصله ی ستاره از ما نداره، برای همین این کمیت رو خیلی ساده نمیشه حساب کرد.

اما یه جور ساده سازی برای محاسبه ی این سوال وجود داره که بهش می گن قضیه ی اسلیگر؛ میان می گن که فرض کنید ستاره ها همگی به یک اندازه درخشندگی دارن و چگالی عددی (تعداد ستاره ها بر واحد حجم) برای کل فضا ثابته. حالا نسبت تعداد ستاره هایی که از قدر m+1 می درخشن به تعداد ستاره هایی که از قدر m می درخشن چه قدره ؟ که جوابش حدودا میشه 4 .

المپیاد نجوم
04-10-2013, 08:01 PM
1.روشنایی رو می تونین با کمک نوشتن رابطه ی L=4PIR^2SIGMAT^4 که بر 4PId^2 تقسیم می شود به اندازه ی ظاهری مجسم مربوط کنید.
2.این چیزی که میگید بهش میگن قضیه ی اسلیگر.بسته به این که فرض مساله چی باشه فرق می کنه.اگر خیلی ساده در نظر بگیرین که همه ی ستاره ها یک قدر مطلق دارند وهمگی به طور یکنواخت پراکنده شده اند آنگاه نسبت تعداد میشه نسبت حجم ها چون چگالی عددی ثابته و میره. حالا نسبت حجم ها هم چون توزیع کرویه همون نسبت فواصل به توان 3 هست.
از طرفی نسبت فواصل را با مدول فاصله می توان به دست آورد حالا ساده می کنیم و می بینیم که با فرض های بالا جواب میشود:3.98!
حالا اگه دوست دارید این سوالو حل کنید.
با فرض این که دمای موثر سطحی تمام ستاره ها برابر دمای موثر سطحی خورشید باشند و شعاع 70 درصد آنها برابر شعاع خورشید و شعاع 20 درصد آنها 0.01 شعاع خورشید و شعاع 10 درصد آنها 100 برابر شعاع خورشید باشد و با این فرض که هر نوع از این سه دسته به صورت یکنواخت در عالم توزیع شده اندتعداد ستارگان با قدر کمتر از 27 چند برابر تعداد ستارگان با قدر کمتر از 6 است؟اگر فاصله ی متوسط بین دوستاره مانند خورشید 1.5 پارسک باشد تقریبا چه تعداد ستاره را با چشم غیر مسلح می توان دید؟

این پست با پست آقای گلمحمدی همزمان شد!

erfan bayat
04-10-2013, 08:27 PM
http://up.avastarco.com/images/eup27ccpgdllmf2tbd9n.png (http://up.avastarco.com/)
--------------------------------------------------------------------------------------------------------
تصحیح:
نسبت تعداد ستاره های با قدرکمتر از m به تعداد ستاره های با قدرکمتر ازm+1

المپیاد نجوم
04-10-2013, 09:38 PM
جواب سوالی که گذاشتم هم اینه!
البته اگه اشتباهه بهم بگید!
http://up.avastarco.com/images/kiswx0k2z2fhrarqv.jpeg

arashgmn
04-11-2013, 08:16 PM
گویا کمتر از 1 ماه به مرحله 2 باقی مونده. اگر سوال های مرحله دویی دارید، بپرسیدها !!! نگید نگقتیم ...

--------------
من ؛ بعد از مرحله 2 : گفتم یا نگفتم ؟! گفتم یا نگفتم ؟! گفتم یا نگفتم ؟! :))

المپیاد نجوم
04-11-2013, 09:04 PM
ممنون که گفتین سوالامونو بپرسیم.
من یه سوال دارم.
در چه محدوده ای از عرض های جغرافیایی در کره ی زمین ناظران می توانند عبور تلسکوپ فضایی هابل از قرص ماه را مشاهده کنند؟
زاویه ی میل مداری تلسکوپ فضایی هابل نسبت به استوا 28 درجه و ارتفاع آن از سطح زمین 570 کیلومتر است.مدار ماه را دایروی با شعاع 384000کیلومتر و زاویه آن با دایره البروج را 5 درجه فرض کنید.
ممنون

A.ALAVI
04-11-2013, 10:50 PM
بدون در نظر گرفتن شعاع ماه و با در نظر گرفتن شعاع زمین برابر 6380 کیلومتر؛بازه ای بین مثبت و منفی 37.1 بدست میارم!

erfan bayat
04-12-2013, 07:48 AM
ممنون که گفتین سوالامونو بپرسیم.
من یه سوال دارم.
در چه محدوده ای از عرض های جغرافیایی در کره ی زمین ناظران می توانند عبور تلسکوپ فضایی هابل از قرص ماه را مشاهده کنند؟
زاویه ی میل مداری تلسکوپ فضایی هابل نسبت به استوا 28 درجه و ارتفاع آن از سطح زمین 570 کیلومتر است.مدار ماه را دایروی با شعاع 384000کیلومتر و زاویه آن با دایره البروج را 5 درجه فرض کنید.
ممنون
حد این اتفاق جاییه که هر دوتا شون تو بالاترین نقطه نسبت به استوا باشن بعد با وصل کردن ماه به هابل و به سطح زمین و حل یه مثلث میشه عرض رو به دست آورد

المپیاد نجوم
04-12-2013, 10:23 AM
عدد من 31.5 نمیشه میشه لطفا راهتونو بگین اشتباهمو بفهمم؟؟؟
ممنون

A.ALAVI
04-12-2013, 10:54 AM
http://up.avastarco.com/images/cxk2shltieru9sbo61g5.jpg (http://up.avastarco.com/)
همانظور که در شکل می بینیم برای تعیین حدّ بالاترین عرض،ما دو متغیّر داریم. یکی مکان هابل و دیگری مکان ماه.
برای فرض شروع می کنیم از عرض جغرافیایی 90 درجه خطّی به بالا ترین مکان ممکن هابل وصل کرده و در نتیجه میبینیم علاوه بر این که این خط از داخل زمین می گذرد(!)،خط مکان ممکن ماه را نیز قطع نمی کند! همینطور ادامه می دهیم تا به جایی برسیم که خطّ عبور کننده از هابل،خط ماه را نیز قطع کند!
بعد از محاسبات لازم و دانستن این که این رویداد نسبت به استوا قرینه است بنده از مثبت 37.1 تا منفی آن بدست می آورم!

المپیاد نجوم
04-12-2013, 04:36 PM
الان پاسخنامه ی آزمونمونو گرفتم مثل این که اشتباه نمی کردم جواب اون سوال 84.07 میشه که حالا از منفی تا مثبتش میشه.
راه حلم می ذارم اگه بچه ها دوست دارن چک کنند!
برای به دست آوردن حداکثر عرض‌ جغرافیایی که گذر تلسکوپ هابل از مقابل ماه قابل مشاهده است باید فرض کنیم که مطابق شکل زیر میل زمین مرکزی ماه حداقل و میل زمین مرکزی هابل حداکثر است (و برای حداقل عرض جغرافیایی برعکس). یعنی اگر ماه در پایین‌ترین نقطه مدار خود نسبت به استوای زمین و هابل در بالاترین نقطه باشد، با توجه به شکل زیر می‌توان محلی که گذر در آن قابل مشاهده است را محاسبه کرد:


در مثلث omh یعنی هابل-ماه- مرکز زمین، زاویه hom برابر است با میل مداری هابل (28 درجه) به علاوه قدر مطلق میل حداقل ماه (28.5 درجه). ضلع om برابر 384000 کیلومتر و ضلع oh برابر با فاصله هابل از مرکز زمین یعنی 6950 کیلومتر است. با توجه به این اطلاعات زاویه ohm با استفاده از رابطه سینوسها برابر با 122.63 درجه به دست می‌آید. حال در مثلث ohk مقدار زاویه ohk برابر است با متمم زاویه ohm که برابر خواهد بود با 57.37 درجه.
در نهایت در مثلث ohk با مشخص بودن 2 ضلع و یک زاویه، سایر اضلاع و زوایا محاسبه می‌شوند. زاویه مورد نیاز برای ما زاویه hok است که برابر با 56.1 درجه به دست می‌آید. پس حداکثر عرض جغرافیایی که گذر در آن مشاهده می‌شود برابر خواهد بود با 28+56.1 یا به عبارتی 84.1 درجه. به همین ترتیب طبق تقارن، حداقل عرض جغرافیایی برابر با 84.1 درجه است پس در محدوده عرض‌های 84.1+ تا 84.1 – امکان مشاهده گذر هابل از مرکز قرص ماه وجود دارد.

saeed77
04-12-2013, 06:17 PM
دوستان از جواب هایی که فرستادید متشکرم ولی منظور من از سوال 1 این بود که اگر تصویری از منطقه ای از آسمان به ما دادند و گفتند که نسبت روشنایی بین 2 ستاره که در تصویر مشخص شده رو بیابید شما چه جوابی به سوال میدهید؟( ما از روی تصویر فقط میتوانیم نسبت قطر ظاهری 2 ستاره رو بیابیم و قطر ظاهری هر ستاره هم با روشنایی دریافتی ccd از اون ستاره رابطه داره. در ضمن اگر بین قطر ظاهری 1 ستاره در تصویر ccd با روشنایی خودش رابطه ی مشخصی وجود داره ممنون میشم اگه بگید.)

A.ALAVI
04-12-2013, 07:34 PM
الان پاسخنامه ی آزمونمونو گرفتم مثل این که اشتباه نمی کردم جواب اون سوال 84.07 میشه که حالا از منفی تا مثبتش میشه.
راه حلم می ذارم اگه بچه ها دوست دارن چک کنند!
برای به دست آوردن حداکثر عرض‌ جغرافیایی که گذر تلسکوپ هابل از مقابل ماه قابل مشاهده است باید فرض کنیم که مطابق شکل زیر میل زمین مرکزی ماه حداقل و میل زمین مرکزی هابل حداکثر است (و برای حداقل عرض جغرافیایی برعکس). یعنی اگر ماه در پایین‌ترین نقطه مدار خود نسبت به استوای زمین و هابل در بالاترین نقطه باشد، با توجه به شکل زیر می‌توان محلی که گذر در آن قابل مشاهده است را محاسبه کرد:


در مثلث omh یعنی هابل-ماه- مرکز زمین، زاویه hom برابر است با میل مداری هابل (28 درجه) به علاوه قدر مطلق میل حداقل ماه (28.5 درجه). ضلع om برابر 384000 کیلومتر و ضلع oh برابر با فاصله هابل از مرکز زمین یعنی 6950 کیلومتر است. با توجه به این اطلاعات زاویه ohm با استفاده از رابطه سینوسها برابر با 122.63 درجه به دست می‌آید. حال در مثلث ohk مقدار زاویه ohk برابر است با متمم زاویه ohm که برابر خواهد بود با 57.37 درجه.
در نهایت در مثلث ohk با مشخص بودن 2 ضلع و یک زاویه، سایر اضلاع و زوایا محاسبه می‌شوند. زاویه مورد نیاز برای ما زاویه hok است که برابر با 56.1 درجه به دست می‌آید. پس حداکثر عرض جغرافیایی که گذر در آن مشاهده می‌شود برابر خواهد بود با 28+56.1 یا به عبارتی 84.1 درجه. به همین ترتیب طبق تقارن، حداقل عرض جغرافیایی برابر با 84.1 درجه است پس در محدوده عرض‌های 84.1+ تا 84.1 – امکان مشاهده گذر هابل از مرکز قرص ماه وجود دارد.
من متوجّه شدم که در پاسخ به سوال اشتباهاً زاویه ی ماه را از استوا حساب کردم و نه از دایره البروج! :shy:
امّا یک اشتباه در حلّ‌ شما وجود دارد!
با توجّه به اینکه من شکلی در حلّ‌ شما نمی بینم پس حدث می زنم که منظور از k همان نقطه ی جغرافیایی مد نظر ما باشد! در نتیجه با توجّه به حلّ‌ شما در مثلّث hok زاویه ی ohk برابر 57.37 و زاویه ی hok برابر 56.1 بدست آوردید! پس زاویه ی okh باید برابر 66.5 درجه باشد! امّا این زاویه از 90 درجه کمتر است و این بدین معناست که ناظر عرض 84.1 در ارتفاع منفی 23.5 آنها را رصد می کند که امکان پذیر نیست!
این اشتباه در حلّ شما از جایی شروع شد که چون سینوس یک زاویه با سینوس مکمّل آن زاویه برابر است،وقتی ماشین حساب برای زاویه ی okh عدد 66.5 ‌را گزارش نمود،شما با تفریق نمودن 57.4 و 66.5 از عدد 180،عدد 56.1 را مستقیماً‌ گزارش نمودید!! در صورتی که شما باید برای زاویه ی okh عدد 113.5 که مکمّل عدد 66.5 است را گزارش کنید! با توجّه به راه حلّ‌ شما زاویه ی koh برابر با 9.1 درجه و در نتیجه بازه ی عرض ها از مثبت 37.1 تا منفی 37.1 خواهد بود!
باز به خاطر اشتباه گذشته ی خود عذر می خواهم!

المپیاد نجوم
04-12-2013, 08:26 PM
این باید که میگید که باید مکمل زاویه گزارشبشه از کجا میگید؟؟؟

A.ALAVI
04-12-2013, 08:43 PM
این باید که میگید که باید مکمل زاویه گزارشبشه از کجا میگید؟؟؟
برای اینکه فکر کنم باید در ضمین این که هابل و ماه هم خط میشن؛ باید بالای افق باشن تا ناظر بتونه ببیندشون!‌
شما چی فکر می کنید؟!

strn_au
04-13-2013, 11:51 AM
حجم عرقچین چطوری بدست میاد؟

starscream4002
04-13-2013, 02:29 PM
حجم عرقچین چطوری بدست میاد؟

مگه عرقچین حجم داره؟ اگه منظورتون حجم اون شکل مخروط ماننده، حجمش می شه http://latex.codecogs.com/gif.latex?v=%5Cfrac%7B1-%5Ccos%5Ctheta%7D%7B2%7D%5Ctimes%20%5Cfrac%7B4%5Cp i%7D%7B3%7Dr%5E%7B3%7D راه به دست آوردنش تقسیم کردن مساحت عرقچین بر مساحت کل کره و ضرب کردن اون در حجم کل کره هستش.

س.ا.

A.ALAVI
04-13-2013, 03:20 PM
مگه عرقچین حجم داره؟ اگه منظورتون حجم اون شکل مخروط ماننده، حجمش می شه http://latex.codecogs.com/gif.latex?v=%5Cfrac%7B1-%5Ccos%5Ctheta%7D%7B2%7D%5Ctimes%20%5Cfrac%7B4%5Cp i%7D%7B3%7Dr%5E%7B3%7D راه به دست آوردنش تقسیم کردن مساحت عرقچین بر مساحت کل کره و ضرب کردن اون در حجم کل کره هستش.

س.ا.
البتّه جناب انصارین راه جذّاب تر و زاویه فضایی بدست آوردن این حجم را بیان کردند،امّا شما از طریق انتگرال از مساحت عرقچین نسبت به المان dR هم به همین جواب می رسید!

strn_au
04-13-2013, 03:58 PM
مگه عرقچین حجم داره؟ اگه منظورتون حجم اون شکل مخروط ماننده، حجمش می شه http://latex.codecogs.com/gif.latex?v=%5Cfrac%7B1-%5Ccos%5Ctheta%7D%7B2%7D%5Ctimes%20%5Cfrac%7B4%5Cp i%7D%7B3%7Dr%5E%7B3%7D راه به دست آوردنش تقسیم کردن مساحت عرقچین بر مساحت کل کره و ضرب کردن اون در حجم کل کره هستش.

س.ا.

ممنون! منم اولش با انتگرال گیری رفتم اما فک کردم شاید غلط باشه!
واسه ی این سوال میخاستم! ک البته با منظرشم مشکل دارم!
سوال: فرض کنید جو زمین ارتفاع محدودی به شعاع زمین دارد!چند درصد از جو قابل رویت برای یک ناظر استوایی در نیمه شب اعتدال بهاری روشن است؟ (هر ذره که در مقابل خورشید قرار میگیرد روشن شده و در تمام جهات بازتاب میکند. از جذب نور صرف نظر کنید)

arashgmn
04-14-2013, 12:25 AM
دوستان از جواب هایی که فرستادید متشکرم ولی منظور من از سوال 1 این بود که اگر تصویری از منطقه ای از آسمان به ما دادند و گفتند که نسبت روشنایی بین 2 ستاره که در تصویر مشخص شده رو بیابید شما چه جوابی به سوال میدهید؟( ما از روی تصویر فقط میتوانیم نسبت قطر ظاهری 2 ستاره رو بیابیم و قطر ظاهری هر ستاره هم با روشنایی دریافتی ccd از اون ستاره رابطه داره. در ضمن اگر بین قطر ظاهری 1 ستاره در تصویر ccd با روشنایی خودش رابطه ی مشخصی وجود داره ممنون میشم اگه بگید.)
فکر نکنم بشه بدون محاسبه های تقریبی راجع به روشنایی صحبت کرد . نمی دونم ! مطمئن نیستم !

برای مربوط کردن شار دریافتی و قطر ظاهری بر روی تصویر ccd ، من تا حالا رابطه ای ندیدم. شاید وجود داشته باشه . (اگر تابعی باشه احتمالا به عمق چاه پتانسیل ccd، تابع نقطه پخش ، دید ، و دهانه ربط داره)

تو کدوم امتحان از این سوالا دادن؟! یا داده ها و فرض های اضافی می خواد ، یا این که ... :دی

saeed77
04-14-2013, 11:29 AM
یه راه حلی بود که برای یه مثال تو کلاس مطرح شد.ولی فکر نکنم برای نسبت بین 2 ستاره اطلاعاتی که خواستید لازم باشه چون این ها خصوصیات ccd و برای هر دو ستاره صادق است و تنها چیزی که اختلاف قطر را به وجود میاره اختلاف روشنایی.(شاید لازم باشه راحت تر به سوال نگاه کنید)
ممنون از جوابتون

strn_au
04-14-2013, 10:44 PM
مگه علت وجود تعدیل زمان ، بیضوی بودن مدار زمین هم نیست ؟

پیمان اکبرنیا
04-14-2013, 11:04 PM
مگه تعدیل زمان بخاطر بیضوی بودن زمین هم نیست وجود داشتنش؟!

سلام

تعدیل زمان هم به دلیل بیضوی بودن مدار زمین هست و هم به دلیل زاویه 23.5 درجه ای مسیر حرکت خورشید یعنی دایره البروج با استوا :)

strn_au
04-15-2013, 07:48 AM
سلام

تعدیل زمان هم به دلیل بیضوی بودن مدار زمین هست و هم به دلیل زاویه 23.5 درجه ای مسیر حرکت خورشید یعنی دایره البروج با استوا :)


حالا اگه بگن تعدیل رو پیدا کنید بدون توجه به خروج از مرکز مدار زمین ینی چی؟ فقط 23.5 رو در نظر بگیریم ؟

arashgmn
04-15-2013, 07:01 PM
حالا اگه بگن تعدیل رو پیدا کنید بدون توجه به خروج از مرکز مدار زمین ینی چی؟ فقط 23.5 رو در نظر بگیریم ؟

بله . شما کافیه که مدار زمین رو دایره ای در نظر بگیرید که 23.5 درجه نسبت به دایره البروج میل داره ...

amin-z
04-16-2013, 06:26 PM
سلام به همگی
ببخشید معادله ی سرعت شعاعی ومماسی چطوری اثبات میشن؟؟؟؟

starscream4002
04-16-2013, 07:10 PM
سلام به همگی
ببخشید معادله ی سرعت شعاعی ومماسی چطوری اثبات میشن؟؟؟؟

سرعت شعاعی و مماسی چه جسمی؟ هر ستاره ای تو آسمون نسبت به ما سرعت شعاعی و مماسی داره. ضمنا کدوم فرمول؟

س.ا.

المپیاد نجوم
04-16-2013, 07:28 PM
من در حد جواب دادن نیستم ولی اگر درست متوجه سوالتون شده باشم سرعت شعاعی در مدار به شکلی به دست می آید که شما باید از رابطه ی مربوط به فاصله در هر لحظه در مدار مشتق بگیرید و با کمک تعریف تکانه زاویه ای آن را ساده کنید.رابطه ی سرعت مماسی هم که h/r میگذارین و حل می کنین توی تاپیک مکانیک اینا اثبات شدند.
بازم میگم اگه اشتباه فهمیدم سوالتونو از افراد دیگه بپرسین!

strn_au
04-17-2013, 06:38 AM
بیشترین کشیدگی سیارات داخلی تو جای خاصی از مدارشون اتفاق میفتن؟

erfan bayat
04-17-2013, 08:09 PM
نه جای خاصی اتفاق نمی افته در واقع تعریف اصلی همون بیشترین فاصله زاویه ای سیاره و خورشیده و هرجایی از مدار ممکنه رخ بده البته با فرض داییروی بودن مدارها.
فکر کنم منظور شما مدار بیضوی باشه)؟!( در اینصورت سوال جالبیه و احتمالا جای خاصی از مداره که فکر کنم به ویژگی دو مدار بیضوی ربط داره.ایده ای که واسه ی حلش دارم اینه که زمین و سیاره داخلی رو دو جای مجهول از مدارشون فرض کن بعد با خور شید و این دو سیاره یه مثلث درست کن و سعی کن واسه زاویه کشیدگی یه رابطه بر حسب یه متغییر دراری بعد مشتق بگیری صفر بزاری تا ماکزیمم کشیدگی در بیاد

strn_au
04-17-2013, 08:31 PM
khob y joore dg am mishe goftesh fk konam! andaze zavie khob ba zele robe roosh ziad mishe dg! asanam be joft madar k rabt nadare faghat be madare sayyare dakheli rabt dare! k un ja k bishtarin faselase az khorshid oas zaviasham bishtarine dg pas yani ammalan bayad vaghty bashe k sayyare dakhelie too owje! na? albate ehtemalanam eshteb mikonam nemidoonam! =)

خب یجور دیگه هم میشه گفتش فکر کنم! اندازه زاویه خب یا ضلع ربع روش زیاد میشه دیگه! اصن هم به جفت مدار که ربط نداره فقط به مدار سیاره داخلی ربط داره! که اونجا که بیشترین فاصلشه از خورشید چس زاویشم بیشترینه دیگه پس یعنی عملا باید وقتی باشه که سیاره داخلیه تو اوجه! نه؟ البته احتمالا هم اشتب می کنم نمیدونم! =))


پیام مدیر:
دوست گرامی در فروم آوا استار پستها باید به فارسی نوشته شوند. لطفا زین پس پستهای خود را با حروف فارسی تایپ نمایید. در صورت نداشتن حروف فارسی می توانید از سایتهای مبدل حروف مثل بهنویس استفاده نمایید
با تشکر

A.ALAVI
04-17-2013, 09:30 PM
خب یجور دیگه هم میشه گفتش فکر کنم! اندازه زاویه خب یا ضلع ربع روش زیاد میشه دیگه! اصن هم به جفت مدار که ربط نداره فقط به مدار سیاره داخلی ربط داره! که اونجا که بیشترین فاصلشه از خورشید چس زاویشم بیشترینه دیگه پس یعنی عملا باید وقتی باشه که سیاره داخلیه تو اوجه! نه؟ البته احتمالا هم اشتب می کنم نمیدونم! =))


[/
دقّت کنید که خط مماس به بیضی به این راحتیا نیستا! یعنی این اتّفاق هم به مکان سیّاره ی اوّل و هم ستاره ی دوّمی ربط داره حتّی اگه مدار سیّاره ی بیرونی رو دایروی بگیریم!!!

strn_au
04-18-2013, 08:32 PM
اصولا بیرونی رو دایره ای میگیرن!

saeed77
04-19-2013, 10:38 PM
سوال 121 مجموعه سوالات صدرا صدرالدینی (صفحه 27 سوال دوم) (گراویتون) اگه زاویه ی تتا (ابیراهی) کم نباشد و نتوان تقریب زد و چگونه میتوان مسئله را در حالت کلی و بدون هیچگونه تقریب حل کرد؟

المپیاد نجوم
04-19-2013, 11:19 PM
با نیرو بهش نگاه نکن.سعی کن با کمک گشتاور و تکانه زاویه ای حلش کنی.مثل اصطکاک دینامیکی!

A.ALAVI
04-19-2013, 11:28 PM
اصولا بیرونی رو دایره ای میگیرن!
بله اصولاً بیرونی رو دایروی می گیرن!
منظور من این بود که مدار بیرونی چه بیضی و چه دایروی باشه باز محاسباتش به این راحتی که شما بیان می کنید نیست!!

saeed77
04-20-2013, 03:23 PM
با نیرو بهش نگاه نکن.سعی کن با کمک گشتاور و تکانه زاویه ای حلش کنی.مثل اصطکاک دینامیکی!
سعی کردم ولی به بن بست رسیدم آخه در هر دو جهت نیرو (تتا هت و آر هت ) باعث افزایش شعاع میشه!!!
در ضمن تکانه زاویه ای هم ثابت نیست.چجوری ازش استفاده کنم؟؟؟

Siavash9874
04-20-2013, 08:47 PM
در راستای‌ آر هت نسبتش به نسبت گرانش واقعا کمه ، در راستای‌ تتا هت سرعت زاویه رو کاهش میده .

سرعتش هم که توی هر لحظه دایره‌ای هستش

تعریف گشتاور هم که مشتق تکانه زاویه ای است !

saeed77
04-20-2013, 08:58 PM
در راستی‌ آر هت نسبتش به نسبت گرانش واقعا کمه ، در راستی‌ تتا هت سرعت زاویه رو کاهش میده .

سرعتش هم که توی هر لحظه دایره‌ای هستش

تعریف گشتاور هم که مشتق تکانه است !

اولا که در راستای تتا هت سرعت رو افزایش میده

ثانیا تقریبیش خودم حل کردم ولی اگه تقریب تزنیم (تتا زیاد باشه) اونوقت به مشکل میخورم.

ممنون از جوابت

Siavash9874
04-20-2013, 09:25 PM
نیرو در راستی‌ منفی هستش ، سرعت زاویهِ باید کاهش پیدا کنه .

و سرعت مداری چقدر میتون باشه که ما به مشکل بر بخوریم در رابطه با تقریب ؟
ولی‌ در کًل معادله دیفرانسیلی که بدون تقریب به وجود میاد فکر نمیکنم به این راحتی‌ حل بشه . میتونید توی سایت ولفرم آلفا برسی‌ کنید که جواب معادله چی‌ می‌شه

المپیاد نجوم
04-20-2013, 09:26 PM
سلام من راه حل خودمونوشتم.امیدوارم کمک کنه!
http://upload.tehran98.com/img1/a8jk6hqmi502w2907d3.jpeg

soroosh
04-23-2013, 05:22 PM
حالا یه سوال که فکر کنم از مجموعه سوالات عطا باشه
در چه روزی از سال اختلاف بین زمان اذان صبح (فلق) و طلوع خورشید کم ترین مقدار است ؟

arashgmn
04-26-2013, 11:41 AM
حالا یه سوال که فکر کنم از مجموعه سوالات عطا باشه
در چه روزی از سال اختلاف بین زمان اذان صبح (فلق) و طلوع خورشید کم ترین مقدار است ؟

این سوال به نظرم خیلی ممارست می خواد برای حل. احتمالا باید کلی نسبت مثلثاتی رو توی هم ضرب و تقسیم کرد و ساده کرد تا جواب به دست بیاد. البته ممکنه تقریب هایی هم بشه زد که کار رو خیلی آسون کنه. :22:
-----------------
من یه بار به سرعت نوشتم ، و به یک معادله درجه 6 برخوردم! البته درست و غلطش رو نمی دونم چون چک نکردمش. تقریب هم نزدم . شاید با تقریب درجه 2 هم نشه ...:7:

پیمان اکبرنیا
04-26-2013, 02:06 PM
این سوال به نظرم خیلی ممارست می خواد برای حل. احتمالا باید کلی نسبت مثلثاتی رو توی هم ضرب و تقسیم کرد و ساده کرد تا جواب به دست بیاد. البته ممکنه تقریب هایی هم بشه زد که کار رو خیلی آسون کنه. :22:
-----------------
من یه بار به سرعت نوشتم ، و به یک معادله درجه 6 برخوردم! البته درست و غلطش رو نمی دونم چون چک نکردمش. تقریب هم نزدم . شاید با تقریب درجه 2 هم نشه ...:7:

یک راهش اینه که فرض کنیم تغییرات زاویه سمت الراسی زاویه نسبتا کوچکی هست و از فرمول تقریبا زوایای کوچک بریم. یعنی فرض کنیم دلتا z برابر است با یک زاویه کوچک. بعد از فرمول تغییرات زاویه ساعتی بر حسب تغییرات فاصله سمت الراسی که در فصل 2 نجوم کروی اومده استفاده کنیم. در این صورت فکر کنم اثبات میشد که این اختلاف زمانی در زمانی که میل خورشید مینیموم است اتفاق می افته یعنی روی اول زمستون.

saeed77
04-26-2013, 05:10 PM
یک راهش اینه که فرض کنیم تغییرات زاویه سمت الراسی زاویه نسبتا کوچکی هست و از فرمول تقریبا زوایای کوچک بریم. یعنی فرض کنیم دلتا z برابر است با یک زاویه کوچک. بعد از فرمول تغییرات زاویه ساعتی بر حسب تغییرات فاصله سمت الراسی که در فصل 2 نجوم کروی اومده استفاده کنیم. در این صورت فکر کنم اثبات میشد که این اختلاف زمانی در زمانی که میل خورشید مینیموم است اتفاق می افته یعنی روی اول زمستون.

بزرگان ببخشند در حضورشون نظر میدم ولی فکر کنم در میانه ی راه حل سوال 31 فصل 2 کروی به یه مثلث میرسیم که 3 جزء رو داریم و جزء دیگش میل است که بقیشم میدونید.(با z چرخان حل میشه)

mhfire
04-27-2013, 04:16 PM
سلام

یه راه ساده اینه که 3 تا روز رندوم انتخاب کنی بعد با آزمایش‌و خطا برسیبه کمترین مقدار

بعضی وقت ها این راه حل ها بهتر جواب میدن !

soroosh
04-27-2013, 07:03 PM
سلام

یه راه ساده اینه که 3 تا روز رندوم انتخاب کنی بعد با آزمایش‌و خطا برسیبه کمترین مقدار

بعضی وقت ها این راه حل ها بهتر جواب میدن !

این که فقط بتونیم جواب رو پیدا کنیم مهم نیست
اگر توی امتحان یه همچین سوالی بیاد اگر با این روش بخوایم حل کنیم نمره نمیگیریم

strn_au
04-28-2013, 07:00 AM
سلام!
میشه لطفا سوال 81 از مجموعه سوالات آقای چرتاب رو حل کنید! ممنون

همچنین سوال 107 رو! :d

برای حساب کردن مساحت عرق چینی که 3 نقطه روی اون رو با فاصله هاشون که یکسانند داده باید چی کار کرد؟

mohammad23
04-29-2013, 12:35 PM
سلام به همه خسته نباشید
ببخشید میشه در رابطه با اختلاف منتظر فرمول های مهم و کاربردی برای حل سوالاشو بگین
ممنون میشم.

arashgmn
04-29-2013, 07:35 PM
سلام به همه خسته نباشید
ببخشید میشه در رابطه با اختلاف منتظر فرمول های مهم و کاربردی برای حل سوالاشو بگین
ممنون میشم.

اختلاف منظر کلا فقط همین یه فرمول رو داره که :

فاصله ( برحسب پرسک ) = معکوس ( زاویه اختلاف منظر ( بر حسب ثانیه قوس) )

البته یه اختلاف منظر هم هست که توی نجوم کروی پیداش میشه که چند نوعه خودش. ولی پایه ی همه ی این اختلاف منظر ها ، نگریستن به یه جسمه از منظرهای مختلف !

به این پست (http://wiki.avastarco.com/index.php?title=%D8%A7%D8%AE%D8%AA%D9%84%D8%A7%D9% 81_%D9%85%D9%86%D8%B8%D8%B1)از ویکی نجوم و به این پست (http://en.wikipedia.org/wiki/Parallax) از ویکی پدیا توجه کنید. (البته هم پوشانی هم دارند کمی ;) )

arashgmn
04-29-2013, 08:03 PM
سلام!
میشه لطفا سوال 81 از مجموعه سوالات آقای چرتاب رو حل کنید! ممنون

همچنین سوال 107 رو! :d

برای حساب کردن مساحت عرق چینی که 3 نقطه روی اون رو با فاصله هاشون که یکسانند داده باید چی کار کرد؟

برای سوال 81 باید از قسمت تاریک زحل (مخصوصا سایه ای که رو حلقه ها افتاده) استفاده کنی.

برای سوال 107 هم باید این نکته رو قبلا دیده باشید که : زاویه ای که سرعت مماسی و سرعت فضایی از دید کانون با هم می سازن ، در سهمی ، نصف زاویه ی آنومالی (بی هنجاری) حقیقیه . با استفاده از این و یه کم هندسه مسئله حل میشه. جوابش هم میشه رادیکال (دو بارابر اومگا × ال) !

برای حساب کردن مساحت این عرق چین ، باید زاویه راس عرق چین رو پیدا کرد. چون فاصله ها(ی یکسان) بر روی دایره عظیمه سنجیده میشن ، پس میشه یک کسینوس ها نوشت که دوضلعش همون زاویه راس دایره عظیمه ، و یک ضلعش هم همون فاصله یکسان باشه. از این جا زاویه عرق چین به دست میاد و مسئله حل میشه.

payam57
05-02-2013, 09:36 PM
درود یه سوال
یک سیستم دو تایی از دو ستاره a و b با نسبت روشنایی 2 تشکیل شده است و نمی توانیم انها را از هم تفکیک کنیم و مقدار قدر ظاهری ان دو روی هم 5 می باشد. قدر ظاهری هر یک را به دست اورید؟

arashgmn
05-05-2013, 04:34 PM
درود یه سوال
یک سیستم دو تایی از دو ستاره a و b با نسبت روشنایی 2 تشکیل شده است و نمی توانیم انها را از هم تفکیک کنیم و مقدار قدر ظاهری ان دو روی هم 5 می باشد. قدر ظاهری هر یک را به دست اورید؟
فرض کنید قدر ستاره ی a برابر با ma باشه و قدر b برابر با mb . حالا رابطه قدر رو برای یکی از این ستاره ها و قدر مجموع کل (m=5) بنویسید. روشنایی کل ، برابره با مجموع روشنایی های دو ستاره a و b . پس عبارت داخل لگاریتم ، که نسبت روشنایی کل به روشنایی یکی از ستاره هاست ، رو داریم. (نسبت روشنایی دو ستاره 2 بود) حالا قدر یکی به دست میاد. همین کارو برای اون یکی ستاره میشه انجام داد و قدر اونم پیدا کرد. یه راه دیگه هم هست که بیایم دو ستاره a و b رو با هم مقایسه کنیم و از نسبت رو شنایی قدر ستاره دیگری رو پیدا کنیم.

univers
06-14-2013, 09:19 PM
سلام یه سوالی برام پیش اومده لطف می کنین جواب بدید:می دانیم 1pc=206265AU این عدد 206265 از کجا می یاد یعنی چه محاسباتی انجام می دهیم تا به دست بیاد؟
یه پیشنهاد هم داشتم اگه میشه مثل اون زمانی که آقای بیات اینجا بودن شما هم هر هفته یک سوال مطرح کنین تا بقیه روش فکر کنن. ممنون

arashgmn
06-14-2013, 09:45 PM
سلام یه سوالی برام پیش اومده لطف می کنین جواب بدید:می دانیم 1pc=206265AU این عدد 206265 از کجا می یاد یعنی چه محاسباتی انجام می دهیم تا به دست بیاد؟
یه پیشنهاد هم داشتم اگه میشه مثل اون زمانی که آقای بیات اینجا بودن شما هم هر هفته یک سوال مطرح کنین تا بقیه روش فکر کنن. ممنون
خیلی هم پیشنهاد خوبیست ! استقبال می نماییم. :دی

تعریف پارسک اینه: اگر اختلاف منظر یک ستاره ، از دید ما 1 ثانیه قوس باشه ، فاصله اون ستاره رو یک پارسک تعریف می کنیم.

اگه کتانژانت زاویه 1 ثانیه رو حساب کنی ، به همون عدد 206265 می رسی (البته این عدد رند شده و یخورده اعشار هم داره) ;)

univers
06-15-2013, 12:06 AM
سلام ببخشید دوباره سوال می پرسم ولی من یکم گیج شدم دقیقا با چی تو ماشین حساب بزنم که این عدد ظاهر بشه. ممنون از راهنماییتون

پیمان اکبرنیا
06-15-2013, 12:44 AM
سلام ببخشید دوباره سوال می پرسم ولی من یکم گیج شدم دقیقا با چی تو ماشین حساب بزنم که این عدد ظاهر بشه. ممنون از راهنماییتون

بگذار مفهوم پارسک را با این مثلث توضیح بدم:

http://up.avastarco.com/images/43dqldhk3up4l1uf3ti.jpg

این یک مثلث قائم الزاویه است که یک ضلعش 1 واحد نجومی (شعاع مدار زمین) و ضلع دیگرش 206265 واحد نجومیه. در این صورت اون زاویه کوچیکه معادله با 1 ثانیه قوسی :) به اون 206265 واحد نجومی هم میگیم 1 پارسک :) یعنی ما انقدر اون ضلع بزرگه رو کشیدیم تا اون زاویه بشه 1 ثانیه قوس بعد طول ضلع را گذاشتیم 1 پارسک که حساب کردیم دیدیم شد 206265 واحد نجومی.

یعنی تانژانت زاویه 1 ثانیه قوس معادل است با 1/206265 :)

به همین سادگی :)

arashgmn
06-15-2013, 12:45 PM
اینم سوال :

میانگین زمانی بردار شعاع یک سیاره در مدار بیضوی را در طول یک دوره تناوب به دست آورید.

توضیح : ما توی بعضی از مسائل نیاز داریم بدونیم که سیاره به متوسط کجای مدار خودش قرار داره. یعنی اگه ما به صورت تصادفی ، به تعداد زیاد به مدار سیاره نگاه کنیم ، سیاره رو بیشتر توی کدوم موقعیت از مدارش می بینیم. این عملا میانگین زمانی بردار شعاع هستش. یعنی <r>

m.Sadat
06-15-2013, 01:00 PM
اینم سوال :

میانگین زمانی بردار شعاع یک سیاره در مدار بیضوی را در طول یک دوره تناوب به دست آورید.

توضیح : ما توی بعضی از مسائل نیاز داریم بدونیم که سیاره به متوسط کجای مدار خودش قرار داره. یعنی اگه ما به صورت تصادفی ، به تعداد زیاد به مدار سیاره نگاه کنیم ، سیاره رو بیشتر توی کدوم موقعیت از مدارش می بینیم. این عملا میانگین زمانی بردار شعاع هستش. یعنی <r>
با اجازه دوستان من جوابو میذارم :

http://up.avastarco.com/images/tvkrcq1ktxdfwhmaqrmg.gif

univers
06-15-2013, 01:43 PM
منظورتون از <r> همون انتگرال r هست بین بازه ی t1 وt2 هست؟

arashgmn
06-15-2013, 02:06 PM
منظورتون از <r> همون انتگرال r هست بین بازه ی t1 وt2 هست؟
بله . منظور اینه :

http://up.avastarco.com/images/fkel78svtkqoi9b086l.png (http://up.avastarco.com/)

که t1 برابر با صفر و t2 برابر با T(دوره تناوب مداری) هستش.

univers
06-15-2013, 03:15 PM
سلام من اگه بخوام جوابم رو به صورت عکس یعنی با فرمت jpeg اینجا بزارم چه جوری باید این کارو انجام بدم.لطفا راهنماییم کنید. ممنون

starscream4002
06-15-2013, 08:25 PM
سلام من اگه بخوام جوابم رو به صورت عکس یعنی با فرمت jpeg اینجا بزارم چه جوری باید این کارو انجام بدم.لطفا راهنماییم کنید. ممنون

اول عکسو اینجا(up.avastarco.com) آپلود کن و بعد تو پستت به وسیله ی گزینه ی قرار دادن تصویر(از طریق لینک) عکسو قرار بده.

س.ا.

univers
06-15-2013, 09:11 PM
سلام من مراحل کار رو می نویسم ببنید درسته یا نه اگه اشتباه بود راهنماییم کنید(هر چند خودم می دونم اشتباهه!!!!!)
به جای dt از تکانه زاویه ای واحد جرم(h) جا گذاری می کنیم و بعد از معادله ی بیضی مشتق می گیریم و از آن dتتا را به دست می آوریم.

arashgmn
06-16-2013, 12:02 AM
سلام من مراحل کار رو می نویسم ببنید درسته یا نه اگه اشتباه بود راهنماییم کنید(هر چند خودم می دونم اشتباهه!!!!!)
به جای dt از تکانه زاویه ای واحد جرم(h) جا گذاری می کنیم و بعد از معادله ی بیضی مشتق می گیریم و از آن dتتا را به دست می آوریم.
باید به رابطه ای برسید که بشه راحت اونو محاسبه کرد. این راه شما گرچه غلط نیست اما محاسبه اش خیلی سخت میشه.

m.Sadat
06-16-2013, 12:51 AM
باید به رابطه ای برسید که بشه راحت اونو محاسبه کرد. این راه شما گرچه غلط نیست اما محاسبه اش خیلی سخت میشه.
با تشکر از آقا آرش برای طرح این سوال
در این سوال اگه از معادله کپلر استفاده کنی خیلی ساده میشه

univers
06-16-2013, 07:47 PM
میشه راهنماییم کنید چطوری از معادله کپلر استفاده کنم چون من زیاد سوال حل نکردم اگه میشه راهنماییم کنید. ممنون T/π2t=E- e sin E

arashgmn
06-17-2013, 08:30 AM
میشه راهنماییم کنید چطوری از معادله کپلر استفاده کنم چون من زیاد سوال حل نکردم اگه میشه راهنماییم کنید. ممنون T/π2t=E- e sin E
اگر r رو برحسب E بنویسی ، و dt رو با استفاده از مشتق معادله کپلر جایگذاری کنی ، به جاهای خوبی می رسی. ;)

univers
06-18-2013, 12:51 PM
سلام من سوال رو حل کردم میشه جواب رو بزارید اینجا که باهم چک کنیم. ممنون

arashgmn
06-18-2013, 02:32 PM
سلام من سوال رو حل کردم میشه جواب رو بزارید اینجا که باهم چک کنیم. ممنون
بهتره هر کسی جوابشو بذاره. آخرسر همه جواب ها رو با هم مقایسه می کنیم و جواب درست رو اعلام می کنیم. چون ممکنه هنوز کسی باشه که حل نکرده باشه سوالو.

برای کسانی که نمی دونن سوال چی بود ؛ متن سوال :


میانگین زمانی بردار شعاع یک سیاره در مدار بیضوی را در طول یک دوره تناوب به دست آورید.

توضیح : ما توی بعضی از مسائل نیاز داریم بدونیم که سیاره به متوسط کجای مدار خودش قرار داره. یعنی اگه ما به صورت تصادفی ، به تعداد زیاد به مدار سیاره نگاه کنیم ، سیاره رو بیشتر توی کدوم موقعیت از مدارش می بینیم. این عملا میانگین زمانی بردار شعاع هستش. یعنی <r>

http://up.avastarco.com/images/fkel78svtkqoi9b086l.png

univers
06-19-2013, 10:28 PM
سلام منم می خوام دو تا سوال درباره ی میانگین زمانی بزارم :
1-میانگین زمانی اندازه ی نیروی وارد شده به یک جسم که در یک مدار بیضوی با خروج از مرکز بسیار کوچک در حال حرکت است را به دست آورید.
راهنمایی:باید به این رابطه برسید: f>=GMm/a^2>
2-میانگین زمانی انرژِی پتانسیل را برای جسمی در مدار بیضی به دست آورید.
راهنمایی:معلومه باید به چی برسید!!!!!!!!!!!!!!انرژی مدار بیضی

univers
06-21-2013, 01:53 AM
در مورد سوال 2 باید بگم که انتگرال dθ/1+e cos θ دربازه ی بین 2π تا 0 برابر 2π/√1-e^2- 0 (رادیکال روی همه ی عبارت است.).
در مورد سوال 1 نیز باید بگم که دور میانگین زمانی باید یک قدر مطلق قرار داده بشه. و توضیح ذیگه اینکه حکم سوال درسته.!!!!!!

[/URL]http://up.avastarco.com/images/us6wfc19y98mnsymqq2a.jpg (http://up.avastarco.com/images/us6wfc19y98mnsymqq2a.jpg)
[URL="http://up.avastarco.com/images/agp62dxpocr6fs9zxxti.jpg"]http://up.avastarco.com/images/agp62dxpocr6fs9zxxti.jpg (http://up.avastarco.com/images/agp62dxpocr6fs9zxxti.jpg)
جواب سوال 1 هست.امیدوارم که درست باشه.فقط ببخشید که کیفیت عکس ها و دست خط بنده خیلی پایین است.

A.ALAVI
06-21-2013, 04:08 AM
با سلام!
این سوالایی که شما مطرح کردین از مشهورترین سوالات میانگین زمانی هستند که هر کدوم متد خودشونو دارن!
برای میانگین زمانی نیرو بهتره که ما از راه زیر بریم:
http://up.avastarco.com/images/kn4ii7abj668lu868l.png (http://up.avastarco.com/)
چرا که از گرفتن انتگرال های غیر معمولی جلوگیری میکنه و نیاز به حالت خاص نداره!
امّا برای میانگین انرژی پتانسیل مداری،بهترین کار همون کار قرار دادن ((r=a(1-eCos(E هستش که با همون تغییر متغیّری که شما برای نیرو نوشتین میشه به دستش آورد!

univers
06-21-2013, 12:52 PM
ممنون از جوابی که دادین ولی من این سوالات رو طرح نکردم از کسی گرفتم چون همه میدونن این سوالات خیلی معروف هستند.ولی جمله ی آخر حرفتون رو نفهمیدم.من برای حل سوال دوم از تکانه زاویه ای واحد جرم استفاده کردم.خوشحال می شم روش حل سوال شما رو هم بدونم.

univers
06-21-2013, 02:26 PM
سلام یه سوال داشتم اگه میشه راه حل اونو بگید:
بنا بر باور مصریان آب رود نیل زمانی بالا می آید که ستاره شعرای یمانی همزمان با خورشید طلوع کند.تاریخ این رویداد را به دست آورید.
مشخصات ستاره شعرای یمانی: 16- =ra=6 46 δ

المپیاد نجوم
06-21-2013, 02:57 PM
آها این سوال رو سعی نکنین از توابع وارون اینا استفاده کنین باید از مفهوم lst و ارتباطش با بعد استفاده کنین و توی مثلث کروی معروف خورشید هم چهارجزئی بنویسی باز یه رابطه به دست میاری که توش بعد خورشید هست و همه چیز بر حسب بعد میشه.حالا ساده می کنی.اگه می خوای حلو بذارم.

A.ALAVI
06-21-2013, 04:15 PM
بنده از راه زیر برای میانگین زمانی انرژی پتانسیل مداری استفاده کردم!
http://up.avastarco.com/images/nxrw3hb4wfj7lm0hnv9.png (http://up.avastarco.com/)
مشتاقم راه شما رو هم بدونم!

univers
06-21-2013, 04:21 PM
سلام . المپیاد نجوم اگه میشه حل سوال رو بزارید.

m.Sadat
06-21-2013, 04:35 PM
سلام یه سوال داشتم اگه میشه راه حل اونو بگید:
بنا بر باور مصریان آب رود نیل زمانی بالا می آید که ستاره شعرای یمانی همزمان با خورشید طلوع کند.تاریخ این رویداد را به دست آورید.
مشخصات ستاره شعرای یمانی: 16- =ra=6 46 δ
برای حل این سوال به عرض جغرافیایی محل نیاز نداریم ؟

A.ALAVI
06-21-2013, 04:52 PM
این سوال فکر کنم سوال جناب صدرالدینی باشد چرا که در اونجا هم از ذکر کردن عرض جغرافیایی محلّ رصد خبری نبود! امّا با یه سرچ تو ویکیپدیا در مورد قاهره و فرض کردن اینکه محلّ رصد قاهره با عرض جغرافیایی تقریباً 30 درجه ی شمالش هستش جلو می ریم!
http://up.avastarco.com/images/wvfw750sujpejn0oyni.png (http://up.avastarco.com/)
پس فهمیدیم زاویه ساعتی اعتدال بهاری با LST محل برابره و برابر 1h24m می باشد!
حالا یک مثلّث کروی تشکیل می دیم:
http://up.avastarco.com/images/6pb7jv7q0gm3ul47en4u.png (http://up.avastarco.com/)
که در این شکل ها خطّ آبی استوای سماوی و رنگ قرمز دایرة البروج هستش!
بقیه ی نقاط هم خورشید،نقطه ی شرق و اعتدال بهاری هستند.
پس یه چهار جزئی می نویسیم تا به جواب برسیم:
http://up.avastarco.com/images/rd11ok91p4b7fsxah33x.png (http://up.avastarco.com/)

univers
06-21-2013, 07:39 PM
راستی برای اینکه شکلی خوبی بکشیم به نظر شما بهتره از چه نرم افزاری استفاده کنیم و اینکه چطوری باید اینجا قرارش بدیم؟

اینم راه من:
لینک عکس :
http://up.avastarco.com/images/ajrdflbwyv9p77awni6p.jpg

عکس :
http://up.avastarco.com/images/ajrdflbwyv9p77awni6p.jpg (http://up.avastarco.com/images/ajrdflbwyv9p77awni6p.jpg)
باز هم شرمنده برای کیفیت پایین عکس.

A.ALAVI
06-21-2013, 09:44 PM
ببینید جواب های شما هر دو درست هستند امّا نکته ای وجود داره و اون فلسفه ی المپیاده!
المپیاد یعنی بدون راه های پیچ واپیچ به ته سوال برسی!
اون راهنمایی انتگرال که گفتید،اگه نباشه احتمالاً گرفتن اون انتگرال زیاد آسون نیست!
راه شما راه ایده ای و قشنگیه امّا نکته اینه که شما تو صورت سوال اصلاً از این راهنمایی صحبت نکردید!
مثلاً همه ی سوالات نجوم کروی رو میشه با برداری حل کرد و نمی شه همه ی سوالای برداری رو با کروی حل کرد!
این یعنی کروی متد به درد نخوریه؟! نه! تازه خیلی ایده ای تر و کوتاه تره!
پس هر سوال کوتاه ترین راه خودشو داره در عین حال که ممکنه کلّی راه حلّ دیگه داشته باشه!

univers
06-22-2013, 07:10 PM
قبول دارم راه حل سوال دوم با اون راهنمایی سخته ولی مشکل سوال اول رو نفهمیدم.

A.ALAVI
06-22-2013, 10:32 PM
قبول دارم راه حل سوال دوم با اون راهنمایی سخته ولی مشکل سوال اول رو نفهمیدم.
ببینید شما تو حلّ سوال اوّل اومدین فرض کردین که خروج از مرکز خیلی کوچیکه و در نتیجه توی انتگرال تقریب زدین. من نمی دونم این کار غلطه یا درست،امّا دارم میگم وقتی با این راحتی میشه حلّش کرد و توی نتیجه ی انتگرال و نه توی خود انتگرال تقریب زد، چرا از این راه استفاده نکنیم؟!
به نظر شما مصحّح با دیدن هر کدوم از راه حل ها چه حسّی بهش دست میده؟!
هر دو تا راه شما ایده ای و قشنگن امّا بازم میگم، اون متد اصلی و فراگیر هر سواله که تعیین کننده است!

univers
06-23-2013, 12:04 AM
شما هم اشتباه یکی از دوستان رو کردید.تو خود صورت سوال به طور مستقیم ذکر شده که خروج از مرکز خیلی کوچیکه.یه سوال مصحح در اینجور سوالات به اون رابطه ای که بین کسینوس e و کسینوس تتا نوشتم و اثبات نکردم گیر می ده؟اگه میشه اثباتشو برام بنویسید. ممنون

univers
06-23-2013, 12:16 AM
سلام یه سوال مکانیک داشتم اگه میشه راهنماییم کنید.
دو جرم M , m داریم که در فاصله ی r از یکدیگر قرار دارند و نیروی گرانش به یکدیگر وارد می کنند.در این منحنی که به دور این دو جرم کشیده شده است تمام نقاط به یک اندازه دارای انرژی پتانسیل هستند.اندازه زاویه ی a را که در شکل مشخص شده است را به دست آورید.
لینک شکل سوال: http://up.avastarco.com/images/lsu2do0csc2z3j6zt2b9.png

arashgmn
06-23-2013, 12:30 AM
شما هم اشتباه یکی از دوستان رو کردید.تو خود صورت سوال به طور مستقیم ذکر شده که خروج از مرکز خیلی کوچیکه.یه سوال مصحح در اینجور سوالات به اون رابطه ای که بین کسینوس e و کسینوس تتا نوشتم و اثبات نکردم گیر می ده؟اگه میشه اثباتشو برام بنویسید. ممنون

ببینید شما تو حلّ سوال اوّل اومدین فرض کردین که خروج از مرکز خیلی کوچیکه و در نتیجه توی انتگرال تقریب زدین. من نمی دونم این کار غلطه یا درست...
این که همون اول کار، جمله های شامل e^2 و بالاتر رو حذف کنیم و یا جمله های شامل e رو تا مرتبه اول بسط بدیم، مممکنه مارو به جواب اشتباه برسونه.مثلا همین سوال مورد بحث رو درنظر بگیرید.

گفته شده خروج از مرکز کوچک است. یعنی e<<1 . این یعنی دقتی که ما داریم تا مرتبه اول e رو می تونه تشخیص بده و توان های 2 و 3 و بالاتر از e غیر قابل تشخیص هستن. پس جواب نهایی ما بی معنی خواهد بود اگر جمله ای از توان های بزرگتر از یکِ e داشته باشه.

خوب چرا حذف کردن توان های بزرگتر از یکِ e قبل از شروع محاسبات و در حین انجام محاسبات غلطه ؟ فرض کنید توی محاسباتتون یه جمله شامل e^2 دارید. ممکنه ادامه محاسبات باعث بشه یه جمله ی 1 تقسیم بر e توی جمله اول ضرب بشه. در این صورت ، جمله شامل e^2 تبدیل میشه به e . که دیگه قابل حذف کردن نیست. حالا اگر اون e^2 در آغاز حذف می شد، نتیجه محاسبات هم به احتمال زیاد تغییر می کرد. البته ممکنه مواردی هم وجود داشته باشه که به صورت شانسی (واقعا شانسی!) جواب ها یکی در بیان.

برای همین توصیه می کنن که که اول محاسبات رو (بدون حذف جملات با مرتبه بالا) تا آخر انجام بدین، بعد ، در جواب آخر جملات دارای توان های بزرگتر از 1 برای e رو حذف کتید...

arashgmn
06-23-2013, 10:40 PM
سلام یه سوال مکانیک داشتم اگه میشه راهنماییم کنید.
دو جرم M , m داریم که در فاصله ی r از یکدیگر قرار دارند و نیروی گرانش به یکدیگر وارد می کنند.در این منحنی که به دور این دو جرم کشیده شده است تمام نقاط به یک اندازه دارای انرژی پتانسیل هستند.اندازه زاویه ی a را که در شکل مشخص شده است را به دست آورید.
لینک شکل سوال: http://up.avastarco.com/images/lsu2do0csc2z3j6zt2b9.png
سوال قشنگیه . سعی کنید حتما حلش بکنید. جواب جالبی هم داره...
راه اول :پتانسیل یه جسم که روی خطوط متقاطع و بسیار نزدیک به نقطه تقاطع قرار داره رو بنویسید ... .
راه دوم: سعی کنید معادله خم رو در نزدیکی نقطه تقاطع پیدا کنید و بعد با مشتق گیری، شیب نمودار رو حساب کنید.
http://up.avastarco.com/images/lsu2do0csc2z3j6zt2b9.png

arashgmn
06-24-2013, 12:49 AM
یه سوال دیگه که همراه با سوال پست قبلی می تونید روش فکر کنید.

برای یک بیضی، شعاع استوایی و شعاع قطبی رو به شکل زیر تعریف می کنیم.

http://up.avastarco.com/images/027gs3diwh7zjmy7vvyb.png (http://up.avastarco.com/)

ثابت کنید که :
http://up.avastarco.com/images/1i8f5gpzpe5l366v5gbg.png (http://up.avastarco.com/)

univers
06-24-2013, 01:22 AM
سلام اگه میشه این سوال جدیدی که گذاستین رو بهتر توضیح بدین چون من این شعاع استوایی و قطبی رو خوب نفهمیدم چی هستند؟

univers
06-24-2013, 01:29 AM
با راه اول رفتم و به نتیجه ی خوبی رسیدم .ولی راه دوم رو نفهمیدم اگه میشه دقیق تر توضیح بدید چی کار کنم. ممنون

m.Sadat
06-24-2013, 08:09 PM
برای حل سوال آقا آرش با استفاده از معادله بیضی و جایگذاری sin و cos به جای y/b و x/a داریم :

http://up.avastarco.com/images/lgserho6ax8stw1hj4c.gif

و
http://up.avastarco.com/images/6i5z3tum9cgvj8rpx75.gif

در نتیجه برای مقدار متوسط توابع فوق داریم:

http://up.avastarco.com/images/u8el3996f9nsptyusqz.gif

و

http://up.avastarco.com/images/mdgcxn149ley9oqmewx4.gif

که در آن از رابطه مقدار متوسط برای تابع های پیوسته استفاده شده است
در نتیجه رابطه اصلی به صورت زیر است :

http://up.avastarco.com/images/lmkyfx69aud1t1zuq90.gif

اگر راه حل اشکالی داشت لطفا اطلاع بدید
با تشکر از آقا آرش برای مطرح کردن این سوال

arashgmn
06-24-2013, 09:26 PM
سلام اگه میشه این سوال جدیدی که گذاستین رو بهتر توضیح بدین چون من این شعاع استوایی و قطبی رو خوب نفهمیدم چی هستند؟

شعاع استوایی، یعنی شعاع دایره ای که از قطع کردن یک صفحه موازی صفحه ی استوا و بیضی گون به دست میاد. (برای همین در قطب، شعاع استوایی صفره)

شعاع قطبی یعنی ، شعاع دایره ای که از قطع کردن یک صفحه عمود بر صفحه استوا و بیضی گون به دست میاد. ( به همین ترتیب شعاع قطبی در استوا صفره)


با راه اول رفتم و به نتیجه ی خوبی رسیدم .ولی راه دوم رو نفهمیدم اگه میشه دقیق تر توضیح بدید چی کار کنم. ممنون

یه دستگاه مختصات بذارید روی نقطه تقاطع خم با خودش. به طوری که محور x از مرکز دو تا جرم بگذره. نمودار این خم، به وضوح نمودار یک تابع نیست . اما اگر فقط قسمت موجود در ربع اول رو درنظر بگیریم، یه تابع میشه. حالا سعی کنید که معادله ی این تابع رو در نزدیکی مبدا پیدا کنید...

arashgmn
06-26-2013, 11:11 AM
با دو تا سوال جدید و ساده اومدم :

1- در نیمه شعبانی که گذشت، ماه در حضیض مداری خودش قرار داشت و چون کامل بود، بزرگترین قطر زاویه ای ممکن را داشت. به این ماه اصطلاحا "ابرماه" گفته می شود. محاسبه کنید که در چه تاریخی ابرماه بعدی رویت خواهد شد. (با دقت 0.01 روز)

2- فردی هر روز برای رفتن به سمت کار، مسیری مستقیم به صورت پیاده طی می کند و در برگشت نیز همان مسیر را در جهت عکس می پیماید. این قرد متوجه می شود که در اکثر روز ها در این مسیر ، وقتی در حال پیاده روی است، سایه اش دقیقا مقابلش قرار دارد.برای زمانی که میل خورشید + است، مدت زمانی را که این فرد سر کار بوده است(!) را محاسبه کنید. مدار زمین را دایروی فرض کنید. پارامترهایی که لازم دارید را فرض کنید و جوابتان را برای حالات خاص ساده کتید.

univers
06-26-2013, 12:36 PM
اگه میشه جواب سوالات قبلی رو هم بگذارید. ممنون

univers
06-26-2013, 07:26 PM
فک کنم جواب بشه روز 22 تیر ماه سال 1393

arashgmn
06-27-2013, 05:19 PM
برای حل سوال آقا آرش با استفاده از معادله بیضی و جایگذاری sin و cos به جای y/b و x/a داریم :

http://up.avastarco.com/images/lgserho6ax8stw1hj4c.gif

و
http://up.avastarco.com/images/6i5z3tum9cgvj8rpx75.gif

در نتیجه برای مقدار متوسط توابع فوق داریم:

http://up.avastarco.com/images/u8el3996f9nsptyusqz.gif

و

http://up.avastarco.com/images/mdgcxn149ley9oqmewx4.gif

که در آن از رابطه مقدار متوسط برای تابع های پیوسته استفاده شده است
در نتیجه رابطه اصلی به صورت زیر است :

http://up.avastarco.com/images/lmkyfx69aud1t1zuq90.gif

اگر راه حل اشکالی داشت لطفا اطلاع بدید
با تشکر از آقا آرش برای مطرح کردن این سوال
دقت کنید که ( میانگین Re )به توان 2 و میانگین (Re به توان 2) با هم متفاوت هستن. توی حکم هم، میانگین (Re به توان 2) اومده. یعنی با این نشانه گذاری، شما باید از مجذور کسینوس میانگین بگیرید. نه این که میانگین کسینوس رو به توان 2 برسونید. (احتمالا هم همین کار رو کردید.چون در غیر این صورت جواب ها یکی نمی شدن!)

اگه میشه جواب سوالات قبلی رو هم بگذارید. ممنون
شما گفته بودید که به جاهای خوبی رسیدید. خوب جوابتون رو بذارید. منم می گم که درسته یانه . بقیه هم فکر کنن رو سوال . ;)

univers
06-28-2013, 02:45 AM
من یه اشتباه بدی کردم یه حلت خاص رو درنظر گرفتم و فرض کردم m=M برای همین اگه میشه شما جوابو بگید چون اگه یه سوال دیگه به سرعت یکی جواب می داد ولی در مورد این سوال اینطوری نیست.

arashgmn
06-28-2013, 12:47 PM
من یه اشتباه بدی کردم یه حلت خاص رو درنظر گرفتم و فرض کردم m=M برای همین اگه میشه شما جوابو بگید چون اگه یه سوال دیگه به سرعت یکی جواب می داد ولی در مورد این سوال اینطوری نیست.

نکته جالب این سوال اینه که این زاویه مستقل از جرم هاست. چه برابر باشن و چه نباشن. (البته تا مرتبه دوم !). من تانژانت نصف اون زاویه رو برابر با رادیکال 2 محاسبه کردم. بنابراین ، زاویه ی آلفا 109 درحه و 28 دقیقه به دست میاد.

-------------------------

نکته انحرافی : مسئله ی معروفی وجود داره که میگه زاویه بین پیوند های مولکول متان رو محاسبه کنید. در واقع اثباتیه برا همون 109.5 درجه. زاویه ی آلفا هم در سوال بالا، همون 109.5 درجه شد! البته واضحه که این دوتا مسئله هیچ ربطی به هم ندارن!!! :دی

m.Sadat
06-28-2013, 07:12 PM
دقت کنید که ( میانگین Re )به توان 2 و میانگین (Re به توان 2) با هم متفاوت هستن. توی حکم هم، میانگین (Re به توان 2) اومده. یعنی با این نشانه گذاری، شما باید از مجذور کسینوس میانگین بگیرید. نه این که میانگین کسینوس رو به توان 2 برسونید. (احتمالا هم همین کار رو کردید.چون در غیر این صورت جواب ها یکی نمی شدن!)
آره اونجا اشتباه تایپی شده که باید توی مخرج a توان 2 داشته باشه
ممنون :thumbsup:

univers
06-28-2013, 07:28 PM
من از یکی ازمعلم هام پرسیدم گفت جواب باید حدود 53 درجه بشه و راه حل ایشونم فک کنم درست بوده.اگه میشه شما هم راهتون رو بزارید تا ببینیم مشکل کار کجاست.من احتمال می دم شما به کسینوس آلفا رسیدید که برابر 1/3 - شده.

arashgmn
06-29-2013, 11:35 AM
من از یکی ازمعلم هام پرسیدم گفت جواب باید حدود 53 درجه بشه و راه حل ایشونم فک کنم درست بوده.اگه میشه شما هم راهتون رو بزارید تا ببینیم مشکل کار کجاست.من احتمال می دم شما به کسینوس آلفا رسیدید که برابر 1/3 - شده.
من این بار از اون یکی راه رفتم، باز هم آلفا رو 109.5 به دست آوردم. این بار کسینوس نصف آلفا برابر با 1 تقسیم بر رادیکال 3 شد. دفعه پیش هم تانژانتش شده بود رادیکال 2. شاید اون 53 ، 54 و خورده ای بوده... .:دی

univers
06-29-2013, 07:39 PM
ببخشید من یادم رفت که بگم نصف زاویه رو 54 و خورده ای به دست آوردم و بعد که ضربدر 2 کردم برابر 109.47 شد.فقط نفهمیدم چرا اگر دو جسم جرم یکسان داشته باشند یا نه فرقی نداره؟

univers
07-01-2013, 11:14 AM
من با استفاده از کدوم معادلات می تونم فرمول M/M sun)^3.5 =L/L sun) را اثبات کنم.لطفا راهنماییم کنید.

m.Sadat
07-02-2013, 02:35 PM
من با استفاده از کدوم معادلات می تونم فرمول M/M sun)^3.5 =L/L sun) را اثبات کنم.لطفا راهنماییم کنید.
اثباتش توی جلد 3 کتاب اریکا-بوم (اگه نبود جلد 1 ) هست میتونید نگاه کنید

arashgmn
07-02-2013, 03:31 PM
ببخشید من یادم رفت که بگم نصف زاویه رو 54 و خورده ای به دست آوردم و بعد که ضربدر 2 کردم برابر 109.47 شد.فقط نفهمیدم چرا اگر دو جسم جرم یکسان داشته باشند یا نه فرقی نداره؟
توی فایل پیوست این پست (http://forum.avastarco.com/forum/showthread.php?97-%D8%B1%D9%81%D8%B9-%D8%A7%D8%B4%D9%83%D8%A7%D9%84-%D8%A7%D9%84%D9%85%D9%BE%DB%8C%D8%A7%D8%AF-%D9%86%D8%AC%D9%88%D9%85&p=63185&viewfull=1#post63185) ، تو معادله ی نهایی ، دو تا جمله هست که m و M توشون ظاهر شده. جمله های اول ، از جنس شتاب هستن. و چون روی اون نقطه تقاطع، نیرو ها برابره ، بنابراین ، صفر میشن. بنابراین جمله ی دومم کلا باید صفر باشه که چون G,mو d ها عمگی مثبتن، بنابراین ضریب باید صفر بشه. پس جول کلا مستقل از جرم به دست میاد.

البته دقت کنید که اینم تا مرتبه دوم هست. اگه تا مرتبه ی سوم بر حسب l بنویسیم ، m و M حتما ظاهر میشن توی جواب آخر کسینوس ...

univers
07-04-2013, 07:31 PM
سلام دو سوال فیزیک داشتم :سوال 36و35 کلپنر فصل 2 رو مشکل دارم.اگه میشه برای حلشون راهنماییم کنید.

arashgmn
07-06-2013, 03:42 PM
سلام دو سوال فیزیک داشتم :سوال 36و35 کلپنر فصل 2 رو مشکل دارم.اگه میشه برای حلشون راهنماییم کنید.
اول : سلام

دوم : توی سوال 35 ، باید شتاب مرکز گرا رو با نیروی عمود بر سطح برابر بذارید. (ممکنه که از روی شکل موقعیت حلقه رو درست متوجه نشده باشید . یه حلقه داریم به شعاع داخلی l که روی یک میز قرار گرفته. یعنی اون جسم ، موازی زمین حرکت می کنه...) . بعد با ساده کردن به معادله دیفرانسیلی می رسید که مرتبه اوله و غیر خطی. اما ساده حل میشه.

سوم: توی سوال 36، قرار می دیم F=Ma (البته با علامت منفی! به علت کاهنده بودن نیرو). بعد ساده می کنیم و به معادله دیفرانسیل میرسیم. اگر نمی تونید مستقیما، جواب رو به دست بیارید، سعی کنید که جواب معادله دیفرانسیل رو حدس بزنید. شاید این نکته به درد بخوره : مشتق تابع نمایی، ضریبی از خود تابع است...

mjaasgari
07-06-2013, 06:15 PM
با سلام
١-در حركت گلوله تفنگ اگر اصتكاك هوا وجود داشته باشد آيا مركز جرم سيستم گلوله تفنگ ثابت است؟
٢-اثبات رابطه زير چيست؟
Αχ΅+βχ=0
Χ=αsinώτ-bcosώτ
;ω= radical(β/m)
اون همون اومگا خودمونه اون علامت بالاشو نتونستم پاك كنم جمله اول هم كه آ ايكس دبل دات!!!!
٣-نظرتون راجع به كتاب اصول و عمل چيه كي بايد شروعش كنم ؛نجوم ديناميكي چطور و آيا كتاب ديگري با آن ها هم پو شاني دارد؟؟؟

arashgmn
07-06-2013, 09:41 PM
با سلام
١-در حركت گلوله تفنگ اگر اصتكاك هوا وجود داشته باشد آيا مركز جرم سيستم گلوله تفنگ ثابت است؟
٢-اثبات رابطه زير چيست؟
Αχ΅+βχ=0
Χ=αsinώτ-bcosώτ
;ω= radical(β/m)
اون همون اومگا خودمونه اون علامت بالاشو نتونستم پاك كنم جمله اول هم كه آ ايكس دبل دات!!!!
٣-نظرتون راجع به كتاب اصول و عمل چيه كي بايد شروعش كنم ؛نجوم ديناميكي چطور و آيا كتاب ديگري با آن ها هم پو شاني دارد؟؟؟

از من هم سلام :)

1- مرکز جرم سامانه ی گلوله - تفنگ ، به علت جلو رفتن گلوله تفنگ ، جابه جا میشه. اصطکاک هوا، صرفا سرعت جلو رفتن مرکز جرم رو کندتر می کنه تا وقتی که با ثابت شدن گلوله ، مرکز جرم هم ثابت بشه.

2- فک کنم منظور این بوده :

http://up.avastarco.com/images/p4sz8xnogo7i85suisgh.png (http://up.avastarco.com/)

اگر X با عبارت زیر برابر باشه، با دوبار مشتق گیری بر حسب زمان داریم :

http://up.avastarco.com/images/f7kquzo6dhvw8dc789h.png (http://up.avastarco.com/)

پس به یک شرط، X می تونه جواب معادله دیفرانسیل یاشه. و اون اینه که بتا برابر با اومگا به توان 2 باشه. (یعنی با نماد گذاری شما، اومگا باید برابر با رادیکال بتا تقسیم بر A بشه)

3- برای نجوم کروی ، میشه اصول و عمل یا اسمارت رو از سال اول یا دوم خوند (البته به شرطی که با توابع مثلثاتی به طور کامل آشنا باشین) .
نجوم دینامیکی، کتاب حجیم و قدیمی ایه. برای خوندن نجوم عمومی ، می تونید به نجوم به زبان ساده مراجعه کنید.
کلا کتاب هایی مثل نجوم دینامیکی یا نجوم به زبان ساده، کتاب های عمومی هستن. یعنی سعی شده که توش همه چی به طور کلی و بدون روابط توضیح داده بشه. برای همین میشه گفت که این کتابا با کل نجوم هم پوشانی دارن(!) اما مطلب تخصصی به آدم یاد نمی دن...

univers
07-06-2013, 09:48 PM
آقای گل محمدی جواب اون دو تا سوال که قبلا گذاشتید رو نمی دید؟؟؟؟؟؟ ببینیم جوابمون درسته یا نه؟؟؟

mjaasgari
07-06-2013, 10:23 PM
از من هم سلام :)

1- مرکز جرم سامانه ی گلوله - تفنگ ، به علت جلو رفتن گلوله تفنگ ، جابه جا میشه. اصطکاک هوا، صرفا سرعت جلو رفتن مرکز جرم رو کندتر می کنه تا وقتی که با ثابت شدن گلوله ، مرکز جرم هم ثابت بشه.

2- فک کنم منظور این بوده :

http://up.avastarco.com/images/p4sz8xnogo7i85suisgh.png (http://up.avastarco.com/)

اگر X با عبارت زیر برابر باشه، با دوبار مشتق گیری بر حسب زمان داریم :

http://up.avastarco.com/images/f7kquzo6dhvw8dc789h.png (http://up.avastarco.com/)

پس به یک شرط، X می تونه جواب معادله دیفرانسیل یاشه. و اون اینه که بتا برابر با اومگا به توان 2 باشه. (یعنی با نماد گذاری شما، اومگا باید برابر با رادیکال بتا تقسیم بر A بشه)

3- برای نجوم کروی ، میشه اصول و عمل یا اسمارت رو از سال اول یا دوم خوند (البته به شرطی که با توابع مثلثاتی به طور کامل آشنا باشین) .
نجوم دینامیکی، کتاب حجیم و قدیمی ایه. برای خوندن نجوم عمومی ، می تونید به نجوم به زبان ساده مراجعه کنید.
کلا کتاب هایی مثل نجوم دینامیکی یا نجوم به زبان ساده، کتاب های عمومی هستن. یعنی سعی شده که توش همه چی به طور کلی و بدون روابط توضیح داده بشه. برای همین میشه گفت که این کتابا با کل نجوم هم پوشانی دارن(!) اما مطلب تخصصی به آدم یاد نمی دن...
آقا آرش خان ممنون فقط اينكه من الان دوم ام دارم ميرم سوم (ايشاالله)خوب پس به نظر شما اسمارت رو بخونم الان يا اصول و عمل برا كروي بعد هم يه سوال فني ديگه !

كتاب هايي مثل modern astrophysicsرو كه زبان انگليسي اند چطور بخونم كه هم بهتر بفهمم هم سريع تر باشه (ميخوام از صفر شروع كنم) خودتون چيكار ميكرديد؟(جسارتا)
راستي كتاب نجوم و اختر فيزيك مقدماتي نوشته زيليك و اسميت چطوره؟؟؟؟
ممنون

راستي ببخشيد من منظورم تو قسمت اول سوال اين بود كه تفنگ روي ميز هست بدون اصتكاك و قابليت حركت هم داره يعني هم گلوله حركت ميكنه و به واسطه قانون سوم نيوتن توپ هم به عقب حركت ميكنه حالا ديگه مركز جرم ثابت است درسته؟؟؟؟؟؟؟؟؟؟؟؟
حالا اگر اصتكاك هوا دخيل باشه چي ؟
آيا و جود فقط اصتكاك هوا با وحود فقط اصتكاك ميز(به يك اندازه)هم ارز است؟؟؟؟؟؟؟؟
ممنون

arashgmn
07-07-2013, 12:37 PM
آقای گل محمدی جواب اون دو تا سوال که قبلا گذاشتید رو نمی دید؟؟؟؟؟؟ ببینیم جوابمون درسته یا نه؟؟؟
اول جواب سوال 2 که راحت تره . سوال این بود :

2- فردی هر روز برای رفتن به سمت کار، مسیری مستقیم به صورت پیاده طی می کند و در برگشت نیز همان مسیر را در جهت عکس می پیماید. این قرد متوجه می شود که در اکثر روز ها در این مسیر ، وقتی در حال پیاده روی است، سایه اش دقیقا مقابلش قرار دارد.برای زمانی که میل خورشید + است، مدت زمانی را که این فرد سر کار بوده است(!) را محاسبه کنید. مدار زمین را دایروی فرض کنید. پارامترهایی که لازم دارید را فرض کنید و جوابتان را برای حالات خاص ساده کتید.
نباید سخت فکر کنید. فرض کنید که مسیری که این فرد هر روز طی می کند ، در راستای سمت A باشد. بنابراین وقتی فرد در این راستا حرکت می کند ، خورشید در سمت های A و A+180 قرار گرفته است. هدف پیدا کردن مدت زمانی است که خورشید از سمت A به سمت A+180 میرود. زیرا فرد در این زمان سر کار بوده است. به شکل زیر توجه کنید :
http://up.avastarco.com/images/x952azkxess49bc9yib5.png (http://up.avastarco.com/)
یا نوشتن یک چهار جزئی، در مثلث PZSun داریم :

cos (H) sin (fi) = cos (fi) tan (delta) - sin H cot A

که دلتا هم از رابطه ی از رابطه ی زیر به دست میاد (لاندا طول سماوی خورشید است) :

sin (delta) = sin(landa) sin 23.5

بنابراین می توان H رو در حالت کلی با حل یک معادله مثلثاتی نوع اول و یا تبدیل به یک معادله درجه 2، بر حسب عرض جغرافیایی ، روز سال و سمت A پیدا کرد.

چون این فرد درحقیقت خود من بودم ، و من در مسیر اتوبان شرقی- غربی صدر پیاده روی می کنم ، بنابراین A=90 و fi = 35.5. پس به عنوان مثال برای روز 2 تیر (در این روز واقعا رفتم دانشگاه! ) داریم :

دلتا = 23.47 پس : H =71.96 درجه که هم ارزه با 4 ساعت و 48 دقیقه . چون زمان سر کار بودن ، دو برابر H خواهد شد ، زمان نهایی برابر است با 9 ساعت و 36 دقیقه.

arashgmn
07-07-2013, 12:53 PM
اما سوال اول :

1- در نیمه شعبانی که گذشت، ماه در حضیض مداری خودش قرار داشت و چون کامل بود، بزرگترین قطر زاویه ای ممکن را داشت. به این ماه اصطلاحا "ابرماه" گفته می شود. محاسبه کنید که در چه تاریخی ابرماه بعدی رویت خواهد شد. (با دقت 0.01 روز)

این سوال رو میشه با 4 نوع دیدگاه حل کرد. دیدگاه اول ساده اس اما تقریبا نادرست. دیدگاه دوم و سوم نیمه درست اما سخت هستن. دیدگاه چهارم کاملا درسته و به اندازه ی دیدگاه سوم و دوم سخته. اما قدم به قدم پیش رفتن بر اساس این دیدگاه ها می تونه آموزنده باشه. پس همه ی اون ها رو می نویسم.

دیدگاه اول :
دوره تناوب هلالی متوسط ماه (29.53 روز) و دوره تناوب متوسط بی هنجار (مدت زمان دو گذر متوالی ماه از حضیض)(27.55 روز) رو در نظر می گیریم. به شکل زیر توجه کنید.

http://up.avastarco.com/images/4m7ujj9wvbnsklrjb.png (http://up.avastarco.com/)
در حالت 1 ، ابرماه رخ داده. بعد از گذشت یک ماه بی هنجار ،ماه دوباره بر روی حضیض قرار خواهد گرفت. اما این بار خورشید و ماه و زمین در یک راستا قرار ندارند. خورشید در مدت 27.55 روز ، به میزان زیر جابجا شده است :

(360/29.53) * 27.55 = 335.862

بنابراین خورشید در هر دوره تناوب بی هنجار ِ ماه به اندازه ی 24.138 درجه عقب می افتد. حالا باید ببینیم که این میزان عقب افتادگی ، بعد از چند بار رسیدن ماه به حضیض ، تبدیل به یک عدد صحیح می شود. اگر N تعداد دورهای ماه، و n یک عدد صحیح باشد، باید داشته باشیم :


http://up.avastarco.com/images/0mv1e63wcpujbj0bpnth.png (http://up.avastarco.com/)
پس اولین عددی که n را صحیح می کند ، N=2953 است. یعنی بعد از 2953 ماه بی هنجار ، خورشید و زمین و ماه دوباره ذقیقا در یک راستا قرار می گیرند. که برابر است با 81355.15 روز . با تبدیل آن به سال داریم :222.738 سال . ابر ماه فعلی در تاریخ 2 تیر رخ داد. پس با احتساب 55 روز کبیسه ای که در این 222 سال وجود دارد ، تاریخ ابرماه بعدی به شکل زیر به دست می آید :
سال :
1392 + 222 =1614
روز
0.738*365.25 = 269.65 با کم کردن روزهای کبیسه داریم 269.65- 55 = 214.65

پس 214.65 روز بعد از 2 تیر سال 1614 ابر ماه رخ می دهد. که معادل است با 4 بهمن ماه 1614.

------------------
پ.ن : توجه کنید که در محاسبات بالا ما هدفمان این بود که ماه، خورشید و زمین ، هر سه دقیقا بر روی یک خط قرار گیرند. در حالی که اگر این سه دارای موقعیتی نزدیک به حالت بالا هم باشند ، عرفا یک ابرماه خواهیم داشت. پس نیازی نیست که هر سه دقیقا بر روی یک خط قرار داشته باشند. به همین دلیل رخداد بالا به میزانی که در بالا به دست آمد ، نادر نیست.

اما این که انحراف از خط این سه چگونه باشد، دیدگاه های دوم تا چهارم را به وجود می آورد... ادامه دارد ...

arashgmn
07-07-2013, 01:15 PM
آقا آرش خان ممنون فقط اينكه من الان دوم ام دارم ميرم سوم (ايشاالله)خوب پس به نظر شما اسمارت رو بخونم الان يا اصول و عمل برا كروي بعد هم يه سوال فني ديگه !

كتاب هايي مثل modern astrophysicsرو كه زبان انگليسي اند چطور بخونم كه هم بهتر بفهمم هم سريع تر باشه (ميخوام از صفر شروع كنم) خودتون چيكار ميكرديد؟(جسارتا)
راستي كتاب نجوم و اختر فيزيك مقدماتي نوشته زيليك و اسميت چطوره؟؟؟؟
ممنون

راستي ببخشيد من منظورم تو قسمت اول سوال اين بود كه تفنگ روي ميز هست بدون اصتكاك و قابليت حركت هم داره يعني هم گلوله حركت ميكنه و به واسطه قانون سوم نيوتن توپ هم به عقب حركت ميكنه حالا ديگه مركز جرم ثابت است درسته؟؟؟؟؟؟؟؟؟؟؟؟
حالا اگر اصتكاك هوا دخيل باشه چي ؟
آيا و جود فقط اصتكاك هوا با وحود فقط اصتكاك ميز(به يك اندازه)هم ارز است؟؟؟؟؟؟؟؟
ممنون
اسمارت رو شروع کنید بهتره. برای کتاب های انگلیسی هم می تونید یک نرم افزار دیکشنری (مثل Babylon) رو دانلود کنید تا سرعت خوندنتون بالا بره. کتاب زیلیک کلا ترجمه ی خوبی نداره . اگر من بودم به جای زیلیک ، فاندامنتل می خوندم.

توی سوال تفنگ ، فرض کنید اصطکاک هوا وجود نداره. چون در راستای افقی هیچ نیرویی به سامانه وارد نشده ، سرعت مرکز جرم ، بی تغییر باقی می مونه. یعنی اگر V1 و V2 بردار های سرعت گلوله و تفنگ نسبت به یک ناظر لخت باشن (برای راحتی ناظر رو ساکن فرض می کنیم.) ، در این صورت(m1V1 + m2V2) تقسیم بر (m1 +m2) برابره با سرعت اولیه مرکز جرم. اگر تفنگ قبل از شلیک ساکن بوده باشه ، سرعت مرکز جرم ، برابر با صفر میشه و اون حاصل جمع هم صفر میشه. حالا اصطکاک هوا رو وارد می کنیم. بلافاصله بعد از شلیک ، هوا هنوز بر سرعت گلوله اثر نکرده. بنابراین سرعت ها ، همون سرعت ها در حالت بدون اصطکاک خواهند بود. اما با گذر زمان، اصطکاک سرعت گلوله (V1) رو کم خواهد کرد. بنابراین جمله ی m1V1 تغییر می کنه و سرعت مرکز جرم هم تغییر خواهد کرد. اگر میز هم اصطکاک داشته باشه ، به دلیلی مشابه عبارت m2V2 عوض خواهد شد. اما این که این دو اثر یکدیگر راخنثی کنند، در حالت کلی صحیح نیست. چون اصطکاک ها به جنس و سطح مقطع گلوله مربوط است و اصطکاک میز به جنس میز و تفنگ و وزن آن.

-------------------
پ.ن1 : اگر خواستید babylon رو دانلود کنید، نسخه ی 7 و 9 رو دانلود کنید. 8 خوب نیست!!!

پ.ن2 : توی بابیلون ، اگر ماوس رو وی یک کلمه بگیرید و دکمه ی وسط ماوس رو فشار بدبد، معنی اونو می نویسه. اما این برای کتاب هایی کاربرد داره که pdf یا Djvu باشن. pdf هایی که اسکن شدن ، روی این نرم افزار جواب نمی دن. برای همین اگر خواستید مدرن رو بخونید ، نسخه ی Djvu ی اونو بخونید. تازه حجم کمتری هم نسبت به pdf ش داره ...

arashgmn
07-08-2013, 04:35 PM
دیدگاه دوم : ماه در یک بازه ی زمانی بدر است!

توجه کنید که در محاسبات بالا ما هدفمان این بود که ماه، خورشید و زمین ، هر سه دقیقا بر روی یک خط قرار گیرند. در حالی که اگر این سه دارای موقعیتی نزدیک به حالت بالا هم باشند ، عرفا یک ابرماه خواهیم داشت. پس نیازی نیست که هر سه دقیقا بر روی یک خط قرار داشته باشند.

در حقیقت باید بدانیم که از نظر عرف ، ماه بدر چیست ؟ به عنوان یک تعریف ساده ، می توان این گونه فرض کرد :


در نصف روز قبل و بعد از لحظه ی روی دادن ماه بدر ، ماه را ماه بدر می پنداریم.

بنابراین خورشید می تواند به میزان زیر از خط زمین-ماه انحراف داشته باشد:

0.5* (360/29.53) = 6.095


http://up.avastarco.com/images/5riubylrqj4nlni85ub.png (http://up.avastarco.com/)

در این صورت باید دید بعد از چند ماه بی هنجار ، خورشید به میزان بتا درجه چرخیده است :


http://up.avastarco.com/images/7z4ya5twh4tlvp7d40m1.png (http://up.avastarco.com/)

با محاسبه ی عددی عبارات بالا به ازای مقادیر مختلف n ، برای N=15 شرط بالا برآورده می شود. یعنی

15 * 24.138 = (1*360) + 2.07 .

چون 2.07 درجه کمتر از 6.095 است ، بنابراین نی توانیم طبق تعریف عرفی مان ، به آن ماه ، ماه بدر بگوییم. و از آنجا که آن ماه در حضیض است ، پس ابر ماه است . پس تاریخ این رویداد برابر است با 413.25 روز بعد از 2 تیر 1392. یعنی : 19 مرداد ماه 1393 .

arashgmn
07-13-2013, 10:58 PM
اول : از همه عذر خواهی می کنم بابت این که به مدتی گرفتار بودم و نرسیدم جواب رو کامل کنم. دوم : اگر جایی رو نفهمیدید بپرسید :)

[B]دیدگاه سوم : ماه در یک بازه ی زمانی در حضیض است.

از آن جایی که حضیض ماه در یک لحظه رخ می دهد، به نظر می رسد که بهتر است یک نوع حضیض عرفی نیز برای ماه تعریف کنیم. به طوری که اگر ماه در آن بازه قرار گیرد ، عملا فاصله ی ماه با زمین برابر با فاصله ی حضیض ماه است :


http://up.avastarco.com/images/z2d32ogerbiplw1izctv.png (http://up.avastarco.com/)

می توانیم مانند دیدگاه دوم ، یک تعریف برای حضیض (به صورت عرف) را مبنا قرار دهیم و زاویه ی انحراف ماه با خط زمین خورشید را به دست آوریم. اما از آنجا که در دیدگاه چهارم ، هندسه، یک تعریف ارجح به ما تحمیل می کند ، دیدگاه چهارم (دقیق ترین و بهترین راه) را برای حل انتخاب می کنیم.

arashgmn
07-13-2013, 10:59 PM
دیدگاه چهارم:

بهتر آنست که خیلی کلی به مسئله نگاه کنیم. در نیمه ی شعبان سال 1392 خورشیدی ، (فرض می کنیم) ماه و زمین و خورشید دقیقا بر روی راستای اوجی حضیضی ماه قرار داشته اند. می خواهیم بدانیم بعد از چه مدت این موقعیت دوباره (به طور تقریبی) تکرار می شود. به شکل زیر توجه کنید :


http://up.avastarco.com/images/qkjovy76n2vlwt1v9qt.png (http://up.avastarco.com/)

فرض کنید سرعت زاویه ای ماه Wm و سرعت زاویه ای خورشید Ws باشد . ( Wm متناظر با گشتن ما به دور زمین در 27.55 روز و Ws متناظر با دوره تناوب هلالی ماه در 29.53 روز است.) هم چنین فرض کنید خورشید مجاز باشد به میزان آلفا از راستای اوجی حضیضی انحراف داشته باشد اما با این حال ماه بدر بماند (دیدگاه 2 ؛ که آلفا حدود 6 درجه به دست آمد). و نیز فرض کنید ماه می تواند به میزان بتا از راستای اوجی حضیضی انحراف داشته باشد اما با این حال در از نظر عرف هم چنان در حضیض باشد(دیدگاه 3 ).

با نگاه به شکل متوجه میشویم که برای این که ماه بدر بماند ، بتا الزاما باید از آلفا کوچکتر باشد. اما توجه کنید که در دیدگاه دوم، برای داشتن ماه بدر ، خورشید می توانست به میزان 6 درجه از خط زمین ماه انحراف داشته باشد. بنابراین با توجه به این معیار ، به نظر می رسد برای ترکیب بدر عرفی و حضیض عرفی ، زوایای زیر مناسب اند :


http://up.avastarco.com/images/7e73ruucn7xedar50vu1.png (http://up.avastarco.com/)

علت این انتخاب در شکل زیر مشخص است :


http://up.avastarco.com/images/khgbdiphmdxommgut4jk.png (http://up.avastarco.com/)

ماه می تواند در محدوده ی E و F باشد تا حضیض داشته باشیم. هم چنین خورشید باید در محدوده ی A و B باشد تا بدر داشته باشیم. بنابراین حداکثر اختلاف زاویه ای بین راستای ماه و خورشید از 6.095 درجه تجاوز نخواهد کرد.

اگر t زمان باشد که از نیمه ی شعبان اندازه گرفته می شود ، باید داشته باشیم :

http://up.avastarco.com/images/n8d8mmfb8rvmk7mffi5.png (http://up.avastarco.com/)

که p و q به ترتیب تعداد دور های خورشید و ماه هستند. با حذف t خواهیم داشت :

http://up.avastarco.com/images/r12jevbx29h8cc3lyj4u.png (http://up.avastarco.com/)

از آنجا که p و q الزاما اعدادی صحیح هستند ، و از رابطه ی زیر به دست می آیند ،

http://up.avastarco.com/images/mzgyvtfy9fywbll7f6s2.png (http://up.avastarco.com/)

با توجه به برابر بودن آلفا و بتا و با ساده کردن خواهیم داشت :



http://up.avastarco.com/images/1rg39iehu71yyqtd49pl.png (http://up.avastarco.com/)
یا :

http://up.avastarco.com/images/j8ha1vt3f6q7vl1k27p.png (http://up.avastarco.com/)

که T ها دوره تناوب ها هستند. باید دید چه t ای در معادله ی بالا صدق می کند. می توان با رسم نمودار نشان داد که در t های مانند زیر همگی به حدود بالا بسیار نزدیک هستند و عملا هریک می توانند یک ابر ماه باشند:

4934.90 - 9456.00 -13977.14

اما اولین وضعیتی که دقیقا در حدود + و - ی بالا صدق می کند ، در t = 14390.85 رخ می دهد.


http://up.avastarco.com/images/hvgc9gwoknzmldparye5.png (http://up.avastarco.com/)
رنگ آبی و سبز + و - ی 6 و خورده ای در 10 به توان -4 هستند و رنگ قرمز همون تابع پله ایه...

یعنی ابرماه بعدی (با توجه به تعریف ماه از ماه بدر و حضیض ) ، (با احتساب روز های کبیسه) در تاریخ 16 آبان ماه سال 1431 رخ خواهد داد.[/B]

mjaasgari
07-16-2013, 10:04 AM
با سلام مجدد چند تا سوال برام ايجاد شده تو خوندن كتاب اسمارت:
١-در تعريف مثلث كروي بحاي اينكه از دواير عظيمه براي اتصال نفاط به يك ديگر استفاده كنيم از دواري استفاده منيم كه مختصات مركز آن ها[٠،٠،z]باشد.حال آيا ميتوان روابط مثلثات كروي را براي اين شرايط بازنويسي كرد؟(تعميم دواير از دو نقطه به مختصات [x,y,z] و[x',y',z']عبور كند؟!)
٢-آيا در نيمكره ي جنوبي زمين سنت ستاره ها او جنوب افق تا پاي ستاره محاسبه ميشود ؟چرا؟ودر كدام جهت(شرق به غرب يا غرب به شرق)؟
خيلي ممنون ببخشيد.

veno0s
07-16-2013, 12:34 PM
سلام دوستان من هم یه سوال داشتم ازکتاب نجوم دینامیکی:

اگردراستوازندگی می کردیدهرگاه درتمام طول شب چشم به آسمان می دوختید چنددرصدآسمان رامی توانستیدببینید؟


یه سوال دیگه:

آیامی توانیدمحاسباتی انجام دهیدکه به ماکمک کندتاپیش بینی کنیم که ماه هرشب باتغییرمکان تقریبی 13درجه به شرق نسبت به ستارگان ظاهرمی شود؟

m.Sadat
07-16-2013, 06:37 PM
سلام دوستان من هم یه سوال داشتم ازکتاب نجوم دینامیکی:

اگردراستوازندگی می کردیدهرگاه درتمام طول شب چشم به آسمان می دوختید چنددرصدآسمان رامی توانستیدببینید؟


یه سوال دیگه:

آیامی توانیدمحاسباتی انجام دهیدکه به ماکمک کندتاپیش بینی کنیم که ماه هرشب باتغییرمکان تقریبی 13درجه به شرق نسبت به ستارگان ظاهرمی شود؟

میدانیم که دوره تناوب ماه برابر 27.5 روز است یعنی در هر 27.5 روز 360 در جه را طی میکنه حال با یک تناسب بدست میاریم که میشه 13 درجه توی هر روز
چون که توی استوا ستاره دور قطبی نداریم در طول شب کل نیمکره شمالی را میتونیم مشاهده کنیم که میشه 50 درصد اسمون

m.Sadat
07-16-2013, 06:38 PM
سلام دوستان من هم یه سوال داشتم ازکتاب نجوم دینامیکی:

اگردراستوازندگی می کردیدهرگاه درتمام طول شب چشم به آسمان می دوختید چنددرصدآسمان رامی توانستیدببینید؟


یه سوال دیگه:

آیامی توانیدمحاسباتی انجام دهیدکه به ماکمک کندتاپیش بینی کنیم که ماه هرشب باتغییرمکان تقریبی 13درجه به شرق نسبت به ستارگان ظاهرمی شود؟

میدانیم که دوره تناوب ماه برابر 27.5 روز است یعنی در هر 27.5 روز 360 در جه را طی میکنه حال با یک تناسب بدست میاریم که میشه 13 درجه توی هر روز
چون که توی استوا ستاره دور قطبی نداریم در طول شب کل نیمکره شمالی را میتونیم مشاهده کنیم که میشه 50 درصد اسمون و در طول سال میتونیم کل آسمون رو ببینیم

soroosh
07-16-2013, 08:49 PM
یه سوال برای یادگیری بیش تر این مطلب
البته اگر نجوم کروی بلدید
حساب کنید در تهران با عرض جغرافیایی 36 درجه در طول سال چند درصد آسمان هیچ وقت دیده نمی شود

veno0s
07-19-2013, 06:16 PM
سلام یه سوال:

نیم قطراطول مدارسیاره ای بافاصله ی اوج خورشیدی 160میلیون کیلومتروفاصله ی حضیض خورشیدی 100میلیون کیلومتررابیابید.

univers
07-19-2013, 09:51 PM
سلام
می دانیم فاصله اوج برابر a+ae که برابر 160 میلیون کیلومتر است و فاصله ی حضیض برابر a-ae که برابر 100 میلیون کیلومتر است.اگر این دو را با هم جمع کنیم حاصل آن برابر است با 2a پس:
160000000+100000000=260000000=2a پس نیم قطر اطول برابر است با 130 میلیون کیلومتر .

mjaasgari
07-20-2013, 09:07 AM
با سلام مجدد چند تا سوال برام ايجاد شده تو خوندن كتاب اسمارت:
١-در تعريف مثلث كروي بحاي اينكه از دواير عظيمه براي اتصال نفاط به يك ديگر استفاده كنيم از دواري استفاده منيم كه مختصات مركز آن ها[٠،٠،z]باشد.حال آيا ميتوان روابط مثلثات كروي را براي اين شرايط بازنويسي كرد؟(تعميم دواير از دو نقطه به مختصات [x,y,z] و[x',y',z']عبور كند؟!)
٢-آيا در نيمكره ي جنوبي زمين سنت ستاره ها او جنوب افق تا پاي ستاره محاسبه ميشود ؟چرا؟ودر كدام جهت(شرق به غرب يا غرب به شرق)؟
خيلي ممنون ببخشيد.
چرا كسي جواب نميده؟؟؟؟

m.Sadat
07-20-2013, 03:55 PM
با سلام مجدد چند تا سوال برام ايجاد شده تو خوندن كتاب اسمارت:
١-در تعريف مثلث كروي بحاي اينكه از دواير عظيمه براي اتصال نفاط به يك ديگر استفاده كنيم از دواري استفاده منيم كه مختصات مركز آن ها[٠،٠،z]باشد.حال آيا ميتوان روابط مثلثات كروي را براي اين شرايط بازنويسي كرد؟(تعميم دواير از دو نقطه به مختصات [x,y,z] و[x',y',z']عبور كند؟!)
٢-آيا در نيمكره ي جنوبي زمين سنت ستاره ها او جنوب افق تا پاي ستاره محاسبه ميشود ؟چرا؟ودر كدام جهت(شرق به غرب يا غرب به شرق)؟
خيلي ممنون ببخشيد.
چرا كسي جواب نميده؟؟؟؟

سوال اولتون رو نفهمیدم
اما در مورد سوال دوم باید بگم که سمت ستاره ها در نیمکره جنوبی عین نیم کره شمالی تعریف میشه با این تفاوت که از قطب جنوب سماوی اندازه گیری میشه

arashgmn
07-20-2013, 05:09 PM
با سلام مجدد چند تا سوال برام ايجاد شده تو خوندن كتاب اسمارت:
١-در تعريف مثلث كروي بحاي اينكه از دواير عظيمه براي اتصال نفاط به يك ديگر استفاده كنيم از دواري استفاده منيم كه مختصات مركز آن ها[٠،٠،z]باشد.حال آيا ميتوان روابط مثلثات كروي را براي اين شرايط بازنويسي كرد؟(تعميم دواير از دو نقطه به مختصات [x,y,z] و[x',y',z']عبور كند؟!)
٢-آيا در نيمكره ي جنوبي زمين سنت ستاره ها او جنوب افق تا پاي ستاره محاسبه ميشود ؟چرا؟ودر كدام جهت(شرق به غرب يا غرب به شرق)؟
خيلي ممنون ببخشيد.
چرا كسي جواب نميده؟؟؟؟
1- این طور که من متوجه شدم ، شما درباره ی این حالت صحبت می کنید :
http://up.avastarco.com/images/rifroat5fpw0wwunvgc.png (http://up.avastarco.com/)
کمان سبز رنگ یک دایره صغیره اس. برای همین نمی تونه با دو ضلع بنفش ، یک مثلث کروی بسازه. به همین علت هم نمیشه روابط مثلثات کروی رو توی این مثلث نوشت. اما اگه به جای کمان سبز، از کمان نارنجی رنگ (که یک دایره عظیمه است) استفاده کنیم ، مشکل حل می شود. البته مشخصه که اگه نقاط واقع بر روی دایره ی آبی رنگ ، خیلی به هم نزدیک باشن ،اندازه ی کمان های سبز و نارنجی ، بسیار نزدیک به هم می شود و می توان به تقریب از طول کمان سبز به جای نارنجی استفاده کرد.

2- من فقط یک نکته بگم که سمت ، خیلی تعریف سفت و سختی نداره. قبلا سمت رو از جنوب می سنجیدن اما حالا از شمال می سنجن. حتی جهت زیاد شدن اون هم خیلی اهمیت نداره. چون به قول ویکی پدیای انگلیسی :


...a bearing might be described as "(from) south, (turn) thirty degrees (toward the) east" (the words in brackets are usually omitted), abbreviated "S30°E"...
میشه برای رفع ابهام سمت اینطوری آدرس داد : " از جنوب 30 درجه به سمت شرق..." .

این تعاریف مختلف برای سمت ، در نیم کره ی جنوبی هم وجود داره. حتی اگر دقت کنید می بینید که نرم افزار هایی مثل استاری نایت یا استلاریوم ، سمت یک ستاره در نیمکره ی جنوبی رو از شمال می سنجند! با این که توی اسمارت گفته شده سمت در نیم کره ی جنوبی از جنوب سنجیده میشه... . مهم اینه که عبارت که به عنوان سمت گزارش میشه ابهامی نداشته باشه. ;)

A.mousavi
07-20-2013, 06:26 PM
در استوا و در یک شب خیلی بیشتر از 50 درصد آسمون قابل دیدنه، فرض کنید الان ساعت 10 شب، تو استوایین. الان بالا سرتون می تونین 50 درصد آسمون رو ببینید، 1 ساعت بعد که میاین بیرون آسمون حدود 14 درجه چرخیده، و شما ستاره ها و صورت فلکی های جدیدتری رو دارین می بینین. دیده نشدن بخشی از آسمان در طول یک شبانه روز در استوا، بر خلاف دیگر نقاط زمین، به خاطر نور خورشیده، چون نزدیک غروب و طلوع خورشید این روشنی آسمونه که نمی زاره ما 100 آسمون رو ببینیم، در حالی که در مناطق غیر استواییف یک سری از ستاره ها به خاطر میلشون کلا بالای افق نمیان.

veno0s
07-20-2013, 06:57 PM
سلام

1.دوره ی گردش سیاره ای فرضی رابیابیدکه درفاصله ی متوسط8auخورشیدرادورمی زند.

m.Sadat
07-21-2013, 10:42 AM
در استوا و در یک شب خیلی بیشتر از 50 درصد آسمون قابل دیدنه، فرض کنید الان ساعت 10 شب، تو استوایین. الان بالا سرتون می تونین 50 درصد آسمون رو ببینید، 1 ساعت بعد که میاین بیرون آسمون حدود 14 درجه چرخیده، و شما ستاره ها و صورت فلکی های جدیدتری رو دارین می بینین. دیده نشدن بخشی از آسمان در طول یک شبانه روز در استوا، بر خلاف دیگر نقاط زمین، به خاطر نور خورشیده، چون نزدیک غروب و طلوع خورشید این روشنی آسمونه که نمی زاره ما 100 آسمون رو ببینیم، در حالی که در مناطق غیر استواییف یک سری از ستاره ها به خاطر میلشون کلا بالای افق نمیان.
آره اون جا باید میگفتم کل آسمون خواستم درستش کنم نشد

garamaleki
07-21-2013, 10:46 AM
این جور سوال هارو چه طور باید حل کرد ؟
دو ستاره بعد و میلشون داده شده ، بعد می گه یه جا هم ارتفاع می شند عرض جغرافیایی رو پیدا کنید

arashgmn
07-21-2013, 05:33 PM
سلام

1.دوره ی گردش سیاره ای فرضی رابیابیدکه درفاصله ی متوسط8auخورشیدرادورمی زند.
می تونید از قانون سوم کپلر استفاده کنید که میگه : برای منظومه ی شمسی (دوره تناوب سیاره برحسب سال) به توان 2 = (فاصله متوسط سیاره از خورشید بر حسب واحد نجومی) به توان 3 !
در نتیجه دوره تناوب این سیاره میشه حدود 22.6 سال.


این جور سوال هارو چه طور باید حل کرد ؟
دو ستاره بعد و میلشون داده شده ، بعد می گه یه جا هم ارتفاع می شند عرض جغرافیایی رو پیدا کنید

می تونید دو ستاره رو روی کره ی آسمان در نظر بگیرید، بعد سرسوی ناظر رو اونقدر روی کره ی آسمان جابجا کین که فاصله ی دوستاره از سرسو یکسان بشه.

garamaleki
07-22-2013, 10:19 AM
می تونید دو ستاره رو روی کره ی آسمان در نظر بگیرید، بعد سرسوی ناظر رو اونقدر روی کره ی آسمان جابجا کین که فاصله ی دوستاره از سرسو یکسان بشه.
خوب من می دونم این کارو باید بکنم ولی نمی تونم معادلشو تشکیل بدم(همیشه یه چیزی کم میاد :دی)

saeed77
07-22-2013, 02:30 PM
خوب من می دونم این کارو باید بکنم ولی نمی تونم معادلشو تشکیل بدم(همیشه یه چیزی کم میاد :دی)
اگر به طور شهودی هم روی کره ی سماوی نگاه کنی میبینی که اگه اون ارتفاعی که 2 ستاره در آن هم ارتفاع میشن رو نداده باشه(تنها داده ی مسئله بعد و میل 2 ستاره باشند) سوال جواب نداره (یا بهتره بگم بی نهایت جواب داره)!
اگه بخوایم با اطلاعات مسئله به ترتیب پیش بریم وقتی 2 ستاره رو روی کره ی سماوی مشخص کنیم و اون 2 تا رو به هم وصل کنیم و بعدش دایره ی عظیمه ی عمود منصف این کمان رو بکشیم، تمام نقاط روی این دایره ی عظیمه نشان دهنده ی ارتفاع های یکسان برای 2 ستاره هستند(فاصه شون از 2 ستاره یکی است)! پس اگه بخوای عرض جغرافیایی یکتا پیدا کنی باید یه داده ی دیگه داشته باشی که 1 نقطه رو روی این دایره ی عظیمه یکتا کنه تا جواب مسئله بدست بیاد!

arashgmn
07-22-2013, 07:37 PM
اتفاقا نکته همینه . اون داده به شما داده شده ! "...1 جا هم ارتفاع میشن ..."

---------------
این پست پر است از ابهام و آرایه های ادبی :دی

saeed77
07-24-2013, 01:55 PM
خب اون 2 ستاره در بی نهایت جا هم ارتفاع میشن و این که بخوایم از این نکته که فقط در 1 جا هم ارتفاع میشن استفاده کنیم اشتباهه چون اولین داده ی سوال بعد و میل 2 ستاره است و در 1 نقطه هم ارتفاع شدن داده ی قبلی رو نقض میکنه!

arashgmn
07-24-2013, 03:17 PM
خب اون 2 ستاره در بی نهایت جا هم ارتفاع میشن و این که بخوایم از این نکته که فقط در 1 جا هم ارتفاع میشن استفاده کنیم اشتباهه چون اولین داده ی سوال بعد و میل 2 ستاره است و در 1 نقطه هم ارتفاع شدن داده ی قبلی رو نقض میکنه!

نقض ؟! نه . هم چین اتفاقی نمی افته...

ببین منظور از یک نقطه ، یک نقطه روی کره ی زمین نیست ها ... منظور اینه : "روی یک شهر خاصی (که قراره عرضش رو پیدا کنید) ، فقط در یک زمان ستاره ها هم ارتفاع میشن..."

saeed77
07-24-2013, 05:25 PM
این که فقط در 1 عرض جغرافیایی هم ارتفاع میشن اشتباهه!
درسته توی هر عرض جغرافیایی فقط در یک زمان هم ارتفاع میشن ولی توی خیلی از عرض ها این هم ارتفاعی صورت میگیره!یکی از داده های دیگه ای که میتونه بده زمان هم ارتفاع شدنه!
بیاید مثال عددی بزنیم:
ستاره ی 1:(30degو2h) ستاره ی 2:(35degو1.5h)
حالا شما بگید عرض جغرافیایی چنده؟؟؟؟؟؟

arashgmn
07-24-2013, 06:56 PM
این که فقط در 1 عرض جغرافیایی هم ارتفاع میشن اشتباهه!
درسته توی هر عرض جغرافیایی فقط در یک زمان هم ارتفاع میشن ولی توی خیلی از عرض ها این هم ارتفاعی صورت میگیره!یکی از داده های دیگه ای که میتونه بده زمان هم ارتفاع شدنه!
بیاید مثال عددی بزنیم:
ستاره ی 1:(30degو2h) ستاره ی 2:(35degو1.5h)
حالا شما بگید عرض جغرافیایی چنده؟؟؟؟؟؟
باشه. به شکل زیر نگا کن :

http://up.avastarco.com/images/he9xtefj5pshxlqhaw2.png (http://up.avastarco.com/)
A و B همون دوستاره هستن. امای اصلن بعد و میل عددی شون رو توی شکل لحاظ نکردم. تا شکل خوب از آب در بیاد.
گفتم که سرسوی ناظر باید روی عمود منصف دو ستاره (دایره آبی رنگ) باشه. هم چنین می دونم که سرسو در هر شبانه روز ،یه دایره حول P می زنه. برای اینکه فقط 1 بار بتونم هم ارتفاعی این ها رو ببینم ، سرسو فقط یک بار باید این دایره آبی رو قطع کنه. در نقطه ای مثل X . برای همین این دایره صغیره ی سرسو باید به دایره آبی رنگ مماس شه. اما همه ی دایره صغیره های سرسو این ویژگی رو ندارن. (یه دایره صغیره ی بنفش نازک توی نیم کره جنوبی می تونی ببینی که دو بار این دایره رو قطع کرده.) واضحه که یه دایره صغیره ی دیگه هم هست که نزدیک قطب جنوب آسمونه. اما من برای این که شکل شلوغ نشه نکشیدمش.
جواب فی رو تا آخر امشب میذارم...

arashgmn
07-24-2013, 11:29 PM
[b]باشه.
...
جواب فی رو تا آخر امشب میذارم...

خب . با یه چهارجزئی و یه سینوس ها میشه نشون داد که میل نقطه ی میانی ستاره ها 32.556 درجه اس.

دایره ی آبی بر دایره سبز رنگ عموده. و همین به ما کمک می کنه که با یه چهارجزئی دیگه تو مثلث pah یا pbh ، مقدار زاویه ی pha یا phb (که مکمل همن) رو به دست بیاریم. اختلاف اون زاویه و 90 درجه میشه زاویه ی phx که به صورت عددی میشه 38.401 درجه.

حالا تو مثلث phx چون زاویه ی x و دایره صغیره ی بنفش رنگ بر هم مماسن ، زاویه ی x برابر با 90 میشه. Px متمم میل وسط دو ستاره ، و px جدایی زاویه ای بین سرسو و قطب شماله که هست 90 -فی. با یه سینوس های ساده عرض جغرافیایی به دست میاد : 58.429 درجه.

veno0s
07-25-2013, 10:39 PM
سلام میشه درمورقدرمطلق ستاره وفرمولش توضیح بدید؟

m.Sadat
07-25-2013, 11:53 PM
سلام میشه درمورقدرمطلق ستاره وفرمولش توضیح بدید؟

توی تاپیک اختر فیزیک از دید المپیادی به طور کامل بحث شده

storm
07-26-2013, 12:00 AM
سلام میشه درمورقدرمطلق ستاره وفرمولش توضیح بدید؟

در واقع ستاره مجهول رو با ستاره ای که فاصله و قدرش رو داریم مقایسه میکنیم
در واقع ستاره رو میذاریم در فاصله ی معین ( 1پارسک) و محاسبات لازم رو انجام بدیم

A.mousavi
07-26-2013, 02:10 AM
البته سناره رو می زاریم تو فاصله‌ی 10 پارسکی که یه لگاریتم اون وسطا 1 بشه

veno0s
07-26-2013, 05:03 PM
سلام

1.به نظرمی رسدکه خورشیدباقدر26/5-اززمین دیده می شود.هرگاه خورشیدازپلوتون که 40باردورتراست دیده شودچه قدری بایدداشته باشد؟


2.اگردمای ستاره ای از3000kبه6000kبرسدافزایش درخشندگی توام بااین تغییردماچقدرخواهدبود;به فرض ان که:(الف)قطرستاره تغییرنکند;(ب)قطرستاره نصف شود؟

univers
07-26-2013, 09:45 PM
سلام به شما
در مورد سوال اول می دانیم قدر ظاهری خورشید برابر 26.8- است(تا اونجایی که من می دونم) حالا می گیریم 26.5- از دید زمین و می دانیم که: m1-m2=2.5 log L2/L1
و ما می دانیم که درخشندگی با فاصله نسبت عکس مجذوری دارد پس: m1-m2=5log d1/d2
سوال دوم هم الآن حال ندارم بنویسم شاید آخر شب نوشتم !!!!!!!!!!ببخشید.

univers
07-27-2013, 01:38 AM
می دانیم درخشندگی یک ستاره برابر است با L= σT^4 4πr^2 که در آن T دما و r شعاع ستاره و σ نیز یک ضریب است و برابر 5.68*10^-8 است.(خدا کنه درست گفته باشم.)
برای قسمت الف وقتی قطر ثابت است پس شعاع نیز ثابت است و فقط شما باید T را در رابطه جایگذاری کنید وببینید چقدر تفاوت می کند.
برای قسمت ب نیز مانند قسمت الف عمل کنید فقط تغییرات شعاع را نیز در آن در نظر بگیرید.

المپیاد نجوم
07-28-2013, 09:38 PM
سلام.
یه سوال دارم.
rotation curveکهکشان چه ارتباطی به فاصله اش از ما داره؟مگه فاصله تا مرکز کهکشان نیست که مهمه؟
آخه توی امتحان تئوری سوال آخرمون اومده که:
برای زسم منحنی دورانی یک کهکشان نیاز به دانستن فاصله ی آن از خودمان داریم.برای تعیین فاصله ی کهکشان از یک متغیر rrشلیاقی استفاده کرده ایم اگر میزان خطا در قدر ذاتی این نوع از ستارگان متغیر برابر 0.1 باشد با صرفنظر از اثرات انبساط هابلی:
خطا در سرعت تخت شدگی(؟!)و محل شروع تخت شدگی چند درصد است؟
و بچه هایی که دوره هستین من کرویم خیلی ضعیفه لطفا سوال 3 آزمون قسمت اول رو بهم راهنمایی کنید.
ممنون!

celestial boy
07-28-2013, 10:31 PM
من جوابش ندادم.
ولی سر امتحان کلی فک کردم.سرعت تخت شدگی اصلا ربطی به فاصله نداره و نباید در این سوال خطایی داشته باشه.چون سرعت از انتقال دوپلر اندازه گیری میشه.مگه اینکه سرعت مماسی هم مورد نظر باشه.
بقیه هم فقط دوتا فرمول ساده لازم دارن.(البته من امروز که چک کردم دیدم خطای هر سه تا یکی شد!!!چی شد؟!!!!!

starscream4002
07-28-2013, 10:48 PM
سلام.
یه سوال دارم.
rotation curveکهکشان چه ارتباطی به فاصله اش از ما داره؟مگه فاصله تا مرکز کهکشان نیست که مهمه؟
آخه توی امتحان تئوری سوال آخرمون اومده که:
برای زسم منحنی دورانی یک کهکشان نیاز به دانستن فاصله ی آن از خودمان داریم.برای تعیین فاصله ی کهکشان از یک متغیر rrشلیاقی استفاده کرده ایم اگر میزان خطا در قدر ذاتی این نوع از ستارگان متغیر برابر 0.1 باشد با صرفنظر از اثرات انبساط هابلی:
خطا در سرعت تخت شدگی(؟!)و محل شروع تخت شدگی چند درصد است؟
و بچه هایی که دوره هستین من کرویم خیلی ضعیفه لطفا سوال 3 آزمون قسمت اول رو بهم راهنمایی کنید.
ممنون!

نکته دقیقا همین بود که سرعت به فاصله ربطی نداره! بنابراین سرعت خطایی نداره!

mjaasgari
07-30-2013, 04:43 PM
سلام ببخشید یه سوال
مشتق یه تابع رو که بگیریم مساوی صفر قرار بدیم min&max
تابع (نقاطش) بدست میاد حالا میخوایم ببینیم که کدوم min وکدوم max هست چیکار کنیم؟ با مشتق دوم میشه کاری کرد؟
------------------------------------------------------
یه لطف کنید طرز کار و تکنیک های کار با نرم افزار STARRY NIGHT pro plus 6
رو بگید و اینکه بعضی بچه ها توصیه میکنن که یه آسمون خام از این برنامه پرینت بگیرم و حل کنم.لطفا راهنمایی کنید چگونه؟

arashgmn
07-30-2013, 05:35 PM
سلام ببخشید یه سوال
مشتق یه تابع رو که بگیریم مساوی صفر قرار بدیم min&max
تابع (نقاطش) بدست میاد حالا میخوایم ببینیم که کدوم min وکدوم max هست چیکار کنیم؟ با مشتق دوم میشه کاری کرد؟
------------------------------------------------------
یه لطف کنید طرز کار و تکنیک های کار با نرم افزار STARRY NIGHT pro plus 6
رو بگید و اینکه بعضی بچه ها توصیه میکنن که یه آسمون خام از این برنامه پرینت بگیرم و حل کنم.لطفا راهنمایی کنید چگونه؟

مشتق یه تابع آهنگ تغییرات اون تایع رو میگه. حالا چون مشتق همون شیب نموداره ، برای همین مشتق دوم میشه آهنگ تغییرات شیب. حالا اگه دقت کنید توی قله ی نمودار ، شیب صفره. اما هرچی جلوتر بریم ، شیب منفی و منفی تر میشه. یعنی تغییرات شیب (که همون مشتق دومه)منفیه. برای یه دره هم به همین صورت میشه نشون داد که مشتق دوم مثبته.

البته توجه کنید که این روش فقط کمینه و بیشینه های موضعی رو می ده و اگه دنبال کمینه و بیشینه ی مطلقید ، باید مقدار عددی تابع رو توی نقاط بحرانی پیدا کنید. (نقاط بجرانی : ابتدا و انتهای بازه ، جاهایی که مشتق صفره یا تعریف نمیشه )

--------------------
برای طرز کار با استاری نایت هم می تونید به این تاپیک مراجعه کنید : تمام ترفندهای استاری نایت (http://forum.avastarco.com/forum/showthread.php?264-%D8%AA%D9%85%D8%A7%D9%85-%D8%AA%D8%B1%D9%81%D9%86%D8%AF%D9%87%D8%A7%DB%8C-%D8%A7%D8%B3%D8%AA%D8%A7%D8%B1%DB%8C-%D9%86%D8%A7%DB%8C%D8%AA)

univers
07-31-2013, 01:41 AM
یه سوال داشتم
آنومالی خروج از مرکزی (e) در کجای هذلولی تعریف می شود ؟(منظورم اینه که جاش روی شکل کجاست)
ممنون میشم جواب بدید.

m.Sadat
07-31-2013, 11:03 AM
یه سوال داشتم
آنومالی خروج از مرکزی (e) در کجای هذلولی تعریف می شود ؟(منظورم اینه که جاش روی شکل کجاست)
ممنون میشم جواب بدید.
اون E که با توابع هایپر بولیک تعریف میشه زاویه نیست که بشه برای اون تعبیر هندسی پیدا کرد و صرفا یک عبارت جبری هست ولی بر اساس اتحاد SEC^2-TAN^2=1 میشه یک جور آنومالی دیگه براش تعریف کرد که تعبیر هندسی اون به صورت زیر است :


http://up.avastarco.com/images/amxbbzspig3umm60hiv.png (http://up.avastarco.com/)

univers
07-31-2013, 12:13 PM
ببخشید فکر کنم شما منظورم منو بد فهمیدید
منظورم همین بود دیگه.d:

Arefe
07-31-2013, 01:19 PM
ببخشيد من اين نمودارو نميفهمم. اگه ممكنه بيشتر توضيح بديد

m.Sadat
07-31-2013, 03:30 PM
ببخشيد من اين نمودارو نميفهمم. اگه ممكنه بيشتر توضيح بديد
یک دایره به شعاع a هذلولی و مرکز مبدا مختصات رسم میکنیم بعد یک نقطه دلخواه روی هذلولی در نظر میگیریم و از اونجا به محور X عمود میکنیم بعد از اون نقطه بر اون دایره مماس میکنیم یک مثلث قائم الزاویه تشکیل میشه که اون زاویه ای که مشخص کرده همون آنومالی خروج از مرکزی برای هذلولی است

shariatzadeh
07-31-2013, 03:42 PM
یه سوال داشتم
آنومالی خروج از مرکزی (e) در کجای هذلولی تعریف می شود ؟(منظورم اینه که جاش روی شکل کجاست)
ممنون میشم جواب بدید.

آنومالی خروج از مرکزی که توی هذلولی معمولا با F نشونش میدن یه تعبیری از مساحت داره.
به شکل زیر نگاه کنید

http://up.avastarco.com/images/799krn6ocahxcl7k.png (http://up.avastarco.com/)

فرض کنید که معادله هذلولی که کشیده شده معادله متعارف هذلولی با نیم قطر بزرگ a و نیم قطر کوچک b باشه.
حالا مساحت ناحیه خاکستری رو یک بار بر حسب زاویه t و یک بار بر حسب زاویه تتا بدست بیارید . حالا سعی کنید این مساحت رو به آنومالی خروج از مرکزی هم ربط بدید.
چه نتیجه ای گرفتید؟

المپیاد نجوم
08-02-2013, 10:42 PM
سلام.
من یک مشکل توی کروی پیدا کردم.
وقتی یه جسمی پشت یه تپه یا چنین چیزی میره و ما می خواهیم ببینیم چه قدر بعد از طلوع واقعی ما طلوع ظاهری اونومی بینیم اگر اون جسم فرضا کروی نبود یا اصلا متقارن نبود ما چه طور بفهمیم که چه قدر بعد دیده میشه؟آخه نه تنها ارتفاعش بلکه سمتش هم عوض میشه و معلوم نیست از کجای اون جسم نامتقارن نور معلوم میشه!مثلا سوال زیراز مجموعه سوالات آقای چرتاب:
اول بهار می خواهیم طلوع خورشید رو ببینیم ولی دقیقا توی نقطه ی شرق یه تپه به شکل نیم بیضی هست که نیم محور بزرگش دقیقا مماس بر افقه و نیم محور کوچیک و بزرگشو با اندازه گیری برحسب فاصله ی ناظر از اون تپه به دست آوردیم که هستند:a=0.08d
b=0.05d
حالا به دست بیارین چن دقیقه بعد طلوع واقعی ستاره طلوع ظاهری(از دید ما)می کنه؟
یا سوال 3 مجموعه سوالات آقای مرادی هم همین مدله!!!
یه سوال خیلی ساده لوحانه دارم!من درک نمی کنم که وقتی یه ناظر میره روی کوه و کوه با افق یه زاویه ای داره چه طوریه که اگه افت افقو در نظر نگیریم ناظر همون چیزیو می بینه که ناظر روی زمین می بینه درحالیکه افقش همون افق نیست و به اندازه ی زاویه ی کوه با زمین زاویه دارن این دوتا افق!و تواین مورد من حتی فکر کنم مفهوم سرسو رو هم نفهمیدم!الان سرسوی ناظر روی کوه با سرسوی ناظر روی زمین یکیه؟سرسو دقیقا چه طوری تعریف میشه؟
تو مورد ناظر روی کوه تنها چیزی که موجب میشه چیزی که می بینیم یه چیز دیگه باشه(با تاخیر)اینه که جسم توی سایه ی مخروط کوه افتاده باشه؟)
و آخرین سوال!
وقتی ناظر ما جلوش یه تپه ی نیم کروی هست و می خوایم ببینیم چه بخشی از آسمونو نمی بینه این که ناظر قد داشته باشه باعث میشه که اون بخشی از عرقچین که تیره شده کوچیک تر شه حالا اگه ناظر قدش از کوه بلند تر باشه کوه هیچی از آسمونو نمی پوشونه؟کلا به جز افت افق من نسبت به تاثیرات قد ناظر توی مسائل توجیه نیستم!شرمنده که اشکالاتم خیلی بدیهی هستند!من واقعا تو کروی ضعف دارم...

storm
08-03-2013, 10:46 AM
سلام.
من یک مشکل توی کروی پیدا کردم.
وقتی یه جسمی پشت یه تپه یا چنین چیزی میره و ما می خواهیم ببینیم چه قدر بعد از طلوع واقعی ما طلوع ظاهری اونومی بینیم اگر اون جسم فرضا کروی نبود یا اصلا متقارن نبود ما چه طور بفهمیم که چه قدر بعد دیده میشه؟آخه نه تنها ارتفاعش بلکه سمتش هم عوض میشه و معلوم نیست از کجای اون جسم نامتقارن نور معلوم میشه!مثلا سوال زیراز مجموعه سوالات آقای چرتاب:
اول بهار می خواهیم طلوع خورشید رو ببینیم ولی دقیقا توی نقطه ی شرق یه تپه به شکل نیم بیضی هست که نیم محور بزرگش دقیقا مماس بر افقه و نیم محور کوچیک و بزرگشو با اندازه گیری برحسب فاصله ی ناظر از اون تپه به دست آوردیم که هستند:a=0.08d
b=0.05d
حالا به دست بیارین چن دقیقه بعد طلوع واقعی ستاره طلوع ظاهری(از دید ما)می کنه؟
یا سوال 3 مجموعه سوالات آقای مرادی هم همین مدله!!!
یه سوال خیلی ساده لوحانه دارم!من درک نمی کنم که وقتی یه ناظر میره روی کوه و کوه با افق یه زاویه ای داره چه طوریه که اگه افت افقو در نظر نگیریم ناظر همون چیزیو می بینه که ناظر روی زمین می بینه درحالیکه افقش همون افق نیست و به اندازه ی زاویه ی کوه با زمین زاویه دارن این دوتا افق!و تواین مورد من حتی فکر کنم مفهوم سرسو رو هم نفهمیدم!الان سرسوی ناظر روی کوه با سرسوی ناظر روی زمین یکیه؟سرسو دقیقا چه طوری تعریف میشه؟
تو مورد ناظر روی کوه تنها چیزی که موجب میشه چیزی که می بینیم یه چیز دیگه باشه(با تاخیر)اینه که جسم توی سایه ی مخروط کوه افتاده باشه؟)
و آخرین سوال!
وقتی ناظر ما جلوش یه تپه ی نیم کروی هست و می خوایم ببینیم چه بخشی از آسمونو نمی بینه این که ناظر قد داشته باشه باعث میشه که اون بخشی از عرقچین که تیره شده کوچیک تر شه حالا اگه ناظر قدش از کوه بلند تر باشه کوه هیچی از آسمونو نمی پوشونه؟کلا به جز افت افق من نسبت به تاثیرات قد ناظر توی مسائل توجیه نیستم!شرمنده که اشکالاتم خیلی بدیهی هستند!من واقعا تو کروی ضعف دارم...

در مورد سوالتون راجع به سر سو..
همه ی این ویژگی ها وابسته با ناظر هستند
ناظر وقتی یه جا میایستد، بالای سرش میشه : سرسو. کف پاش میشه پاسو.
و قطعا سر سوی ناظری که روی کوهه با اونی که روی زمینه فرق داره با توجه به اینکه سر سوی هر کدوم بالای سر خودشه.

در مورد سوال آخرتون هم من دقیقا متوجه نشدم..ناظر قد هم نداشته باشه(یعنی 0 در نظر بگیریم) باز هم خب قسمتی از آسمون رو نمیبینه.کوه مانعه .:-؟
حالا اگه ناظر قدش از کوه بلند باشه که دیگه مشکلی نیس
همه جارو میتونه ببینه دیگه(چون در صورتی که یه جسمی مثل تپه جلوش باشه افقش میشه از ارتفاع اون کوه یه خط مماس رسم شده. که بقیه ی آسمون براش قابله دیدنه)
توی کروی فقط باید تصور کرد و تصور.

univers
08-03-2013, 11:09 AM
من تو حساب کردن مساحت بین دو مقطع مخروطی مشکل دارم.چن وقت پیش یه سوال دیدم که می گفت مساحت مشرک دو سهمی رو پیدا کنید.
اینم متن سوال:مساحت مشترک بین دو سهمی به معادلات زیر چقدر است؟ 2p/1+cosθ و 2p/1-cosθ

arashgmn
08-03-2013, 08:45 PM
سلام.
من یک مشکل توی کروی پیدا کردم.
وقتی یه جسمی پشت یه تپه یا چنین چیزی میره و ما می خواهیم ببینیم چه قدر بعد از طلوع واقعی ما طلوع ظاهری اونومی بینیم اگر اون جسم فرضا کروی نبود یا اصلا متقارن نبود ما چه طور بفهمیم که چه قدر بعد دیده میشه؟آخه نه تنها ارتفاعش بلکه سمتش هم عوض میشه و معلوم نیست از کجای اون جسم نامتقارن نور معلوم میشه!مثلا سوال زیراز مجموعه سوالات آقای چرتاب:
اول بهار می خواهیم طلوع خورشید رو ببینیم ولی دقیقا توی نقطه ی شرق یه تپه به شکل نیم بیضی هست که نیم محور بزرگش دقیقا مماس بر افقه و نیم محور کوچیک و بزرگشو با اندازه گیری برحسب فاصله ی ناظر از اون تپه به دست آوردیم که هستند:a=0.08d
b=0.05d
حالا به دست بیارین چن دقیقه بعد طلوع واقعی ستاره طلوع ظاهری(از دید ما)می کنه؟
یا سوال 3 مجموعه سوالات آقای مرادی هم همین مدله!!!
یه سوال خیلی ساده لوحانه دارم!من درک نمی کنم که وقتی یه ناظر میره روی کوه و کوه با افق یه زاویه ای داره چه طوریه که اگه افت افقو در نظر نگیریم ناظر همون چیزیو می بینه که ناظر روی زمین می بینه درحالیکه افقش همون افق نیست و به اندازه ی زاویه ی کوه با زمین زاویه دارن این دوتا افق!و تواین مورد من حتی فکر کنم مفهوم سرسو رو هم نفهمیدم!الان سرسوی ناظر روی کوه با سرسوی ناظر روی زمین یکیه؟سرسو دقیقا چه طوری تعریف میشه؟
تو مورد ناظر روی کوه تنها چیزی که موجب میشه چیزی که می بینیم یه چیز دیگه باشه(با تاخیر)اینه که جسم توی سایه ی مخروط کوه افتاده باشه؟)
و آخرین سوال!
وقتی ناظر ما جلوش یه تپه ی نیم کروی هست و می خوایم ببینیم چه بخشی از آسمونو نمی بینه این که ناظر قد داشته باشه باعث میشه که اون بخشی از عرقچین که تیره شده کوچیک تر شه حالا اگه ناظر قدش از کوه بلند تر باشه کوه هیچی از آسمونو نمی پوشونه؟کلا به جز افت افق من نسبت به تاثیرات قد ناظر توی مسائل توجیه نیستم!شرمنده که اشکالاتم خیلی بدیهی هستند!من واقعا تو کروی ضعف دارم...

در مورد سوال مجموعه تمرینات آقای چرتاب یا آقای مرادی ، باید بگم که توی جفت سوالا اجسام نقطه ای در نظر گرفته میشن. برای همین منظور از طلوع ، طلوع نقطه ی مرکز (خورشید یا ستاره) اس...

دو مورد اون ابهامی که راجع به افق ناظر بالا و پایین کوه وجود داره ، میشه بیشتر توضیح بدین ؟ من الان نفهمیدم مشکل کجاس.

راجع یه تپه ی نیم کروی شاید شکل زیر بتونه کمک کنه. نقاط 1 و 2 و 3 ، محل چشم یه سری ناظره. یکی قدش بلنده یکی کوتاه و دیگری نزدیکه به کوه.

http://up.avastarco.com/images/kszwyw2nm8wewkazte.png (http://up.avastarco.com/)

اون خط هایی که به تپه مماس شدن ، زاویه ای رو نشون میدن که اون تپه از آسمون اشغال کرده. برای ناظر 1 معلومه که هیچ زاویه ای از آسمون اشغال نشده. برای ناظر 2 خط مماس تقریبا با افق موازیه اما اگه کمی با دقت نگاه کنیم ، می بینیم که با افق زاویه داره . و این زاویه هه بر خلاف ناظر قبل ، به سمت آسمون نشونه رفته و معلومه که ما ازتفاع های بیشتر از این خط رو می بینیم و یه سری ستاره ها با ارتفاع پایین تر ، از دید مون پنهان می مونن. در مورد ناظر 3 وضعیت شدید تره و کسر بزرگی از آسمون توسط تپه اشغال شده. البته باید دقیقا با توجه به ناحیه ی اشغال شده ، زاویه فضایی ای که کوه اشغال می کنه رو حساب کنیم و از (4 * پی) کل کره ی آسان کم کنیم...

اگر مشکلی بود باز بپرسید...

arashgmn
08-03-2013, 08:59 PM
من تو حساب کردن مساحت بین دو مقطع مخروطی مشکل دارم.چن وقت پیش یه سوال دیدم که می گفت مساحت مشرک دو سهمی رو پیدا کنید.
اینم متن سوال:مساحت مشترک بین دو سهمی به معادلات زیر چقدر است؟ 2p/1+cosθ و 2p/1-cosθ

معادله ای که به شما داده شده ، معادله ی قطبیه سهمیه. توی معادله ی قطبی ، r فاصله از مرکز مختصات و تتا هم زاویه با جهت مثبت محور x تعریف میشه. بنابراین این دوتا سهمی این شکلی هستن :

http://up.avastarco.com/images/keio5m8t82h19htodqdy.png (http://up.avastarco.com/)

حالا به توجه به تقارنی که مسئله داره ، میشه ناحیه ای که یکی از سهمی ها از زاویه ی صفر تا 90 درجه جاروب می کنه رو با انتگرال حساب کرد و بعد ضرب در 4 کرد. (البته من چون با Paint سهمی ها رو کشیدم ، تقارن شکل کمی به هم خورده...! )

المپیاد نجوم
08-03-2013, 10:28 PM
بله.توی سوال آقای چرتاب می دونم نقطه ایه ولی بازم نمی تونم حسابش کنم آخه ستاره صاف که تو آسمون جابه جا نمیشه!میشه لطفا حل این سوالو بذارین؟حلشو اگه ببینم مشکلم رفع میشه!

m.Sadat
08-03-2013, 10:51 PM
بله.توی سوال آقای چرتاب می دونم نقطه ایه ولی بازم نمی تونم حسابش کنم آخه ستاره صاف که تو آسمون جابه جا نمیشه!میشه لطفا حل این سوالو بذارین؟حلشو اگه ببینم مشکلم رفع میشه!
درسته که صاف جا به جا نمیشه ولی اون فاصله انقدر کوچیکه که میشه با یک خط تقریبش زد

mjaasgari
08-05-2013, 01:50 PM
با سلام چند تا سوال دارم:
1-(مثال 6-4 کتاب کلپنر)اگر سیاره ای داشته باشیم و بخواهیم سفینه ای را به سمت آن پرتاب کنیم اگر جاذبه نباشد باید در مساحتhttp://latex.codecogs.com/gif.latex?%5Cpi%20r%5E2 سیاره با شعاع rمیباشدحال اگر گرانش باشد آنگاه میتوان سفینه را در مساحت http://latex.codecogs.com/gif.latex?Aآن را پرتاب کردمانند شکل زیر.
5998
همینطور که دیده میشه در عکس .خوب سوال من اینه که اون خط مسیر آخر که بالاتر از سیاره رد میشه هم ممکنه که با چند دور چرخش به دور سیاره به اون برخورد کنه؟درسته؟اگه درسته چرا در حل سوال در نظر گرفته نشده ؟حال اگر بخواهیم این موضوع را هم رد نظر بگیریم حل سوال به چه شکل میشود؟
------------------------------------------------------------------------------------
این کتاب اختر فیزیک ستاره ای اریکا بوم هم که برا خوندش باید علم منظور شناسی بلد باشی؟؟؟؟
-----------------------------------------------------------------------------------
2-عمق اپتیکی چیه؟من که از اریکا چیزی نفهمیدم چیکار کرده؟!!!
---------------------------------------------------------------------------------
3-تصحیح برای جذب تابش در جو زمین چیه ؟
----------------------------------------------------------------------------------
4-این سوالات هم از آخر اریکا بومه من نتونستم حلشون کنم؟
5999
---------------------------------------------------------------------------------
سوال دیگه هم اینه که ترتیب خوندن کتاب های اریکا بوم به چه ترتیبه.من از بعضی از بچه های المپی پرسیدم گفتن که اول جلد 1 بعد 3 بعد 2
میخواستم ببینم نظر شما چیه؟
----------------------------------------------------------------------------------
یه سوال دیگه هم مونده که اینجا حجمش کمه تو پست بعدی میزارم.

مممنون!

arashgmn
08-05-2013, 06:26 PM
با سلام چند تا سوال دارم:



1-(مثال 6-4 کتاب کلپنر)اگر سیاره ای داشته باشیم و بخواهیم سفینه ای را به سمت آن پرتاب کنیم اگر جاذبه نباشد باید در مساحت سیاره با شعاع rمیباشدحال اگر گرانش باشد آنگاه میتوان سفینه را در مساحت آن را پرتاب کردمانند شکل زیر.
برای دیدن سایز بزرگ روی عکس کلیک کنید

نام: 2013-08-05-404.jpg
مشاهده: 3
حجم: 23.6 کیلو بایت
همینطور که دیده میشه در عکس .خوب سوال من اینه که اون خط مسیر آخر که بالاتر از سیاره رد میشه هم ممکنه که با چند دور چرخش به دور سیاره به اون برخورد کنه؟درسته؟اگه درسته چرا در حل سوال در نظر گرفته نشده ؟حال اگر بخواهیم این موضوع را هم رد نظر بگیریم حل سوال به چه شکل میشود؟
اگه فقط گرانش سیاره رو در نظر بگیریم ، اون مسیرهایی که بدون برخورد با سیاره ، از کنارش عبور می کنن ، دلیلی نداره که به سیاره برخورد کنن. اما اگر تاثیر جو سیاره و اصطکاکش رو در نظر بگیریم ، وضعیت متفاوت میشه. الیته چون هیچ صحبتی از اصطکاک ناشی از وجود جو به میون نیومده ، واضحه که باید ازش صرف نظر کرد.


این کتاب اختر فیزیک ستاره ای اریکا بوم هم که برا خوندش باید علم منظور شناسی بلد باشی؟؟؟؟
گویا ... البته کلا کتاب اریکا بوم ، به خورده متن سختی داره.


2-عمق اپتیکی چیه؟من که از اریکا چیزی نفهمیدم چیکار کرده؟!!!
توی جذب نور ، عدد نپر به توان این میرسه : (منفی ِ کاپا × رو × طول پیمایش شده) کل این عبارت رو (بدون منفی البته) اسمشو میذارن عمق اپتیکی. همین.


3-تصحیح برای جذب تابش در جو زمین چیه ؟
احتمالا منظورش تصحیح برای خاموشی جوی بوده که عمق اپتیکیش با سکانت زاویه سرسویی رابطه داره...

البته حدس می زنم ها... شما بگید دقیقا کجا دیدید، تا دقیق روش صحبت کنیم.


4-این سوالات هم از آخر اریکا بومه من نتونستم حلشون کنم؟
توی حل برخی از سوالای اریکا بوم ، باید از جدول ها و نمودار هایی که توی متن کتابه استفاده کنید.


سوال دیگه هم اینه که ترتیب خوندن کتاب های اریکا بوم به چه ترتیبه.من از بعضی از بچه های المپی پرسیدم گفتن که اول جلد 1 بعد 3 بعد 2
میخواستم ببینم نظر شما چیه؟
اریکابوم 1 رو که باید کامل خوند. اما 2 و3 تنها قسمت هاییش نیازه. شما می تونید بیشتر اون مطالبی رو که توی 2 و 3 نیازه رو توی کتاب های دیگه مثل مادرن ، فاندامنتال و ... پیدا کنید. ترتیبش چندان اهمیت نداره . مهم اینه که متوجه بشید.

mjaasgari
08-05-2013, 06:58 PM
آقاarashgmn
اول اینکه ممنونم ازتون
بعد هم اینکه من هنوز عمق اپتیکی رو هم فرمولشو و هم مفهوم و کاربردشو نفهمیدم.اگه ممکنه توضیح بیشتر بدید.
راستی اینکه اگه بخوایم این عمق اپتیکی و تصحیح جو رو بفهمیم باید فصل 4 کروی رو بخونیم درسته؟

m.Sadat
08-05-2013, 07:42 PM
آقاarashgmn
اول اینکه ممنونم ازتون
بعد هم اینکه من هنوز عمق اپتیکی رو هم فرمولشو و هم مفهوم و کاربردشو نفهمیدم.اگه ممکنه توضیح بیشتر بدید.
راستی اینکه اگه بخوایم این عمق اپتیکی و تصحیح جو رو بفهمیم باید فصل 4 کروی رو بخونیم درسته؟

فصل 9 کتاب مدرن خیلی خوب توضیح داده

المپیاد نجوم
08-05-2013, 08:43 PM
عمق اپتیکی به قول آقای چرتاب معیاری از ترافیک سر راه وان فوتونه!یعنی مثلا شما رو از تهران می برن تبریز و ترافیک نیست و ازتون 15 تومان می گیرن حالا شما رو از خیابون آفریقا می برن تهرانپارس و چون ترافیکه بازم ازتون 15 تومان می گیرن حالا عمق اپتیکی نشون می ده چه قدر ترافیک تو مسیر وجود داره!این مفهومشه!
و فرمولش اون منفی وقتی در مورد ستاره حرف می زنیم ظاهر میشه چون تو ستاره از سطح میایم رو به داخل ولی وقتی شما مثلا جو زمین رو در نظر می گیری از داخل میری به سمت بیرون و منفی نمی ذاری صرقا یه قرارداده!
و بقیشم که معلومه دیگه!
در حد سواد ناقص خودم گفتم امیدوارم فایده ای داشته باشه!

univers
08-08-2013, 10:46 AM
ببخشید کسی اینجا ترجمه ی تمرینات آخر فصل مادرن رو نداره؟

garamaleki
08-08-2013, 11:59 AM
سلام .
لطفا این سوالو حل کنید. ممنون
http://up.avastarco.com/images/p2b63sa63lbdugb6wyz6.jpg

veno0s
08-08-2013, 05:05 PM
سلام

1.درباره ی یک ستاره یامنظومه ی ستاره ای چه فرضی بایدکردهرگاه خطوط هلیم یونیده وخطوط مولکول ها درطیف آشکارشوند؟


2.جرم هرستاره ی موجوددریک منظومه ی مزدوج راپیداکنیدهرگاه جرم کل ان ها10جرم خورشیدی وفواصل انهاازگرانیگاه به ترتیب2auو3auباشد(ب)مدت گردش انهارابیابید:

Hooman_G
08-09-2013, 07:08 PM
يه سوال داشتم كه جواب اخرشو ميدونم اما جواب من با اين جواب متفاوت هست، راهنمايي اي مي تونيد به من بكنيد؟
ناظري در فاصله (5 l) از تپه اي نيم كره مانند به شعاع(3 l) قرار دارد اگر طول قد فرد l باشد ، تپه چه درصدي از اسمان را پوشانده؟
(انتخابي ٩١)

المپیاد نجوم
08-10-2013, 07:45 PM
http://http://up.avastarco.com/images/1ypalw8qkye7b8yw.jpeg
جواب سوال تپه و ناظر رو گذاشتم که اگه خوایتین مثلا حالت ناظر بدون قدشو حل کنین بذارین حلشو...
راستی اگه سوالای انتخابی تیمو کامل دارین ممنون میشم به منم بدین.

ali zeynali
08-10-2013, 08:04 PM
سلام .
لطفا این سوالو حل کنید. ممنون
http://up.avastarco.com/images/p2b63sa63lbdugb6wyz6.jpg
حل
این سوال یک راه خوب داره.
در جواب ساعت افتابی افقی اگر فی را به فی-۹۰ و زاویه ساعتی را به ،زاویه ساعتی-۱۸۰ ،تبدیل کنید جواب بدست می آید.
حالا فکر کنید چرا این دو تا یک چیز را بیان می کنند؟؟؟!!!

arashgmn
08-11-2013, 05:01 PM
سلام

1.درباره ی یک ستاره یامنظومه ی ستاره ای چه فرضی بایدکردهرگاه خطوط هلیم یونیده وخطوط مولکول ها درطیف آشکارشوند؟


2.جرم هرستاره ی موجوددریک منظومه ی مزدوج راپیداکنیدهرگاه جرم کل ان ها10جرم خورشیدی وفواصل انهاازگرانیگاه به ترتیب2auو3auباشد(ب)مدت گردش انهارابیابید:

من راجع به سوال 1 شما نظری ندارم . میشه یه اطلاعات بیشتری بدید که از کجا دیدید این سوالو و اینکه بقیه داده های سوال چی بودن ؟ شاید کمک کنه.

راجع به سوال 2 ، میشه گفت که نسبت فاصله ی ستاره ها از گرانیگاه ، برابره با نسبت جرماشون. از طرفی چون مجموع جرمشون 10 جرم خورشیدیه ، پس میشه با دو معادله دو مجهول جرم ها برابر با 6 و 4 جرم خورشید به دست میان. دوره تناوبشون رو هم میشه به کمک قانون سوم کپلر حساب کرد. چون نیم قطر بزرگ مدارشون 5 AU هست ، پس با توجه به رابطه ی P^2 =a^3 /M که M جرم کل برحسب جرم خورشید ، a نیم قطر بزرگ برحسب واحد نجومی و P دوره تناوب برحسب ساله ، P برابر با 10 سال به دست میاد.

M33
08-11-2013, 09:18 PM
سلام یه سوال داشتم ممنون میشم اگه راهنماییم کنید.
منبعی نقطه ای در (2aو0) قرار دارد و محور y محور اینه ای است که راس ان در(0و0) است.
الف)معادله سطح این اینه را تا مرتبه 2^x به گونه ای بدست اورید که نور را در (aو0) کاملا جمع کند.
ب)کانون این سطح را با یافتن نقطه ای که پرتو های موازی با محور اینه در ان کانونی می شوند بیابید.

mjaasgari
08-12-2013, 09:41 PM
6025
من سوال ساعتو حل کردم نمیدونم چرا اینقدر جوابش طولانی شد فکر کنم ساده نکردم یا برحسب هم بدست نیوردم.
cضلع مثلث مماس بر دیوار
xفاصله بین نقطه aتا خط عمود از نقطه c به امتدادAB
b ضلع ac
hارتفاع خورشید
سینوس اول مربوط به سمت خورشید
سینوس دوم مربوط به ارتفاع خورشید
کسینوس مخرج مربوط به میل خورشید

arashgmn
08-14-2013, 01:48 PM
يه سوال داشتم كه جواب اخرشو ميدونم اما جواب من با اين جواب متفاوت هست، راهنمايي اي مي تونيد به من بكنيد؟
ناظري در فاصله (5 l) از تپه اي نيم كره مانند به شعاع(3 l) قرار دارد اگر طول قد فرد l باشد ، تپه چه درصدي از اسمان را پوشانده؟
(انتخابي ٩١)

الان یعنی فاصله ی بین ناظر و مرکز تپه |5 هستش ؟ یا این که فاصله ی تا دامنه ی تپه ؟


http://http://up.avastarco.com/images/1ypalw8qkye7b8yw.jpeg
جواب سوال تپه و ناظر رو گذاشتم که اگه خوایتین مثلا حالت ناظر بدون قدشو حل کنین بذارین حلشو...
راستی اگه سوالای انتخابی تیمو کامل دارین ممنون میشم به منم بدین.

خوب لینکشم میذاشتید.

ali zeynali
08-14-2013, 05:30 PM
[QUOTE=arashgmn;66693]الان یعنی فاصله ی بین ناظر و مرکز تپه |5 هستش ؟ یا این که فاصله ی تا دامنه ی تپه ؟


فاصله تا مرکز ۵l ه. این سواله انتخابی تیمه امساله.
راه حلشم اینجوریه:
اول ارتفاع مشاهده شده ی مرکز کره رو پیدا کنید. بعد شعاع زاویه ای اون. بعد افق را ازش رد کنید تا اون دایره صغیره رو در ۲ نقطه قطع کنه. حالا فقط باید مساحت قسمتی از دایره صغیره رو که بالای افقه محاسبه کنید.

arashgmn
08-14-2013, 07:37 PM
6025
من سوال ساعتو حل کردم نمیدونم چرا اینقدر جوابش طولانی شد فکر کنم ساده نکردم یا برحسب هم بدست نیوردم.
cضلع مثلث مماس بر دیوار
xفاصله بین نقطه aتا خط عمود از نقطه c به امتدادAB
b ضلع ac
hارتفاع خورشید
سینوس اول مربوط به سمت خورشید
سینوس دوم مربوط به ارتفاع خورشید
کسینوس مخرج مربوط به میل خورشید

سعی کنید عبارتتون رو با استفاده از روابط هندسی مثل این ساده کنید : (طول سایه )×(تانژانت ارتفاع) = (طول شاخص عمودی) . شاید ساده تر شد عبارتتون.


سلام یه سوال داشتم ممنون میشم اگه راهنماییم کنید.
منبعی نقطه ای در (2aو0) قرار دارد و محور y محور اینه ای است که راس ان در(0و0) است.
الف)معادله سطح این اینه را تا مرتبه 2^x به گونه ای بدست اورید که نور را در (aو0) کاملا جمع کند.
ب)کانون این سطح را با یافتن نقطه ای که پرتو های موازی با محور اینه در ان کانونی می شوند بیابید.

یه راه معروفی وجود داره که میاد یه جسم رو روی سطح آینه در حال حرکت در نظر می گیره و از دید دو نقطه ، سرعت های شعاعی رو بررسی میکنه. اگه یه یه خم دلخواه رو به عنوان آینه در نظر بگیرید ، و از دید دو نقطه سرعت شعاعی رو بنویسید ، به این نتیجه می رسید که اندازه های سرعت شعاعی ها باید یکسان باشه (اما جهتشون برعکس) ! از روی این میشه به این نتیجه رسید که این خم باید یه بیضی عمودی باشه با نیم قطر بزرگ برابر با 1.5A که بر محور x ها مماسه.

mhfire
08-16-2013, 01:00 PM
سلام
اگر 2 بعدی است خم داریم نه سطح: شما معادله عمومی درجه2 رو بنویس ((x,Y(x) حالا باید هر دو معادله خط بین نقطه دلخواه روی خم و دو نقطه‌مفروض بر هم عمود باشن => جواب
با کمی دقت می‌شه فهمید این یکی از خواص بیضی در کانون‌ها است => جواب

univers
08-22-2013, 12:38 PM
سلام
اگه میشه راه حل خودتون رو برای سوال زیر بگید.
http://up.avastarco.com/images/ajfu6ayyrsueradfsr.png

m.Sadat
08-22-2013, 01:18 PM
سلام
اگه میشه راه حل خودتون رو برای سوال زیر بگید.
http://up.avastarco.com/images/ajfu6ayyrsueradfsr.png

ابتدا باید حساب کنیم که پس از 5 میلیارد سال کهکشان در چه فاصله ای از ما قرار داره

http://up.avastarco.com/images/e5hf5lr3fx6g892j2q5.png (http://up.avastarco.com/)

پس سفینه به جای 10 مگا پارسک باید این فاصله رو بره در نتیجه داریم :

http://up.avastarco.com/images/7nvf8rz0l1p76j49xke8.png (http://up.avastarco.com/)

که با عدد گذاری 2820.89 کیلومتر بر ثانیه بدست میاد
موفق باشید :thumbsup:

mjaasgari
08-22-2013, 04:38 PM
سلام
اگه میشه راه حل خودتون رو برای سوال زیر بگید.
http://up.avastarco.com/images/ajfu6ayyrsueradfsr.png

ببخشيد اين چيزي كه آخر همين سوال نوشته شده ميخواسته چي بگه مفهومش چيه ؟
راهنمايي بوده يا .....

m.Sadat
08-22-2013, 06:23 PM
ببخشيد اين چيزي كه آخر همين سوال نوشته شده ميخواسته چي بگه مفهومش چيه ؟
راهنمايي بوده يا .....

اون تکه آخر سوال میگه که اگه مشتق یک تابع باخودش متناسب شد اون تابع نمایی هست یه جورایی جواب اون معادله دیفرانسیل اول رو داده

tina
08-22-2013, 08:53 PM
حل
این سوال یک راه خوب داره.
در جواب ساعت افتابی افقی اگر فی را به فی-۹۰ و زاویه ساعتی را به ،زاویه ساعتی-۱۸۰ ،تبدیل کنید جواب بدست می آید.
حالا فکر کنید چرا این دو تا یک چیز را بیان می کنند؟؟؟!!!
سلام
مثلثش قائم الزاویه است ؟ :-/

tina
08-23-2013, 04:08 PM
سلام
چرا خطوط طیفی ستاره ها پهن میشه ؟
ا

ali zeynali
08-23-2013, 05:45 PM
سلام
مثلثش قائم الزاویه است ؟ :-/
بله در هر دو ساعت افتابی؛چه افقی وچه عمودی؛یک مثلث قاءم الزاویه تشکیل میشه که اجزاش از فی و گاما و زاویه ساعتی خورشید تشکیل شده اند.

ali zeynali
08-23-2013, 06:24 PM
سلام
چرا خطوط طیفی ستاره ها پهن میشه ؟
ا
پهن شدگی عوامل زیادی داره ولی مهمترین عواملش:
بدلیل اینکه در شعاع لایه برانگیخته عدم قطعیت وجود داره(پهن شدگی طبیعی)
ذره ای که الکترونش برانگیخته می شود سرعتی دارد که تمام ذرات از توزیع ماکسول بولتزمن پیروی می کنه.(دوپلری)
این دو مهمترین عوامل اند ولی حتی پهن شدگی از چرخش خود ستاره هم ناشی می شود.

ali s.k
08-23-2013, 08:31 PM
سلام
لطفا معادله حرکت دایره ای در مختصات دکارتی رو بنویسید.

پیمان اکبرنیا
08-23-2013, 08:43 PM
سلام
لطفا معادله حرکت دایره ای در مختصات دکارتی رو بنویسید.

سلام دوست عزیز

در تاپیک مقاطع مخروطی (http://forum.avastarco.com/forum/showthread.php?768-%D9%85%D9%82%D8%A7%D8%B7%D8%B9-%D9%85%D8%AE%D8%B1%D9%88%D8%B7%DB%8C) درباره دایره و معادله اش در دستگاه های مختصات مختلف صحبت کردم. لیست هم داره در اول تاپیک میتونید مطالب مورد نظرتون را پیدا کنید. اونجا را مطالعه کنید اگر سوالی بود مطرح بفرمایید :)

arashgmn
08-23-2013, 10:12 PM
سلام
چرا خطوط طیفی ستاره ها پهن میشه ؟
ا
من این رو هم اضافه کنم که وجود یه میدان مغناطیسی می تونه باعث پهن شدن خطوط طیفی بشه (اثر زیمان) اگر میدان بزرگتر از مرتبه ی 0.1 تسلا باشه ، این اثر محسوسه تر میشه. عکس زیر رو ببینید.

پهن شدگی خط در اثر میدان مغناطیسی یه لکه خورشیدی

http://upload.wikimedia.org/wikipedia/commons/9/9e/Sunzeeman1919.png


عکس از ویکی پدیای انگلیسی

mjaasgari
08-27-2013, 05:36 PM
سلام
چند سوال از هالیدی؟
------------------------------------------------
1-با معلوم بودن دوره های چرخش و شعاع های مدار ماه به دور زمین و زمین به دور خورشید به راحتی میتوان نسبت جرم زمین به جرم خورشید را یافت.آیا میتوان gرا فقط از رصد ها نجومی تعیین کرد؟
-----------------------------------------------
2-چگونه میتوان جرم ماه را تعیین کرد؟(فکر کنم با پدیده رخگرد و شعاع ماه بشه)
----------------------------------------------
3-اجسام ساکن بر روی سطح زمین با دوره 24 ساعت در مدار های دایره ای حرکت میکنند آیا این اجسام مانند ماهواره ای که در مدار قرار دارند حرکت میکنند؟طول شبانه روز چقدر باشد تا این اجسام در مدار باشند؟(36 ساعت فکر کنم)چرا؟
----------------------------------------------
4-نیروی گرانشی وارد از خورشید به ماه دو برار نیروی گرانشی زمین بر ماه است پس چرا ماه از زمین نمیگریزد؟
----------------------------------------------
5-چرا استدلال زیر غلط است؟"در نیمه شب که خورشید در زیر پای ما قرار دارد و در ظهر در بالای سر ما پس در ظهر وزن ما از نیمه شب کمتر است."
----------------------------------------------
6-جاذبه گرانشی خورشید وماه بر زمین باعث جزر و مد میشود.اصر کشندی خورشید در حدود نصف اثر ماه است. با این همه کشش مستقیم خورشید بر زمین 175 برابر ماه است . پس چرا ماه جزر و مد بزرگتری دارد؟
----------------------------------------------
7-چرا با مطالعه ی حرکت یک ماهواره مصنوعی میتوان اطلاعات بیشتری از مطالعه ماه بدست آورد؟
----------------------------------------------
8-آیا یک ماهواره میتواند در مداری پایدار در صفحه ای که از مرکز زمین نمیگذرد! حرکت آزاد داشته باشد؟

arashgmn
08-28-2013, 10:38 PM
سلام
چند سوال از هالیدی؟
------------------------------------------------
1-با معلوم بودن دوره های چرخش و شعاع های مدار ماه به دور زمین و زمین به دور خورشید به راحتی میتوان نسبت جرم زمین به جرم خورشید را یافت.آیا میتوان gرا فقط از رصد ها نجومی تعیین کرد؟
-----------------------------------------------
2-چگونه میتوان جرم ماه را تعیین کرد؟(فکر کنم با پدیده رخگرد و شعاع ماه بشه)
----------------------------------------------
3-اجسام ساکن بر روی سطح زمین با دوره 24 ساعت در مدار های دایره ای حرکت میکنند آیا این اجسام مانند ماهواره ای که در مدار قرار دارند حرکت میکنند؟طول شبانه روز چقدر باشد تا این اجسام در مدار باشند؟(36 ساعت فکر کنم)چرا؟
----------------------------------------------
4-نیروی گرانشی وارد از خورشید به ماه دو برار نیروی گرانشی زمین بر ماه است پس چرا ماه از زمین نمیگریزد؟
----------------------------------------------
5-چرا استدلال زیر غلط است؟"در نیمه شب که خورشید در زیر پای ما قرار دارد و در ظهر در بالای سر ما پس در ظهر وزن ما از نیمه شب کمتر است."
----------------------------------------------
6-جاذبه گرانشی خورشید وماه بر زمین باعث جزر و مد میشود.اصر کشندی خورشید در حدود نصف اثر ماه است. با این همه کشش مستقیم خورشید بر زمین 175 برابر ماه است . پس چرا ماه جزر و مد بزرگتری دارد؟
----------------------------------------------
7-چرا با مطالعه ی حرکت یک ماهواره مصنوعی میتوان اطلاعات بیشتری از مطالعه ماه بدست آورد؟
----------------------------------------------
8-آیا یک ماهواره میتواند در مداری پایدار در صفحه ای که از مرکز زمین نمیگذرد! حرکت آزاد داشته باشد؟

سلام.
توی بعضی سوالا، با توجه به دونستن مقدار یه کمیت میشه به جواب سوا رسید که من توی متن ذکر کردم...

1-احتمالا منظورتون g بزرگـه(ثابت گرانش). اگه از قانون 3 کپلر بخوایم استفاده کنیم ، میبینیم که g ساده میشه و نمیشه خودش رو مستقلا از مشاهدات صرف به دست آورد. به آزمایش نیازه.

2-میشه باز از قانون سوم کپلر استفاده کرد... در واقع از روی دوره تناوب ماه میشه مجموع جرم زمین و ماهو حساب کرد که چون احتمالا جرم زمینو داریم ، جرم ماه به دست میاد.

3- اصلا چنین نیست !!! ما حرکتمون بر روی زمین ناشی از اصطکاکیه که زمین بهمون وارد می کنه. اگه در حال سقوط آزاد به سمت زمین بودیم، مجبور میشدیم روزی حدود 17- 18بار طلوع و غروب خورشیدو ببینیم.... در حقیقت دوره تناوبمون حول مرکز زمین میشد حدود 5000 ثانیه به جای 86400 ثانیه...

4- الان هم ماه و هم زمین در حال سقوط به سمت خورشید هستن. چون هر دوتاشون با هم دارن دور خورشیئ می چرخن. حالا توی این سقوط آزاد ناشی از نیروی خورشید، ماه گرفتار نیروی گرانش زمین هم شده و روی اون هم داره سقوط می کنه...

5-دستگاهی که ما بر روی زمین اختیار می کنیم ، دستگاه لختی نیست. چون کل زمین در حال سقوط به سمت خورشیده. حالا اگه بخوایم وزن رو توی این دستگاه نالخت حساب کنیم ، باید بگیم که اختلاف شتابی که خورشید به ما و زمین وارد می کنه ، متناظر با چه نیرویه... چون شتاب ، یه برداره ، برای همین تفاضل شتابها وقتی بیشینه میشه که در خلاف جهت هم باشن . یعنی اتفاقا در روز!

6- منظورتون از کشش مستقیم چیه ؟

7-چون میشه انحراف هایی که از مسیر انتظازی ایده آل وجود داره رو بررسی کرد و به تغییرات میدان گرانشی ناه پی برد. از روی میدان هم میشه چگالی اون نواحی رو تخمین زد.

8-یک صفحه مداری در نظر بگیرید که از مرکز زمین نمی گذره. نیرویی که زمین به این ماهواره وارد میکنه ، در راستای مرکز زمینه. یعنی در راستای خارج صفحه. پس جسم مجبوره برای حفظ مدارش ، یه جور نیروی کنترلی داشته باشه که علاوه بر خنثی کردن نیروی زمین ، اونو توی مدارش نگه داره. بنابراین این مدار به خودی خود اصلا پایدار نیست...

mjaasgari
08-28-2013, 11:15 PM
سلام.
توی بعضی سوالا، با توجه به دونستن مقدار یه کمیت میشه به جواب سوا رسید که من توی متن ذکر کردم...

1-احتمالا منظورتون g بزرگـه(ثابت گرانش). اگه از قانون 3 کپلر بخوایم استفاده کنیم ، میبینیم که g ساده میشه و نمیشه خودش رو مستقلا از مشاهدات صرف به دست آورد. به آزمایش نیازه.

2-میشه باز از قانون سوم کپلر استفاده کرد... در واقع از روی دوره تناوب ماه میشه مجموع جرم زمین و ماهو حساب کرد که چون احتمالا جرم زمینو داریم ، جرم ماه به دست میاد.

3- اصلا چنین نیست !!! ما حرکتمون بر روی زمین ناشی از اصطکاکیه که زمین بهمون وارد می کنه. اگه در حال سقوط آزاد به سمت زمین بودیم، مجبور میشدیم روزی حدود 17- 18بار طلوع و غروب خورشیدو ببینیم.... در حقیقت دوره تناوبمون حول مرکز زمین میشد حدود 5000 ثانیه به جای 86400 ثانیه...

4- الان هم ماه و هم زمین در حال سقوط به سمت خورشید هستن. چون هر دوتاشون با هم دارن دور خورشیئ می چرخن. حالا توی این سقوط آزاد ناشی از نیروی خورشید، ماه گرفتار نیروی گرانش زمین هم شده و روی اون هم داره سقوط می کنه...

5-دستگاهی که ما بر روی زمین اختیار می کنیم ، دستگاه لختی نیست. چون کل زمین در حال سقوط به سمت خورشیده. حالا اگه بخوایم وزن رو توی این دستگاه نالخت حساب کنیم ، باید بگیم که اختلاف شتابی که خورشید به ما و زمین وارد می کنه ، متناظر با چه نیرویه... چون شتاب ، یه برداره ، برای همین تفاضل شتابها وقتی بیشینه میشه که در خلاف جهت هم باشن . یعنی اتفاقا در روز!

6- منظورتون از کشش مستقیم چیه ؟

7-چون میشه انحراف هایی که از مسیر انتظازی ایده آل وجود داره رو بررسی کرد و به تغییرات میدان گرانشی ناه پی برد. از روی میدان هم میشه چگالی اون نواحی رو تخمین زد.

8-یک صفحه مداری در نظر بگیرید که از مرکز زمین نمی گذره. نیرویی که زمین به این ماهواره وارد میکنه ، در راستای مرکز زمینه. یعنی در راستای خارج صفحه. پس جسم مجبوره برای حفظ مدارش ، یه جور نیروی کنترلی داشته باشه که علاوه بر خنثی کردن نیروی زمین ، اونو توی مدارش نگه داره. بنابراین این مدار به خودی خود اصلا پایدار نیست...

سلام خسته نباشيد ممنون
اينا سوالاي آخر فصل گرانش هاليدي بود
سوال ٦ رو خودمم نميفهمم.
سوال ٥ يعني در روز وزن مابيشتره ديگه اينت منطقي كه با بردار گفتيد رو قبول كرديم و با شهود يكم نا سازگاره؟

arashgmn
08-29-2013, 12:06 AM
سلام خسته نباشيد ممنون
اينا سوالاي آخر فصل گرانش هاليدي بود
سوال ٦ رو خودمم نميفهمم.
سوال ٥ يعني در روز وزن مابيشتره ديگه اينت منطقي كه با بردار گفتيد رو قبول كرديم و با شهود يكم نا سازگاره؟

من ترجیح میدم عکس زیر رو بذارم . (چه می کنه این پینت :دی )

http://up.avastarco.com/images/zkgmijy8gbzoyf3p83v0.png (http://up.avastarco.com/)

توجه کنید که از دید ناظر روی زمین ، مجموع نیرو های وارد بهش باید صفر بشه. برای همین نیروی قرمز و آبی کمرنگ و بنفش ، هم دیگه رو خنثی میکنن. (توجه کنید که نیروی آبی کمرنگ واقعا وجود نداره و یه نیروی مجازیه که به علت نالخت بودن دستگاه داره وارد میشه... و متناظره با شتاب سقوط آزاد زمین به سمت خورشید...)

الان فک کنم دیگه از نظر شهودی هم مشکل حل شده باشه ;)

mjaasgari
08-29-2013, 08:00 AM
سلام مجدد ببخشيد يه سوال ديگه
يعني با رخگرد ماه نميشه جرمشو حساب كرد با رخگرد ميتونيم دوره تناوب گردش ماه به دور خودش رو پيدا كنيم. بعد با قانون سوم كپلر و دونستن شعاع ماه جرمش رو بگيريم.
البته يه فرضي نياز داره كه نميدونم درسته يانه ؟
بايد فرض كنيم كل جرم ماه تو مركزشه(اصل دوم پوسته كروي)و نيز يه جرمي روي سطح ماه داره به همون دوره تناوب ماه به دور مركزش ميگرده.
ابن فرض آخريه فمر نكنم درست باشه.

arashgmn
08-29-2013, 05:19 PM
سلام مجدد ببخشيد يه سوال ديگه
يعني با رخگرد ماه نميشه جرمشو حساب كرد با رخگرد ميتونيم دوره تناوب گردش ماه به دور خودش رو پيدا كنيم. بعد با قانون سوم كپلر و دونستن شعاع ماه جرمش رو بگيريم.
البته يه فرضي نياز داره كه نميدونم درسته يانه ؟
بايد فرض كنيم كل جرم ماه تو مركزشه(اصل دوم پوسته كروي)و نيز يه جرمي روي سطح ماه داره به همون دوره تناوب ماه به دور مركزش ميگرده.
ابن فرض آخريه فمر نكنم درست باشه.
سلام! چرا ببخشیم؟ پرسیدن که عیب نیست ...

اول قفل مداری:
ما همین الان میدونیم که ماه فقل مداری شده. یعنی یه روش همیشه رو به زمینه. به همین خاطر دوره تناوبش دور خودش با دوره تناوبش به دور زمین با هم برابرن.

دوم رخ گرد:
این که ماه قفل فقط یه سمتش به ما هست ، باعث میشه که ما همیشه نصف ماهو ببینیم. اما بیضوی بودن مدار ماه گرد زمین و همین طور داشتن میل مداری، باعث میشه که ما کمی بیشتر از 50 درصد ماه رو ببییم. که به این ها رخگرد (افقی و عمودی) میگن.

سوم جرم ماه:
جواب سوال 3 همین رو می گفت. ذرات سطح ماه به علت جاذبه به دور ماه نمی گردن. بلکه نیروی داخلی بین ذره هاس که سطح ماه رو دور مرکز ماه دوران میده. به همین خاطر نمیشه قانون سوم کپلر رو نوشت...

باز اگر سوالی پیش اومد ، بپرسید ... ;)

mjaasgari
09-01-2013, 11:53 AM
سلام مجدد
چند تا سوال مفهومی!!!؟
1-آیا مفهوم فیزیکی گرا دیاناین است؟"جهتی را که در آن تغییرات یک تابع بیشتر است گرادیان آن تابع گویند"
2-منظور از مدار کپلری چیه؟(یعنی ممداری که قوانین کپلر توش صدق کنه؟)
3-خوشه گویسان چیه؟تلفظش چیه؟؟؟؟؟؟ d:
4-خطوط فرانهوفر چیستند؟
5-"شید سپهر"فام سپهر"خرمن"چیست؟
6-امواج اکوستیک چیست؟
7-نظریه دینامو چیست؟
8-اثر هال و اثر زیمان چیست؟
9-طیف نگار بدون شکاف چگونه کار میکند؟
10-من تو درک یه طیف نگار یعنی طیفی که از یک ستاره در یافت کردیم و مفهوم آن و فرمول های بکار گیری از این خطوطو طیفی مشکل دارم!کلا با طیف مشکل دارم!اگه ممکنه راهنمایی کنید
ممنون

veno0s
09-03-2013, 10:54 AM
سلام
یه سوال : برای المپیادلازمه که اثبات های فرمول های سینوس وکسینوس و... بلدباشیم یا خودفرمول کافیه؟

univers
09-04-2013, 12:57 AM
سلام
یه سوال : برای المپیادلازمه که اثبات های فرمول های سینوس وکسینوس و... بلاید همه چدباشیم یا خودفرمول کافیه؟

سلام بر شما
اگه منظورتون مرحله دو هستش شما باید اونجا همه چی رو به غیر از فرمول های بنیادی اثبات کنید که فرمول سینوس ها و کسینوس ها از اون دسته موارد هستند.البته به نظر من شما اثباتش رو بلد باشید که ضرری نداره.تو اسمارت هم اثباتش هست.

mjaasgari
09-04-2013, 10:32 PM
سلام مجدد
چند تا سوال مفهومی!!!؟
1-آیا مفهوم فیزیکی گرا دیاناین است؟"جهتی را که در آن تغییرات یک تابع بیشتر است گرادیان آن تابع گویند"
2-منظور از مدار کپلری چیه؟(یعنی ممداری که قوانین کپلر توش صدق کنه؟)
3-خوشه گویسان چیه؟تلفظش چیه؟؟؟؟؟؟ d:
4-خطوط فرانهوفر چیستند؟
5-"شید سپهر"فام سپهر"خرمن"چیست؟
6-امواج اکوستیک چیست؟
7-نظریه دینامو چیست؟
8-اثر هال و اثر زیمان چیست؟
9-طیف نگار بدون شکاف چگونه کار میکند؟
10-من تو درک یه طیف نگار یعنی طیفی که از یک ستاره در یافت کردیم و مفهوم آن و فرمول های بکار گیری از این خطوطو طیفی مشکل دارم!کلا با طیف مشکل دارم!اگه ممکنه راهنمایی کنید
ممنون

كسي نيست كه سوالات مارا پاسخ دهد.

arashgmn
09-05-2013, 03:44 PM
سلام مجدد
چند تا سوال مفهومی!!!؟
1-آیا مفهوم فیزیکی گرا دیاناین است؟"جهتی را که در آن تغییرات یک تابع بیشتر است گرادیان آن تابع گویند"
2-منظور از مدار کپلری چیه؟(یعنی ممداری که قوانین کپلر توش صدق کنه؟)
3-خوشه گویسان چیه؟تلفظش چیه؟؟؟؟؟؟ d:
4-خطوط فرانهوفر چیستند؟
5-"شید سپهر"فام سپهر"خرمن"چیست؟
6-امواج اکوستیک چیست؟
7-نظریه دینامو چیست؟
8-اثر هال و اثر زیمان چیست؟
9-طیف نگار بدون شکاف چگونه کار میکند؟
10-من تو درک یه طیف نگار یعنی طیفی که از یک ستاره در یافت کردیم و مفهوم آن و فرمول های بکار گیری از این خطوطو طیفی مشکل دارم!کلا با طیف مشکل دارم!اگه ممکنه راهنمایی کنید
ممنون

1- بله همونه.

2- یعنی مداری که قوانین کپلر براشون صادق باشن (در حقیقت سیستم واقعا از دوتا جسم تشکیل شده باشه). مثلا مدار زمین به دور خورشید به خاطر اختلال هایی که ماه و دیگر سیارات وارد می کنن، کاملا کپلری نیست ...

3- خوشه ی گوی - سان (Gooy-San) همون خوشه ی کرویه.

4- همون خطوط جذبی توی طیف هستن که به افتخار ژوزف فون فرانهوفر که بیشتر از 570 خط جذبی در طیف خورشید رو شناسایی کرد ، بهشون میگن خطوط فرانهوفر. (منبع : ویکیپدیای فارسی)

5- شید سپهر ترجمه ی فارسی فوتوسفره! شید یعنی درخشان؛ این همون بخشی از خورشیده که ما می بینیمش. یعنی همون سطح خورشید.

6- نمی دونم .

7- اصلا نمی دونم.

8- اثر زیمان تو اینجا (http://fa.wikipedia.org/wiki/%D8%A7%D8%AB%D8%B1_%D8%B2%DB%8C%D9%85%D8%A7%D9%86) هست. اثر هال هم این جا (http://fa.wikipedia.org/wiki/%D8%A7%D8%AB%D8%B1_%D9%87%D8%A7%D9%84) نوشته چیه.

9- نمی دونم.

10 - طیف اینه و خطوطش هم روشه. چی رو مشکل دارید ؟

http://upload.wikimedia.org/wikipedia/commons/3/34/Fraunhofer_lines.jpg
(عکس از ویکیپدیای فارسی : صفحه ی مربوط به خطوط فرانهوفر)

mjaasgari
09-05-2013, 05:10 PM
1- بله همونه.

2- یعنی مداری که قوانین کپلر براشون صادق باشن (در حقیقت سیستم واقعا از دوتا جسم تشکیل شده باشه). مثلا مدار زمین به دور خورشید به خاطر اختلال هایی که ماه و دیگر سیارات وارد می کنن، کاملا کپلری نیست ...

3- خوشه ی گوی - سان (Gooy-San) همون خوشه ی کرویه.

4- همون خطوط جذبی توی طیف هستن که به افتخار ژوزف فون فرانهوفر که بیشتر از 570 خط جذبی در طیف خورشید رو شناسایی کرد ، بهشون میگن خطوط فرانهوفر. (منبع : ویکیپدیای فارسی)

5- شید سپهر ترجمه ی فارسی فوتوسفره! شید یعنی درخشان؛ این همون بخشی از خورشیده که ما می بینیمش. یعنی همون سطح خورشید.

6- نمی دونم .

7- اصلا نمی دونم.

8- اثر زیمان تو اینجا (http://fa.wikipedia.org/wiki/%D8%A7%D8%AB%D8%B1_%D8%B2%DB%8C%D9%85%D8%A7%D9%86) هست. اثر هال هم این جا (http://fa.wikipedia.org/wiki/%D8%A7%D8%AB%D8%B1_%D9%87%D8%A7%D9%84) نوشته چیه.

9- نمی دونم.

10 - طیف اینه و خطوطش هم روشه. چی رو مشکل دارید ؟

http://upload.wikimedia.org/wikipedia/commons/3/34/Fraunhofer_lines.jpg
(عکس از ویکیپدیای فارسی : صفحه ی مربوط به خطوط فرانهوفر)

سلام خيلي ممنون كه وقت گذاشتيد و جواب ديد
اگه ممكنه همين طيف رو يه تفسير بكنيد و خواص شو بگيد شايد انشا الله مشكل ماهم حل شد.

mhfire
09-06-2013, 02:25 PM
سلام
سوال 9 یعنی به وسیله پراش و... نه شکاف . شما روش های طیفنگاری رو مطالعه کنید
جواب 6: http://en.wikipedia.org/wiki/Acoustic_wave
7 : نظریه مربوط زمین شناسی برای ساختار ستارگان و ... استفاده می‌شود .

arashgmn
09-06-2013, 07:19 PM
سلام خيلي ممنون كه وقت گذاشتيد و جواب ديد
اگه ممكنه همين طيف رو يه تفسير بكنيد و خواص شو بگيد شايد انشا الله مشكل ماهم حل شد.

آقای Ehsan یه تاپیکی زدن به اسم طیف. اگه از اول بخونیدش و بحث رو دنبال کنید ، متوجه همه چی می شید ;)

تاپیک طیف (http://forum.avastarco.com/forum/showthread.php?1113-%D8%B7%DB%8C%D9%81)

univers
09-08-2013, 10:51 PM
سلام
من سوال زیر رو مشکل دارم از دوستانی که بلدن خواهش میکنم جواب بدین.
http://up.avastarco.com/images/l6z9ffykj7w6j5c0jsh.png
ممنون.

amir navid
09-11-2013, 09:33 PM
سلام دوستان من یه مشکلی دارم اینه که قوانین کپلر رو به طور شهودی فهمیدم ولی بیان ریاضی هر چی سرچ می کنم پیدا نمی کنم میشه یکی بیان ریاضیشو بگه ؟ در ضمن من کتاب مرجع هم ندارم...

arashgmn
09-11-2013, 11:39 PM
سلام
من سوال زیر رو مشکل دارم از دوستانی که بلدن خواهش میکنم جواب بدین.
http://up.avastarco.com/images/l6z9ffykj7w6j5c0jsh.png
ممنون.

صفحه ای رو در نظر بگیرید که از خورشید ، و تیرک می گذره. تقاطع این صفحه و افق ، دقیقا همون سایه ی تیرکه. حالا دوباره فکر کنید رو سوال ;)


سلام دوستان من یه مشکلی دارم اینه که قوانین کپلر رو به طور شهودی فهمیدم ولی بیان ریاضی هر چی سرچ می کنم پیدا نمی کنم میشه یکی بیان ریاضیشو بگه ؟ در ضمن من کتاب مرجع هم ندارم...

فایل های این پست (http://forum.avastarco.com/forum/showthread.php?564-%D9%82%D9%88%D8%A7%D9%86%D9%8A%D9%86-%D9%83%D9%BE%D9%84%D8%B1&p=12720&viewfull=1#post12720) رو نگاه کنید . خیلی آموزنده اس. اما از نظر ریاضی قوانین کپلر بیان سختی ندارن :

قانون اول : مدار سیاره ها به دور خورشید ، یک مقطع مخروطیه .

http://upload.wikimedia.org/math/e/2/3/e23e800b9c04ae4e7007c54c9f3c2c84.png

قانون دوم: آهنگ جارو شدن مساحت توی مدار یه سیاره ، ثابته.

http://upload.wikimedia.org/math/e/b/0/eb0443b789ec4c75ef5cb1804109f00a.png
قانون سوم : دوره تناوب سیاره به توان 2 ، متناسبه با فاصله ی متوسط سیاره از خورشید به توان 3.

http://upload.wikimedia.org/math/5/7/6/5761f8aaff0f33500ed21c8cf31f97b3.png

توضیحات بیشتر در ویکیپدیا (http://en.wikipedia.org/wiki/Kepler%27s_laws_of_planetary_motion) هست ! امتیاز عکس ها هم به ویکیپدیا متعلقه ! اگر باز هم بیشتر می خواید مطالعه کنید ، می تونید به فصل گرانش کتاب فیزیک هالیدی (جلد 1) مراجعه کنید.

amir navid
09-13-2013, 01:50 PM
ببخشید میشه یک توضیحی در مورد قانون اول فرمولشو بدین مثلن اون p یا اون اپسیلون چی هست یعنی جه مقدارهایی رو نشون می دن؟ اگر قانون دوم هم رو بگین خیلی خوب میشه

m.Sadat
09-13-2013, 03:10 PM
ببخشید میشه یک توضیحی در مورد قانون اول فرمولشو بدین مثلن اون p یا اون اپسیلون چی هست یعنی جه مقدارهایی رو نشون می دن؟ اگر قانون دوم هم رو بگین خیلی خوب میشه

اون p لاتوس رکتوم نام داره و به مشخصات مدار بر میگرده و برابر است با فاصله از کاننون هنگامی که تتا 90 درجه باشه
اپسیلون یا e همون خروج از مر کزه و تعریف های متعددی داره که یکی از اونها اینه :
نسبت فاصله هر نقطه روی محیط یک مقطع مخروطی از کانون به نسبت فاصله ان از خط هادی خروج از مرکز گفته میشه
قانون دوم هم میگه بردار شعاعی در زمان های مساوی مساحت های مساوی جاروب میکنه
برای مطالعه بیشتر میتونید به کتاب های مرجع در مورد مکانیک سماوی مراجعه کنید

storm
09-22-2013, 12:11 PM
سلام
من دوتا سوال اخترفيزيك داشتم
١.براي بدست اوردن جرم هاي يك دوتايي كه مدارشون بيضي هست،
و ما ميخوايم قطر بزرگ مدار نسبي رو بدست بياريم در محاسبات فاصله مدار ها تا گرانيگاه دوبار حساب ميشه كه حاصل به ما ميدهد: 2a
مشكلم اين هست كه چرا d1+d2 =a پاسخ نيست؟
چرا فواصل b1 , b2 هم بايد محاسبه بشن؟
(اريكا بوم)
٢.منحني ستاره هايي كه مدارشون دايره اي نيست، به چه شكل هست؟
در مورد زاويه ميل i و خط ديد ٩٠ درجه اش هم اگر ممكنه توضيح بدين
با تشكر :گل

storm
09-23-2013, 11:46 PM
سلامي دوباره خدمت دوستان
ما اينقدر اصرار و پافشاري ميورزيم تا به جواب اشكالاتمون برسيم:دي
سوالي كه داشتم در مورد گرفت هاست.
١. اگر شعاع دو ستاره برابر باشه گرفت جزئي نميتونيم داشته باشيم؟اگر داريم، منحني نوري اش چه شكلي ميشه؟
٢. توي منحني نوري t با انديس t مدت زمان گرفت هست؟
t با انديس e چيه؟
اون مدت زمان پيمودن شعاع هاي دوستاره است؟
پس اين دو چه فرقي دارن؟
در حالي كه t با انديس t پايين منحني نوريه و t با انديس e بالا(دهانه) منحني نوريه.
ممنون ميشم اگر پاسخ بدين:):گل

starscream4002
09-24-2013, 05:26 PM
سلامي دوباره خدمت دوستان
ما اينقدر اصرار و پافشاري ميورزيم تا به جواب اشكالاتمون برسيم:دي
سوالي كه داشتم در مورد گرفت هاست.
١. اگر شعاع دو ستاره برابر باشه گرفت جزئي نميتونيم داشته باشيم؟اگر داريم، منحني نوري اش چه شكلي ميشه؟
٢. توي منحني نوري t با انديس t مدت زمان گرفت هست؟
t با انديس e چيه؟
اون مدت زمان پيمودن شعاع هاي دوستاره است؟
پس اين دو چه فرقي دارن؟
در حالي كه t با انديس t پايين منحني نوريه و t با انديس e بالا(دهانه) منحني نوريه.
ممنون ميشم اگر پاسخ بدين:):گل

1. اگر شعاع دو ستاره برابر باشد، می توانیم هر نوع گرفت داشته باشیم. فقط در این حالت گرفت کامل 1 لحظه است که آن هم در میل مداری دقیقا 90 درجه می تواند رخ دهد. در این حالت تغییرات منحنی نوری تیز نیستند و انحنا دارند. به جز حالتی که میل مداری 90 است که در این صورت افت های نمودار به صورت دره های تیز هستند.
2. از توضیحتان به نظر میاد که t با اندیس t مدت زمان گرفت کامل است ولی t با اندیس e مدت زمان کل گرفت است.

موفق باشید،

س.ا.

storm
09-26-2013, 05:13 PM
سلامي دوباره خدمت دوستان
چندين سوال داشتم...
١. ستاره هايي كه مدارشون دايره ايه، چرا بيضي ديده ميشن؟ اگر ممكنه كامل توضيح بدين:گل
٢.توي فرمول مربوط به دوتايي هاي طيف نمايي، دقيقا علت وجود ضريب sin^3 رو متوجه نميشم در قانون سوم كپلر...؟
و چند سوال مكانيك سماوي هم داشتم:
١. توي اثبات فرمولb^2=a^2(1-e^^2)علت برابري a و r چيه؟
٢.ايا جرم كاهيده دقيقا چيه؟جز m1m2/m1+m2 ؟
٣.براي محاسبه جاذبه بين يك جسم نقطه اي و يك جسم گسترده، در نهايت پس از حل انتگرال ها به اين نتيجه ميرسيم كه مثل اجسام نقطه اي در نظر گرفته ميشن؟ يعني جرمشون در مركزشون در نظر گرفته ميشه؟
باتشكر:گل

celestial boy
09-26-2013, 07:45 PM
سلام.

1-این به خاطر تصویر شدنه.یعنی چون صفحه مدار دوتایی با صفحه آسمون زاویه داره مدار بر صفحه آسمون تصویر میشه.چی میشه که بیضی دیده میشه؟کافیه دوتا قطر عمود بر هم از دایره رو تصویر کنید.اگه یکی از قطر ها فصل مشترک دوتا صفحه باشه طبیعیه که تغییری نمی کنه.(خودش تصویر خودشه) اما قطر دیگه کوچیکتر میشه.(با یک ضریب کسینوس)

2-اون ضریب به خاطر مولفه سرعت در راستای دید ظاهر میشه.اثبات کاملش:مادرن صفحه 187-188

1-طبق تعریف بیضی مجموع فواصل هر نقطه روی بیضی از دوتا کانون برابر 2a هست.قبول دارید توی اون نقطه فاصله از دوتا کانون با هم برابره؟پس مسئله حله!
2-جرم کاهیده یه موجودیه که خیلی جا ها به درد میخوره.وقتی می خوایم معادلاتو از دید یکی از دو جرم بنویسیم از این استفاده می کنیم.(پیچیدس یه خورده من هنوز شهود کامل ندارم بهش)
3-خب وقتی انتگرال گرفتیم به بازه هاش میرسیم.سه تا نقطه در نظر می گیریم.داخل،رو و بیرون پوسته.داخل پوسته نیرو صفره.روی پوسته و بیرون پوسته مثه همون جرم نقطه ایه.(البته پتانسیل فرق داره)

hodhod96
10-23-2013, 11:50 PM
سلام.من می خواستم بدونم که در کل شرط همزمان طلوع کردن خورشید و شعرای یمانی چیه؟؟

starscream4002
10-24-2013, 12:24 PM
سلام.من می خواستم بدونم که در کل شرط همزمان طلوع کردن خورشید و شعرای یمانی چیه؟؟

فقط کافیه st هر دو جسم در زمان طلوع یکسان باشه.